DocScanner 18-Sep-2022 14-06

You might also like

Download as pdf or txt
Download as pdf or txt
You are on page 1of 205

CoN NT S

CHAPTER NO. CHAPTER NAME PAGE N0.

1 Introduction to Financial Accounting 1

2 Accounting Process 13

3 Financial Statements 35

Preparation of Financial Statements 57

79
5 Financial Reporting Standards I

93
6 Financial Reporting Standards II

105
7 Corporate Financial Statements

141
8 Statement of Cash Flows

159
9 Analysis of Financial Statements I

183
10 Analysis of Financial Statements II

199
11 Case Studies
1
CHA PTE R
INTRODUCTION TO FINANCIAL ACCOUNTING

CONTENTS

1.1 Introduction
1.2 Accounting
1.3 Users and Uses of Accounting Information
Self-Assessment Questions
Activity
1.4 Sub-Fields of Accounting
1.5 Accounting Terms
1.5.1 Asset
1.5.2 Liability
1.5.3 Capital/Owners' Equity
1.5.4 Revenue
1.5.5 Cost
1.5.6 Expense
1.5.7 Goods
1.5.8 Debtors (Accounts Receivable)
1.5.9 Creditors (Accounts Payable)
1.5.10 Debits and Credits
Self-Assessment Questions
1.6 Financial Statements
1.6.1 Income Statement
1.6.2 Balance Sheet
1.6.3 Statement of Cash Flow
Activity
1.7 Generally Accepted Accounting Principles
1.8 Advantages of Financial Accounting
1.9 Limitations of Financial Accounting
1.10 Summary
Key Words
1.11 Descriptive Questions
1.12 Answer Key
Self-Assessment Questions
1.13 Suggested Books and E-References
2 FINANCIAL ACCOUNTING AND ANALYSIS

INTRODUCTORY CASELET

MODERN COFFEE HOUSE

On January 2, 2019, Ashok and Ramesh decided to join hands to onen


Modern Coffee House (MCH) in Delhi. Ashok had been working with
a manufacturing company on the shop floor and had about 10 vears'
experience. Ramesh had been working on an ad-hoc basis with a multi-
national company for quite some time. He became the victim of down-
sizing and was finally retrenched. He sought retirement under Early
Separation Scheme from the company.
They contributed Rs. 2,00,000 each, hired a small premises on rent, pur
chased furniture for Rs. 1,05,000, utensils for Rs. 92,000, equipment for
Rs. 63,000 and made a security deposit of Rs. 1,05,000 with a soft drinks
company. They did not keep proper accounting records but just main-
tained a cash register and day a book. At the end of June 2019, they
found their assets, liabilities and other items as under:

Rs. Rs.
Cash 25,000 Bank balance 100,000
Utensils 92,000 Equipment 63,000
Sale proceeds 260,000 Rent paid 15,000
Total expense on 155,000 Furniture 105,000
food & beverages
Security deposit 105,000
They thought that they ad only Rs. 125,000 left in cash and bank
balance and therefore, their capital had been reduced by Rs. 275,000
representing the loss made by the business during the period of
6 months. They had to take a
decision whether to continue running the
coffee house.

QUESTIONS
1. Analyze the caselet and find out the prevailing situation of
Modern Coffee House. (Hint: Profit or los is determined by
preparing the statement of profit and loss and not on the basis
of cash and bank balance.)
2. Examine the decision whether to continue running the cOL
house or not.
LEARNING OBJECTIVESs

After reading this chapter, you will be able to:


Understand the role of accounting information in making eco-
nomic decisions.
Identify the users and uses of accounting information.
Understand the sub-fields of accounting and their relevance.
Explain different accounting terms.
Describe the contents and the purpose of different financial statements.
Understand the purpose of generally accepted accounting principles.

1.1 INTRODUCTION
Decision making is a part and parcel of carrying on a business. There are
many stakeholders in a business enterprise. These include owners, man-
agers, investors, lenders, customers, suppliers, labor unions and the
government. All these stakeholders make some or the other kind of decision.
For making decisions, the stakeholders need relevant economic information.
"Accounting" provides the relevant economic information required by
stakeholders.

1.2 ACCOUNTING
Accounting has been aptly defined by the American Accounting Association as:
Accounting is the process of identifying, measuring and communicating eco
nomic information to permit informedjudgments and decisions by the users
of accounts.
This definition implies that there are certain users of accounts who need
information for judgment and decision making, and accounting is a process
of identifying users' information requirements and collecting, processing
and communicating such information to the users.

1.3 USERS AND USES OF ACCOUNTING


INFORMATION
Accounting information has both intermal and external users. Internal users are
managers in all areas of functional responsibility such as marketing, finance,
human resources and general management. Marketing managers use account:
ing information to make decisions relating to pricing of products, sales promo-
1on, ete. Finance managers use accounting information to decide on making IMPORTANT CONCEPT
new investments, raising funds, payment of dividends, etc. Human resources
managers make decisions relating to pay revision, declaration of bonus, etc. on Accounting information has
the basis of accounting information. General managers make decisions on the both internal (managers) and
product-mix of the entity using accounting information. The type of reports external (investors, lernders,
generated by the accounting information system for use by managers include customers, suppliers, labour
orecasts of income, projections of funds requirement and availability, compar unions and government users).
1son of financial results of alternative courses of action, ete.
External users of accounting information include investors, lendo
lers,
tomers, suppliers, labor unions and the government. Owners aadl
are interested in knowing whether the business would be able to to provid
a reasonable return on their investment and whether to continno
continue with
investment in the business, how to finance the expansion of business, the
Lenders need information for determining the capacity of the busina
pav interest and to repay loans in time. Customers want to know whether H

business will continue producing the item they are using so that there ar
no problems relating to servicing of its products and associated warranti
Suppliers want to satisfy themselves about the ability of the business t
make payments of their dues on time. Labor unions are interested in kno
ing whether the business will be able to pay increased wages and bonuse
Government wants to know whether the business is rightly determining it
profit or loss and whether it is duly paying the taxes due from it.

SELF-ASSESSMENT 1. The group of users of accounting information charged with achieving


QUESTIONS
the goals of the business is its
a. auditors b. investors
C. managers d. creditors

2. Which of the following groups uses accounting information to


determine whether the company can pay its obligations?
a. Investors in common stock
b. Marketing managers
C. Creditors
d. Chief Financial Officer
to
3. Which of the following groups uses accounting information
determine whether the company's net income will result in a stock
price increase?
a. Investors in conmmon stock
b. Marketing managers
c. Creditors
d. Chief Financial Officer

ACTIVITY 1 Would you advise Ashok and Ramesh to close down the coffee
house or to
take external advice?

1,4 SUB-FIELDS OF ACCOUNTING


sub-
-QUICK TIP Information needed by managers and owners is more detailed, and tne erial
field of accounting that generates this information is known as mantuae
Financial accounting and accounting. Managers use managerial accounting information to set makeo
Managerial accounting are nizational goals, evaluate individual and departmental performances mar
two important sub-fields of decisions relating to the introduction of new products or entering nte
accounting. par
kets, etc. Managerial accounting information need not be organlzed or
ticular format. The presentation depends on the decision at hand. A
Dart of managerial accounting consists of information relating to the cost of
Droducts and services. Managerial accounting uses both historical informa-
tion and projections for the future.
The other sub-field of accounting is called financial accounting. It relates to
thepreparation of financial statements for use by both managers and exter-
nal stakeholders. Financial accounting reports present information about all
activities of the business, be it operating activities (main revenue-producing
activities); investing activities (activities involving purchase and sale of long-
lived assets and investments); or jfinancing activities (activities that change
the amount and composition of financial resources). Financial accounting is
basically historical in nature.

1.5 ACCOUNTING TERMS


The following terms commonly used in financial accounting are of interest to
different users of accounting information:

1.5.1 ASSET
Assets are economic resources controlled by an entity whose cost (or fair
value) at the time of acquisition could be objectively measured. A resource is
an economic resource if it provides future cash flows to the entity. An asset
can be: (i) cash or something convertible into cash (e.g. accounts receivable),
G) goods expected to be sold and cash received from them and (ii) items to
be used in future activities that will generate cash flows.
Land and building, plant and machinery, furniture and fixtures, inventories,
debtors and cash balance are examples of assets.

1.5.2 LIABILITY
Liabilities are claims to assets. A business raises financial resources from
both its owners and outside parties. Both have claims to the assets of the
entity. Liabilities are claims to assets of parties other than owners. Loans,
debentures (bonds), creditors, unpaid expenses are examples of liabilities.
Liabilities create negative future cash flows for the entity.
For example, a business has assets worth Rs. 10 million which are financed
by owners' funds of Rs. 6 million and loans of Rs. 4 million. The loan of
Rs, 4 million represents a claim to 40 percent of the assets and is termed as a
liability of the business.

1.5.3 CAPITAL/OWNERS' EQUITY


Capital (owners' equity) generally refers to the amount invested in an enter
prise by the owners. It is also used to refer to the claim of owners to the assets
of an enterprise. The claims of owners to assets are secondary to thOse of
creditors and lenders.
Changes in owners' equity occurs when: (i) owners either invest in or with-
draw cash or other assets from the business and (ii) the business either
earns income from profitable operations or incurs losses from unprofitable
operations.
1.5.4 REVENUE
Revenue is the gross inflow of cash, receivables or other consideration ari
s
ing during the course of ordinary activities of an enterprise from the sa
of goods, rendering of services, and from the use by others of enter
resources yielding interest, royalties and dividends.

1.5.5 COST
Cost is a monetary measurement of the amount of resources used for son
purpose. For example, an entity incurs a cost when it purchases an item
equipment.

1.5.6 EXPENSE
All costs incurred by an entity are not expenses. An expense is that co
which relates to the operations of an accounting period (e.g. rent) or to tE
revenue earned during the period (cost of goods sold) or the benefits of whic
do not extend beyond that period. Expenses, thus, have a relation witht
accounting period and represent that part of the cost of an asset or servie
that is consumed during the accounting period.
For example, a businessman dealing in televisions buys 1,000 televisi-
sets at a cost of Rs. 20 million during an aceounting year. This amount
QUICK TIP
Rs. 20 million is a cost as it represents the amount of resource (cash) use
An expense is a cost that During this accounting period, the businessman sells only 800 television
satisfies certain conditions. The cost of 800 televisions, that is, Rs. 16 million is the expense of th
accounting year as it represents the cost that corresponds to the revent
earned during the year from the sale of 800 televisions.
Abusiness that prepares its accounts every calendar year(January-Decembe
buys an yearly insurance cover on its assets on 1 April by paying a premu
of Rs. 50,000. This anmount of Rs. 50,000 is a cost as it represents the amount
resource (cash) used. However, the business will not enjoy the entire benefit
this cost in the accounting period that ends on 31 December. The benefit ofth
insurance cover extends to 31 March of the next accounting period. Onlythre
fourth of this cost relates to 9 months of the current accounting period, that i
Rs. 37,500 will be treated as an expense of the current accounting period.

1.5.7 GOODS
The term 'Goods' refers to the property in which the business deals. Goo
are purchased by a business for resale and not for use in the business. F
example, furniture acquired for resale by a furniture dealer will be treate
wI_
as goods and furniture acquired by such a dealer for use in his/her office
be treated as an asset.

1.5.8 DEBTORS (ACCOUNTs RECEIVABLE)


Debtor refers to a person who owes money to the business for goods p
chased from the business.
1.5.9 CREDITORS (ACCOUNTS PAYABLEE)
o0d
Creditor refers to the person to whom the business owes money for go
purchased by the business from that person.
1.5.10 DEBITS AND CREDITTS
The accounting system keeps a separate record for each item of assets, lia-
bilities, income and expense. This record is called an account. An account
has two sides, the left-hand side and the right-hand side. Accounting records
are maintained using a double-entry accounting system. Under this system,
debit and credit entries of equal amount are made to record every business
transaction. Entering anamount on the left-hand side of the account is called
debiting the account and entering an amount on the right-hand side of an
account is called crediting the account. Accounting records are considered
accurate only when the sum of all debits is equal to the sum of all credits.

SELF-ASSESSMENT
4. Which of the following is the most appropriate and modern definition
QUESTIONS
of accounting?
a. The information system that identifies, records and communi-
cates the economic events of an organization to interested users.
b. A means of collecting information.
c. The interconnected network of subsystems necessary to operate
a business.
d. Electronic collection, organization, and communication of vast
amounts of information.
5. The common characteristic possessed by all assets is_
a. long life b. great monetary value
c. tangible nature d. future economic benefit
6. Resources owned by a business are referred to as
a. owners' equity b. liabilities
C. assets d. revenues
7. Debts and obligations of a business are referred to as
a. assets b. equities
c. liabilities d. expenses

1.6 FINANCIAL STATEMENTS


Information to users of accounting information is provided in the form of
financial statements that arrange assets, liabilities, revenue and expenses in
different ways. Every business enterprise generally prepares three financial
statements: income statement, balance sheet and statement of cash flows.

1.6.1 INCOME STATEMENT


The income statement (or the profit and loss account) reports the result of
business operations during the accounting period. It matches the expenses
for the accounting period with the revenues earned, and reports the result
ing net income (profit or loss), The income statement is of particular use to
investors, lenders and creditors. Investors use the past net income as a basis
to predict the future net income and to make their investment decisions.
Lenders to the business use information provided by the income statenment
to form an opinion about the ability of the business to repay loans and to
pay interest on time. Creditors use the income statement to form an opinion
about the ability of the business to pay their dues on time.

NMIMS Global Access- School for Continuing Education


1.6.2 BALANCE SHEET
! IMPORTANTCONCEPT
The balance sheet reports the financial position of the business at a partict.
The accounting equation lar point of time, generally at the end of the accounting period. It shows tho
that always holds is Assets = amount of assets owned by the business and the claims on these assets. The
claims on the assets belong to the providers of
resources to buy the assets
Owners' Capital +Liabilities on
i.e., the owners and creditors/lenders. As the business cannot spend more
called
buying assets than the resources it has, the following relationship (also
the accounting equation) always holds.
Assets = Owners' Capital + Liabilities
of
Creditors and lenders analyze the balance sheet to understand the ability
the business to repay their dues. The proportion of owners' capital in relation
to outside liabilities also serves as an indicator of the financial strength the
of

business.

1.6.3 STATEMENT OF CASH FLOW


The statement of cash flow provides information about the cash receipts
and cash payments during an accounting period. Particularly, it
shows the
sources from which cash is received, the uses to which cash is put and the
change in the cash balance during the accounting period.
The presentation of financial statements for external users depends on
sole-
the type of business organization. A business can be organized as
proprietorship (having a single owner), a partnership (with two or more
or
QUICK TIP
owners or partners) or a company (having a large number of owners
Only companies are required shareholders). Only companies are required by law to keep the prescribed
to prepare annual as well as set of books and to present their financial statements in the prescribed
quarterly financial statements. format. Other forms of business organization need to maintain their
accounts in a manner that enables determination of their income for
income tax purposes.
Normally, companies are required to prepare annual and quarterly finan-
cial statements. The quarterly financial statements are also known as interim
financial statements. Certain business organizations are, however, not
required to prepare interim financial statements for external reporting.

ACTrVITY 2 Find out the profit or loss made by Modern Coffee House during the
period of 6 months.

1.7 GENERALLY ACCEPTED ACCOUNTING


PRINCIPLES
Generally accepted accounting principles (GAAP) are a set of conven-
tions, rules and procedures that define the accepted accounting practice
at a particular time. These result from a broad agreement on the theory
and practice of accounting at a particular time. The purpose of GAAP is to
ensure that the information provided in the financial statements is reliable
and understandable to the users. The users should be able to meaningfully
compare the current performance of a business entity with its past per
formance and the performance of other business entities. The GAAP keep
changing from time to time as the circumstances or the information needs
of the users change.
In India, the sources of GAAP include the Companies Act, 2013, Indian
accounting standards and the pronouncements of the accounting profession.

ADVANTAGES OF FINANCIAL ACCOUNTING


1.8Reveals the financial performance of a business during a period
1. and its
financial position at the end of that period.
2. Provides relevant information to investors and lenders, both present
and prospective to take appropriate investment and lending decisions.

1.9 LIMITATIONS OF FINANCIAL ACCOUNTING


1. Provides only historical information about the performance and
financial performance of business. It fails to provide estimates and
projections for future which form the basis of business decisions.
2. Financial accounting provides information about matters that can
be quantified. Many other items such as quality of management are
important for the success of a business. Since these items cannot be
quantified, these are not reported by Financial Accounting

1.10 SUMMARY
Understand the role of accounting information in making economic
decisions. There are a number of stakeholders in a business who
make some or the other kind of decision. For making these decisions,
the stakeholders need relevant economic information. It is account-
ing that provides the relevant economic information required by the
stakeholders.
Identify the users and uses of accounting information. Accounting
information includes both internal and external users. Managers are
internal users. Investors, lenders, customers, suppliers, labor unions and
the government are external users.
Understand the sub-fields of accounting and their relevance. There
are two sub-fields of accounting: managerial accounting and financial
accounting. Managerial accounting generates detailed information for
owners and managers. On the contrary, financial accounting relates to
the preparation of financial statements for use by both managers and
external stakeholders.
Understand the purpose of generally accepted accounting principles.
Generally accepted accounting principles (GAAP) are a set of conven-
tions, rules and procedures that define the accepted accounting practice
at a particular time. These result from a broad agreement on the theory
and practice of accounting at a particular time.
KEY WORDS 1. Accounting system keeps a separate record i1sfor each item of. assets
accoum
liabilities, income and expense. This record called an
account has two sides, the left-hand side and the right-hand sido
2. Accounting is the process of
identifying, measuring an-
Communicating economic information to permit informo
judgments and decisions by the users of accounts.
on
by an entity whose
3. Assets are economic resources controlled
acquisition could be objectival
(or fair value) at the time of
measured.
to an amount invested in a-
4 Capital/owners' equity generally refers
enterprise by the owners.
of the amount of resources used fo
5. Cost is a monetary measurement
some purpose.
6. Credit results from entering an
amount on the right-hand side of a
account.
7. Creditors (accounts payable) are
persons to whom the busines
owes money for goods purchased by the
business.
left-hand side
Debit results from entering an amount on the
of am
8.
account.
owe money to th
9. Debtors (accounts receivable) are persons who
business for goods purchased.
Expense is the cost relating to the operations of an
accounting
10.
to the benefits
period or to the revenue earned during the period or
of which do not extend beyond that period.

1.11 DESCRIPTIVE QUESTIONS


1.Define accounting.
2. How does managerial accounting differ from financial accounting?
3. How does an expense differ froma cost?
4. Who are the main users of accounting information and how do theyus
this information?
5. Define the terms revenue, asset and liability.

1.12 ANSWER KEY


SELF-ASSESSMENT QUESTIONS

Topics Q. No. Answers


Users and Uses of 1. C. managers
Accounting Information
2. c. Creditors
3. a. Investors in common stock

NMIMS Global Access- School for Continuing Education


INTRODUCTION TO FINANCIAL ACCOUNTING 11

Q. No. Answers
Topics
Accounting Terms 4. a. The information system that identi
fies, records and communicates the
economic events of an organization to
interested users
5. d. future economic benelit
6. C. assets
7. C. liabilities

1.13 SUGGESTED B0OKS AND E-REFERENCES


SUGGESTED BOOKS

o Anthony, R.N., Hawkins, D.E. and


Merchant, K.A. (2015). Accounting
Text and Cases, Tata McGraw
Hill.

Bhattacharyya, A.K. Financial Accounting for Business


(2014).
Managers, Prentice Hall of India.

E-REFERENCES
to
Horngren C.T, Sundem G.L. and Elliot JA. (2013). Introduction
Financial Accounting and Analysis, Pearson Education.

DKhan M.Y. and PK. (2010). Management Accounting: Text, Problems


Jain,
and Cases, Tata McGraw Hill (KJ).

NMIMS GIlobal Acces5-School lor Continuing Dducation


2
CHA PTE R
ACCOUNTING PROCESS

CONTENTS

2.1 Introduction
2.2 Steps in the Accounting Cycle
Self-Assessment Questions
2.3 Analysis of Accounting Transactions
Self-Assessment Question
Activity
2.4 Accounting Records
2.4.1 Account
2.4.2 Journal and Ledger
2.4.3 Subsidiary Books
Self-Assessment Questions
2.5 Summary
Key Words
2.6 Descriptive Questions
2.7 Answer Key
Self-Assessment Questions
2.8 Suggested Books and E-References
INTRODUCTORY CASELET

STATE BANK OF INDIA

You have a savings bank account with State Bank of India. never
vou want to know the balance of money in your account, you are able
to instantly find it online. To ensure the accuracy of the balance in vou
account and availability of an updated balance in your account at all
times, the bank needs to have a proper accounting system.
Hint: For any business other than banking, transactions recorded
at the time of their occurrence are transferred to the ledger accounts
after an interval according to the convenience of the business. But, in
banking, there can be no gap between the initial recording and subse-
quent transfer. Why?)
LEARNING OBJECTIVES

After reading this chapter, you will be able to:


Understand the accounting process that leads to the preparation of
financial statements.
Analyze the effect of accounting transactions on the basic account-
ing equation.
Understand the use of an account in the process of building
accounting records.
Understand the rules of debit and credit in recording business
transactions in relevant accounts.
Understand how a journal is maintained and the concept of sub-
sidiary books.
Understand the posting of entries in the ledger.
Understand how trial balance is extracted and its purpose.

2.1 INTRODUCTION
An enterprise must have a proper accounting system for recording the efect QUICK TIP
of economic events such as purchase of raw materials, sale of goods, acqui-
Examples of economic events
sition and disposal of assets, ete. The final step in the accounting process is
are purchase of an item of
the preparation of financial statements. Financial statements, however, are
equipment, payment of
not prepared after every transaction. A continuous sequence of steps (called
salaries to employees.
accounting cycle) is followed to record, classify and summarize business
transactions in accounting records. The data in these accounting records is
then used to prepare financial statements. Accounting records are also used
for several other purposes.

2.2 STEPS IN THE ACCOUNTING CYCLE


The accounting cycle consists of the following steps:
1. Analysis of transactions in terms of their effect on assets, liabilities and
owners' capital.
2. Accounts are prepared for each item of assets, liabilities, revenues
and expenses using the rules of debit and credit. These accounts are
maintained in a record called ledger. Entries can be made directly
into the ledger or another intermediate record called journal. In
Some cases, the journal is subdivided into a number of journals called
subsidiary books.
3. Closing balances of all accounts are transferred to a statement called
trial balance.
At the end of the accounting period, some adjustment entries are made
and an adjusted trial balance is prepared.
. Financial statements are prepared using the information in the adjusted
trial balance.
6. Certain closing entries are passed and an after-closing trial balance is
prepared.
This chapter covers the accounting cycle up to the stage of preparation of
trial balance. The remaining steps are covered in the next chapter

SELF-ASSESSMENT
QUESTIONS 1. A purchased a car for Rs. 500,000, making a down payment of
Rs. 100,000 and signing a Rs. 400,000 bill payable due in 3 months. As
a result of this transaction
a. total assets increased by Rs. 500,000
b. total liabilities increased by Rs. 400,000
C. total assets increased by Rs. 400,000
d. total assets increased by Rs. 400,000 with corresponding increase
in liabilities by Rs. 400,000
2. Capital brought in by the proprietor is an example of,
a. increase in asset and increase in liability equity
b. increase in liability and decrease in asset
C. increase in asset and decrease in liability
d. increase in one asset and decrease in another asset
3. A transaction results in a Rs. 90,000 decrease in both assets and
liabilities. The transaction could have been a
a. repayment of bank loan of Rs. 90,000
b. collection from debtors of Rs. 90,000
C. purchase of an item of equipment for Rs. 90,000
d. sale of an item of equipment for Rs. 90,000

2.3 ANALYSIS OF ACCOUNTING


TRANSACTIONS
An accounting transaction occurs when an economic event causes a change
in the assets, liabilities or owners' capital. Examples of economie events
are purchase of an item of equipment, payment of salaries to employees.
IMPORTANT CONCEPT Appointment ofa manager is not an economic event as no change occurs in
the assets, liabilities or owners' capital as a result of this event.
An accounting transaction
OcCurs when an economic All accounting transactions are analyzed in terms of their effect on
event causes a change in the assets, liabilities and owners' capital. Since the basic accounting equation
assets, liabilities or owners provides a relationship between assets, liabilities and owners' capital,
capital. the accounting transactions can be analyzed using the basic accounting
equation.
QUICK TIP
Since the accounting equation must balance, each transaction has a dual
Allaccounting transactions can effecton the accounting equation. An inerease in an asset must be matched
by a decrease in another asset, or an increase in a liability or an increase in
be analyzed using the basic
accounting equation. Each
owners capital. On purchase of furniture, either the cash balance will be
transaction has a dual effect on reduced or a liability to the supplier will increase. Alternatively, only a part
the accounting equation. of the cost of furniture may be paid in cash and the balance reflected as an
increase in liabilities.
Illustration 2.1
Consider the following transactions pertaining to A's business:
1. Started business with cash Rs. 300,000.

2. Purchased goods for cash Rs. 120,000.


3. Purchased goods on credit Rs. 60,000.
4. Purchased furniture for cash Rs. 20,000.
5. Deposited Rs. 50,000 in the bank account.
6. Sold goods costing Rs. 5,000 for Rs. 18,000, on credit.
7. Sold goods costing Rs. 30,000 for Rs. 36,000, in cash.
8. Paid rent Rs. 10,000 and salaries Rs. 20,000.
9. Withdrew Rs. 15,000 from the bank account to pay for private
expenses.
10. Received cash against goods sold on credit Rs. 18,000.
Each transaction can be analyzed in the following manner:
o Transaction 1: The business received cash of Rs. 300,000; it is an asset
to the business. The business owes this amount to A, the proprietor,
and therefore, it also represents the capital of the business. Capital
of Rs. 300,000 is equal to assets of Rs. 300,000.
a Transaction 2: Purchase of goods for cash increases goods (an asset)
and reduces cash (another asset). The accounting equation remains
the same as after transaction 1.
Transaction 3: Purchase of goods on credit increases goods (an asset)
and simultaneously increases creditors (a liability). The sum of liabil-
ities and capital is now Rs. 360,000 matched by assets of Rs. 360,000.
Transaction 4: Purchase of furniture for cash increases an asset (fur-
niture) and reduces another asset (cash). The accounting equation
remains the same as after transaction 3.
Transaction 5: Deposit of cash in the bank account increases one
asset (balance in the bank account) and reduces another asset (cash).
The accounting equation remains the same as after transaction 4.
Transaction 6: Sale of goods costing Rs. 15,000 for Rs. 18,000 on credit
decreases an asset (goods) by Rs. 15,000 and increases another asset
(debtors) by Rs. 18,000. The difference of Rs. 3,000 between the two
amounts is profit. The profit belongs to the proprietor and increases
his/her business capital. After this transaction, the liabilities are
Rs. 60,000, capital is Rs. 3,03,000 and assets are Rs. 363,000.
Transaction 7: Sale of goods costing Rs. 30,000 for Rs. 36,000 on cash
basis decreases an asset (goods) by Rs. 30,000, increases another asset
(cash) by Rs. 36,000 and also increases the capital by the amount of
profit (Rs. 6,000). After this transaction, the liabilities are Rs. 60,000,
capital is Rs. 309,000 and assets are Rs. 369,000.

(Continued)
Transaction 8: Payment of rent Rs. 10,000 and salaries Rs. 20.000
decreases an asset (cash) by Rs. 30,000 and also decreases capital bu
Rs. 30,000. After this transaction, the liabilities are Rs. 60,000, capital
is Rs. 279,000 and assets are Rs. 339,000.
Transaction 9: Withdrawal of Rs. 15,000 from the bank account for
meeting private expenses decreases one asset (bank balance) and
also reduces capital by Rs. 15,000. After this transaction, the liabil
ities are Rs. 60,000, capital is Rs. 264,000 and assets are Rs. 324,000
Transaction 10: Receipt of cash against goods sold on credit
increases one asset (cash) and reduces another asset (debtors) by
the same amount. The accounting equation remains the same as
after transaction 9.
The accounting equation after different transactions can be presented
as in Table 2.1.

TABLE 2.1 ACCOUNTING EQUATION FOR


DIFFERENT TRANSACTIONS
Assets Liabilities Capital
No. Transaction (Rs.) (Rs.) (Rs.)
300,000 300,000
1. Started business with cash
Rs. 300,000
300,000 300,000
2. Purchased goods for cash
Rs. 120,000
360,000 60,000 300,000
3 Purchased goods on credit
Rs. 60,000
360,000 60,000 300,000
4. Purchased furniture for
cash Rs. 20,000
60,000 300,000
5. Deposited Rs. 50,000 in the 360,000
bank account
363,000 60,000 303,000
6. Sold goods costing
Rs. 15,000 for Rs. 18,000,
on credit
60,000 309,000
7. Sold goods costing 369,000
Rs. 30,000 for Rs. 36,000,
in cash
60,000 279,000
8. Paid rent Rs. 10,000 and 339,000
salaries Rs. 20,000
60,000 264,000
9 Withdrew Rs. 15,000 from 324,000
the bank account to pay for
private expenses
60,000 264,000
10. Received cash against 324,000
goods sold on credit
Rs. 18,000

The simple accounting equation in Illustration 2.1 can be expanded to show


the effect of business transactions on specific assets and liabilities.
SELF-ASSESSMENT
4. Indicate which alternative in each of the following cases is consid- QUESTION
ered to be correct:
) The liabilities of a firm are Rs. 30,000. The capital of the proprie-
tor is Rs. 70,000. The total assets are
a. Rs. 70,000
b. Rs. 100,000
C. Rs. 40,000
i) The assets of the business as on March 31, 2015 are worth
Rs. 500,000 and its capital is Rs. 350,000. Its liabilities on that
date shall be
a. Rs. 850,000
b. Rs. 150,000
C. Rs. 350,000
(iii) The accounting equation states that
a. Capital = Assets + Liabilities
b. Capital + Liabilities = Assets
=
C. Assets + Liabilities Capital
(iv) The owners' equity (i.e., capital) shall stand increased by.
a. proprietor's drawings
b. purchasing furniture on eredit
C. profit earned during the accounting year

ACTvITY 1
Analyse the following transactions in terms of their effect on assets, lia-
bilities and capital using the accounting equation. The starting capital is
Rs. 200,000.
1. Purchased goods for cash Rs. 100,000
2. Purchased goods on credit Rs. 50,000
3. Paid salaries Rs. 10,000

IMPORTANT CONCEPT
2.4 ACcOUNTING RECORDS !
The accounting system
2.4.1 ACCOUNT keeps a separate record for
The accounting system keeps a separate record for each item that appears each item that appears in
in the financial statement. This record is called an account. The account for the financial statement. This
any item records increases and decreases in that item as a result of business record is called an account.
Accounts can be classified
transactions and determines the balance of that item at any time after one or
more transactions affecting that item have taken place. into the following categories:

For example, a person starts a business with say Rs. 50,000. In this case, his/ Asset accounts
her capital is Rs. 50,000 and assets in the form of cash are also Rs. 50,000. Liability accounts
Transactions entered into by the firm will either increase the cash balance Capital accounts
(e.g., transactions such as sales for cash and collections from customers, etc.) Expense accounts
or decrease the cash balance (e.g., payment for goods purchased, salaries, rent,
etc.). The cash balance can be changed with every transaction by erasing the Income accounts

NMIMS Global Access School for Continuing Education


amount for each transaction. This is quiteut
old amount and entering the new all the transa
it is convenient if
cumbersome and time consuming. Instead, in t
to an increase or decrease in cash are collected togetherSUm
tions that lead adding the m of
balance can be calculated by
cash account. The new cash subtracting the sum of decreasee
starting balance and then
increases to the
English letter p
two columns. It resembles the
An account has a title and left-hand side of the account is called th
"T Account'. The
and is called a illustration of
side is called the credit side. An
debit side and the right-hand
Table 2.2.
an account is shown in
AN ACcOUNT
TABLE 2.2 SIMPLE FORM OF
Cash
(Rs.) Credit Side (Rs)
Debit Side
40,000 Decreases 8,000
Starting balance
4,400 2,000
Increases 4,000
2,000
1,000 1,500

300
47,700 15.500

New balance 32,200

the
of cash on the left-hand side and
What we have done is to put the increase
form in which an account can
decrease on the right-hand side. An alternate
2.3.
be presented is given in Table
PRESENTINGAN ACCOUNT
TABLE 2.3 ALTERNATE FORM OF
Particulars Reference Amount
Date Particulars Reference Amount Date

Note: Reference indicates the source of information.

CLASSIFICATION OF ACCOUNTS
(i) asset accounts,
Accounts can be classified into the following categories: accounts ana
(ii) liability accounts, (ii) capital accounts, (iv) expense
(v) income accounts.
IMPORTANT CONCEPT

Under the double-entry RULES FOR DEBIT AND CREDIT


accounting system, debit syse
Accounting records are maintained using a double-entry accounting tu
and credit entries of equal Under this system, debit and credit entries of equal amount are made
amount are made to record of a
record every transaction. Entering a transaction on the left-hand side
every transaction
account is known as debiting the account and entering a transaction on te
right-hand side is called crediting the account. The rules of debit and ere
differ with the account type, and are as follows:
1. Inereases in assets are debits; decreases in assets are credits.
2. Inereases in liabilities are credits; decreases in liabilities are debits.
3. Increases in owners' capital are credits; decreases in owners' capital QUICK TIP
are debits.
Expenses and losses are debits; incomes and gains are credits. To identify the correct debit
4,
or credit, one needs to first
Accounts can also be classified into the following categories: determine the accounts that
1. Personal accounts: These accounts relate to persons. Accounts of are involved in a transaction.
customers, suppliers, lenders and bankers fall in this category. The Then, one has to identify which
capital account of the owners is also a personal account. Accounts with account has increased or
decreased and apply the debit/
a prefix or suffix, such as prepaid rent, salaries outstanding, etc. are
credit rule.
treated as personal accounts.
2. Real accounts: These accounts relate to assets of the firm such as land,
building, investments, fixed deposits, cash balance, etc.
3. Nominal accounts: These accounts relate to expenses, revenues, TIP
losses and gains. Salaries paid, interest paid, commission received are
(QUIcK
examples of nominal accounts. Nominal accounts are temporary only Entering a transaction on the
as their net result is reflected as profit or loss, which is transferred to left-hand side of an account
is known as 'debiting the
the capital account.
account'and entering a
For this alternative classification of accounts, the following rules of debit and transaction on the credit side is
credit are followed: called'crediting the account
1. For personal accounts, debit the receiver and credit the giver
2. For real accounts, debit what comes in and credit what goes out. PQUICK TIP
3. For nominal accounts, debit all expenses and losses and credit all Increases in assets are debits;
incomes and gains. decreases in assets are credits;
Increases in liabilities are
Illustration 2.2 credits; decreases in liabilities
are debits; Increases in owners
capital are credits; decreases in
A, after starting business on January 1, 2016, with cash of Rs. 100,000,
owners'capital are debits and
entered into the following transactions: Expenses and losses are debits;
Jan 3: Purchased machinery for Rs. 50,000 incomes and gains are credits.
Jan 5: Paid rent for the shop Rs. 2,000
Jan 31: Paid salary to employee Rs. 3,000
Jan 31: Received commission Rs. 15,000
These transactions can be analyzed in terms of accounts involved
(assets, liabilities, expenses, incomes) and debit/credit rules as in Table 2.4.

TABLE 2A ANALYSIS OF TRANSACTIONS


Date Accounts Type of Debit Credit
(2016) Particulars Involved Account Effect (Rs.) (Rs.)
Jan 1 Rs. 100,000 Cash Asset Increased 100,000
cash A's capital Capital Increased 100,000
invested in
business
Jan 3 Purchased Machinery Asset Increased 50,000
machinery Cash Asset Decreased 50,000
for Rs. 50,000
(Continued)
TABLE 2.4 ANALYSIS OF TRANSACTIONS CONTINUBn
Date Accounts Type of
(2016)
Debit
Particulars Involved Account Effect (Rs.)
Cre
Jan 5 Paid rent for Rent (Rs
Expense Increased 2,000
the shop Cash Asset Decreased
Rs. 2,000 2,000

Jan 31 Paid salary Salary Expense Increased 3,000


to employee Cash Asset Decreased
Rs. 3,000 3,000

Jan 31 Received Cash Asset Increased 15,000


commission Commission Income Increased
Rs. 15,000 15,00

The transactions in Table 2.4 can be analyzed in terms of the alternatit


classification of accounts involved (personal, real, nominal) and dehi
credit rules as in Table 2.5.
TABLE 2.5 ANALYSIS OF TRANSACTIONS USING ALTERNATIVE
CLASSIFICATION OF ACCOUNTS
Date Accounts Type of Debit Credi
(2016) Particulars
Involved Account Effect (Rs.) Rs)
Jan 1 Rs. 100,000 Cash Real Comes in 100,000
cash invested A's Capital Personal Giver 100,00
in business
Jan 3 Purchased Machinery Real Comes in 50,000
machinery for Cash Real Goes out 50,000
Rs. 50,000
Jan 5 Paid rent for Rent Nominal Expense 2,000
the shop Cash Real Goes out 2,000
Rs. 2,000
Jan 31 Paid salary Salary Nominal Expense 3,000
to employee Cash Real 3,000
Goes out
Rs. 3,000
Jan 31 Received Cash Real Comes in 15,000
commission Commission Nominal Income 15,000
Rs. 15,000

Based on Tables 2.4 and 2.5, the 'T" form accounts in Table 2.6 can be
prepared.

TABLE 2.6 LEDGER ACCOUNTS


Amount
Amount
Date Particulars Reference (Rs.) Date Particulars Reference (RS
Cash account
Increases Decreases
(Dr) (Cr:)
Jan 1 As Capital 100,000 Machinery 50,000
Jan 3
Jan 31 Commission 15,000 Rent 2,000
Jan 5
Jan 31 Salary 3,000
(Continued
TABLE 2.6 LEDGER ACCOUNTS-cONTINUED
Amount Amount
Date Particulars Reference (Rs.) Date Particulars Reference (Rs.)
A's capital account
Decreases Increases
(Dr) (Cr.)
Jan 1 Cash 100,000

Machinery account
Increases Decreases
(Dr) (Cr.)

Jan 3 Cash 50,000

Rent account
Increases Decreases
(Dr) (Cr:)

Jan 5 Cash 2,000


Salary account
Increases Decreases
(Dr.) (Cr.)
Jan 31 Cash 3,000
Commission account
Decreases Increases
(Dr.) (Cr.)
Jan 31 Cash 15,000

2.4.2 JOURNAL AND LEDGER !) IMPORTANT CONGEPT

The accounting record where all the accounts are kept together is called the
Transactions are first
ledger. It is also referred to as the principal book. Though accounts can be
entered in the Journal in
written directly in the ledger, it is common to use two records for the pur- a chronological order. The
pose. These are the journal and the ledger.
debit and credit amounts
recorded in the journal are
JOURNAL subsequently transferred
to the relevant accounts in
Transactions are first entered in this record to show the accounts to be deb-
ited and credited. Journal is also called a subsidiary book. Transactions are the ledger at convenient
intervals.
entered in a chronological order in the journal. The debit and credit amounts
recorded in the journal are subsequently transferred to the relevant accounts
in the ledger at convenient intervals.

ENTERING TRANSACTIONS INTO THE JOURNAL


The journal is prepared in the following manner (Table 2.7).

TABLE 2.7 FORMAT OF JOURNAL


Date Particulars L.E Dr. Amount Cr. Amount
At #h
1. In the first column, the date of the transaction is entered.,
year, the month and the
the year is written and belbw the ate at
written.

In the second column, the names of the accounts


involved are
2. writte
first with the word 'D whig
The account to be debited is written
stands for debit). It is written towards the end
of the column. n
next line, a little space is let, then the
word "To' is written, after uh
next
to be credited is written. In the
the name of the account
entry
'narration'is written which reters to the explanation for the beina

relating thereto. It starts with thew


made and the necessary details word

Being'
page
LE. refers to "Ledger Holio, which the
is
3. In the third column, c-

account in whieh the entry is written un


the ledger containing the
posted.
column.
to the debit amount. In this th
4. The fourth column refers
entered.
amounts to be debited are
amounts to be credited are entered.
5. In the fifth column, the

transactions in the journal is called


The process of entering the
journalizing.

Ilustration 2.3

during January 2016.


A has entered into the following transactions

(Rs)
Date (2016)
100,000
Jan 1 A started business with cash
75,000
Jan3 Deposited cash into the bank
500
Jan 4 Purchased stationary
20,000
Jan 5 Purchased goods for cash
25,000
Jan 7 Purchased goods from B on eredit
15,000
Jan8 Sold goods tor cash
18,000
Jan9 Sold goods to Con credit

Drew cash from bank


10,00
Jan10
24,500
Jan 15 Paid to B
500
Discount allowed by B

C 17,700
Jan 20 Received cash from

Discount allowed to C 300

2,000
Jan 31 Paid salary

The above transactions will appear in A's journal as shown in lable


TABLE 2.8 JOURNAL OF A
Dr Cr.
Date Amount Amount
(2016) Particulars L.E (Rs.) (Rs.)
Jan 1 Cash account Dr. 100,000
To Capital account 100,000
(being the amount invested by A in
his/her business)
Jan 3 Bank account Dr. 75,000
To Cash account 75,000
(being the amount deposited in bank)
Jan 4 Stationeryaccount Dr. 500
To Cash account 500
(being stationery purchased for cash)
Jan 5 Purchases account Dr. 20,000
To Cash account 20,000
(being goods purchased for cash)
Jan 7 Purchases account Dr. 25,000
To B's account 25,000
(being goods purchased from B3
on credit)
Jan 8 Cash account Dr. 15,000
To Sales account 15,000
(being goods sold for cash)
Jan 9 C's account Dr. 18,000
To Sales account 18,000
(being goods sold to C on credit)
Jan 10 Cash account Dr. 10,000
To Bank account 10,000
(being cash withdrawn from bank)
Jan 15 B's account Dr. 25,000
To Cash account 24,500
To Discount account 500
(being cash paid to B and discount
allowed by him/her)
Jan 20 Cash account Dr. 17,700
Discount account Dr. 300
To C's account 18,000
(being cash received from C and
discount allowed to him/her)
(Continued)
TABLE 2.8 JOURNAL OF A-CONTINUED
Dr. Cr.
Amount Amount
Date L.E (Rs.) (Rs.)
(2016) Particulars
account Dr. 2,000
Jan 30 Salaries
2,000
To Cash account
(being the amount paid for salary)

POSTING ENTRIES INTO THE LEDGER


the journal. The transfer of jour
Entries are posted into the ledger from
nal entry amounts to the ledger is
called posting. The journal shows the
QUICK TIP along with the amounts
account to be debited and the account to be credited
side on the left and the
The accounting record where involved. A ledger account has two sides: the debit
side on the right. For the account, which is to be debited, the entry wil.
the accounts are kept
all credit
together is called the ledger. made on the left-hand side of the account. The date of the transaction will
be
The transfer of journal entry be entered in the date column and the
particulars in the particulars column
amounts to the ledger is of the account. The particulars will be
preceded by the word To'.
called 'posting. made on the right-hand
side-
For the account to be credited, the entry will be
by the word 'By.
of the account. The particulars will be preceded

FINDING THE BALANCE INA LEDGER ACCOUNT


accounting period or a
To find the balance in an account at the end of the
any other time, the two sides of the account are
totaled and the difference
between the two is calculated. This difference
represents the balance in th
of the eredit side
account. The balance is a credit balance when the total tne
side. If the total ot
of the account is greater than the total of the debit ereaa
debit side is greater, the balance in the account is a
debit balance. T'he
where c/d stands to
balance is written on the debit side as "To balance c/d', B
Similarly, a debit balance is written on the credit sideasD
carried down'.
and are writte
balance c/d'. The totals of the two sides are now equal
QUICK TIP the two sides opposite each other. After determining
the total, the ere
where bd St
balance is written on the credit side as 'By balance b/d',
The balance in an account is a next accol
for brought down. It represents the starting balance of the
Credit balance when the
period, Similarly, the debit balance is written on the debit
side as to bal
total of the credit side of the
account is greater than the
b/d' in the next period.
amouns a
total of the debit side. ne revenue and expense accounts are not balanced, but their unts
accou
ransferred to the profit and loss account. This is the reason these acco
If the total of
the debit side
is greater, the balance in the
are also referred to as temporary accounts, while asset and liability
account is a debit balance. are referred to as permanent accounts.
balancing
The posting of transactions given in Ilustration 2.3 and the
ledger accounts is shown in Tables 2.9.
TABLE 2.9 POSTING OF TRANSACTIONS INTO LEDGER ACCOUNTS
LEDGER
Cash account
Dr. Cr.
Amount Amount
Date Particulars (Rs.) Date Particulars (Rs.)
2016 Jan 1 To Capital Alc 100,000 2016 Jan 3 By Bank A/c 75,000
8 To Sales A/c 15,000 4 By Stationary A/c 500
10 To Bank A/c 10,000 5 By Purchases AVc 20,000
20 To C's A/c 17,700 15 By B's Alc 24,500
31 By Salaries A/e 2,000
By Balance c/d 20.700
142,700 142,700
Feb 1
To Balance b/d 20,700

Capital account
Dr Cr.
Amount Amount
Date Particulars (Rs.) Date Particulars (Rs.)
2016 Jan 31 To Balance cld 100.000 2016 Jan 1 By Cash A/c 100.000
100.000 100.000
Feb 1 By Balance b/d 100,000

Bank account
Dr. Cr.
Amount Amount
Date Particulars (Rs.) Date Particulars (Rs.)
2016 Jan 3 To Cash A/c 75,000 2016 Jan 10 By Cash A/c 10,000
31 By Balance c/d 65,000
75,000 75,000
Feb 1 To Balance b/d 65,000

Stationary account
Dr. Cr.
Amount Amount
Date Particulars (Rs.) Date Particulars (Rs.)
2016 Jan 4 To Cash 500
(Continued
TABLE 2.9 POSTING OF TRANSACTIONS INTo
LEDGERACCOUNTS-CONTINUED
LEDGER
Purchases account
Dr. C.
Amount Amouns
(Rs.) Date Particulars (Rs.)
Date Particulars
2016 Jan 5 To Cash A/c 20,000
Jan 7 To B's A/c 25,000

Sales account
Dr. Cr
Amount Amoun

Date Particulars (Rs.) Date Particulars Rs.)


2016 2016 Jan 8 By Cash Alc 15,000

9 By C's Alc 18,00m

B's account
Dr. Cr
Amount Amour
Date Particulars (Rs.) Date Particulars (Rs)
2016 Jan 15 To Cash A/c 24,500 2016 Jan 7 By Purchases A/c 25,0
To Discount A/e 500
25.000 25.000

C's account
Dr.
C
Amount Amou
Date Particulars (Rs.) Date Particulars (Rs)
2016 Jan 9 To Sales A/c 18,000 2016 Jan 20 By Cash A/c 17,70

By Discount A/c 30

18.000 18.00

Discount account
Dr.
Cr
Amount Amou
Date Particulars (Rs.) Date Particulars (Rs)
2016 Jan 20 To C's Account 300 2016 Jan 15 By B's A/c 500

Salaries account
Dr. Cr
Amount Amou
Date Particulars (Rs.) Date (Rs
Particulars
2016 Jan 31 To Cash 2,000
2.4.3 SUBSIDIARY BOOKs
Most of the transactions in a business relate to receipts and payments of
cash, purchase of goods and sale of goods. Instead of routing these trans-
actions through the journal, these transactions are recorded in separate
records meant for each class of transactions. These books are called subsidi-
ary books or books of prime entry because transactions are first recorded in
these books before these are posted into the ledger. The following subsidiary
books are commonly used in a business:
1. Cash book to record receipts and payments of cash and also receipts
into and payments out of the bank.
2. Purchases book to record credit purchases of goods which a business
deals in or of materials and stores required for production.
3. Sales book to record the credit sales of goods in which the firm deals.
4. Purchases returns book to record the returns of purchased goods and
materials to suppliers.
5. Sales returns book to record return of goods by customers
6. Bills rece vable book to record bills of exchange or promissory notes
received from other parties.
7. Bills payable book to record bills of exchange or promissory notes
issued to other parties.
8. Journal proper to record those transactions that cannot be recorded in
any of the above mentioned subsidiary books.
Subsidiary books are maintained because they offer many advantages such as
division of work, specialization, saving of time, availability of separate informa-
tion for each class of transactions and easy detection and correction of errors.

5. Select the best answer: SELFASSESSMENT


QUESTIONS
) Rent account is a
a. nominal account
b. personal account
C. asset account
ii) Salary outstanding account is a.
a. personal account
b. nominal account
C. real account
(iii) Bank account is a
a. real account
b. personal account
c. nominal account
) Loss on account offire is a
a. real account
b. nominal account
C. personal account
6. In the case of balance sheet accounts
a. decreases in asset accounts are recorded by debits
b. increases in asset accounts are recorded by credits
c. decreases in liability accounts are recorded by debits
d. increases in liability accounts are recorded by debits
7. Which of the following statements relating to revenue and expen
e
accounts is not correct?
a. Revenue accounts have debit balances.
b. Expense accounts have debit balances.
C. Expenses are recorded as debits in ledger accounts.
d. Revenues are recorded as credits in ledger accounts.
8. Indicate the incorrect answer.
a. Drawings are withdrawals by the owners from the business.
b. Drawings decrease the net income.
c. Drawings decrease the owners' capital.
d. Drawings are not treated as business expense.
9. Arevenue account_
a. is increased by debits
b. is decreased by credits
C. has a normal balance of a debit
d. is increased by credits

2.5 SUMMARY
Understand the accounting process that leads to the preparation
financial statements. Transactions are analyzed in terms of their el
on assets, liabilities and owners'.capital. Following the rules of debir
credit, these are entered into the journal or the ledger.
D Analyze the effect of business transactions on the basic accouni
equation. The effect of transactions on assets, liabilities and owners
ital can be analyzed using the basic accounting equation.
Understand the use of an account in the process of building aceo nt
records, A "T shaped account is a convenient way of determining
ance of an item at any time. The left-hand side of the account is cale
debit side and the right-hand side is called the credit side. Increase
account are entered on one side and decreases on the other. The a
in the amounts on the two sides represents the balance in the accou
D Understand the rules
of debit and credit in recording busines tr
acco
actions in relevant accounts. Entering the in an
is based on certain rules
transactions crease
that differ with the account type.
assets are debits; decreases in assets Ihil
are credits. Increases inliabi
are credits; decreases in liabilities OW
capital are credits; decreases
are debits. Increases in
e
in the owners' capital are debits. Expe
i
and losses are debits; incomes
and gains are credits.
Understand how a journal is maintained and the concept of subsidiary
books. The journal has a specific format that consists of five columns.
Sometimes, a journal is subdivided according to the nature of transac-
tions. The parts of the journal are called subsidiary books.
aUnderstand the posting of entries in the ledger. The debit and credit
amounts in the journal and the subsidiary books are transferred to the
relevant side of accounts maintained in the ledger. To find the balance in
an account at the end of the accounting period or at any other time, the
two sides of the account are totaled and the difference between the two is
calculated. This difference represents the balance in the account.
aUnderstand how trial balance is extracted and its purpose. The clos-
ing balances of all ledger accounts are transferred to a statement called
the trial balance. It serves as a summary of the contents of the ledger
Agreement of the totals of debit and credit balances in the trial balance is
an indication of absence of arithmetical errors in the accounting process.

1. Account is a two-column format, resembling the English alphabet KEY WORDS


T, used to record accounting transactions.
2. Accounting transaction occurs when an economic event causes a
change in the assets, liabilities or owners' capital.
3. Double-entry accounting system requires debit and credit entries
of equal amount to record every transaction.
4. Journal is an accounting record in which transactions are entered
as they occur.
5. Ledger is an accounting record with separate accounts for each
account classification in which transactions are posted from the
journal.
6. Nominal accounts relate to expenses, revenues, losses and gains.
7. Personal accounts relate to persons. An account of customers,
suppliers, lenders and bankers fall in this category. The capital
account of the owners is also a personal account.
8. Real accounts relate to assets of the firm such as land, building,
investments, fixed deposits, cash balance, etc.
9. Subsidiary books are records used to enter special types of
transactions such as purchase and sale of goods, receipts and
payments of cash, etc. Such transactions may otherwise be recorded
in the journal.
10. Trial balance is the statement that shows the closing balances of
all ledger accounts separately for debit and credit balances.
2.6 DESCRIPTIVE QUESTIONS
I. Following is the list of various accounts. Find the assets, liabi
capital, revenue or expense accounts:
i) Machinery
i) Bank
(ii) Sales
iv) Unsold stock
(v) Bank overdraft
(vi) Ram (customer)
(vii) Purchases
(vii) Cash
(ix) Interest received
(x) Mohan (Proprietor)

2. Classify the following under personal, real and nominal accounts:


1) Stock
(i) Loan
(iii) Bank loan
(iv) Capital
(v) Drawings
(vi) Furniture
(vii) Cash
(vii) Bank
(ix) Ram (a purchaser)

3. Name the steps involved in the accounting cyele.


4. What are the two alternative ways in which accounts can
be classified?

2.7 ANSWER KEY


SELF-ASSESSMENT QUESTIONS

Topics Q. No. Answers


Steps in the d. total assets increased by Rs. 400,000 wiln
Accounting Cycle corresponding increase in liabilities by
Rs. 400,000
a. inerease in asset and increase in
liability equity
. a, repayment of bank loan of Rs. 90,000
Analysis of Accounting 4. i)-b, (i)b, (ii)->b, (iv)->¢
Transactions
Accounting Records 5. )-a, (i)->a, (ii)->b, (iv)-b
. C. decreases in liability accounts are
recorded by debits
Topies Q.No. Answers
7. a. Revenue accounts have debit balances
8. b. Drawings decrease the net income
9. d. is increased by credits

2.8 SUGGESTED BOOKS AND E-REFERENCES


SUGGESTED BOOKS
Anthony R.N., D.E. Hawkins and K.A. Merchant (2015). Accounting Text
and Cases, Tata McGraw Hill.
a Horngren C.T, Sundem G.L. and Elliot JA. (2014). Introduction to
Financial Accounting, Pearson Education.
Weygandt, J.J., Kimmel, PD., and Kieso, D.E. (2015). Accounting Principles,
Wiley India.

E-REFERENCESS
a Anthony, R.N., Hawkins, D.E. and Merchant, K.A. (2015). Accounting
Text and Cases, Tata MeGraw Hil.
Bhattacharyya, A.K. (2014). Financial Accounting for Business
Managers, Prentice Hall of India.

NMIMS Global Access School for Continuing Education


CHAPT
3 ER
FINANCIAL STATEMENTS

CONTENTS

3.1 Introduction
3.1.1 Income Statement
3.1.2 Balance Sheet
3.1.3 Statement of Cash Flow
3.2 Balance Sheet
3.3 Assets
3.3.1 Fixed Assets (Non-Current Assets)
3.3.2 Investments
3.3.3 Current Assets
3.3.4 Order of Presentation of Assets
3.4 Liabilities
3.4.1 Long-Term Liabilities
3.4.2 Short-Term Liabilities
3.4.3 Owners' Capital or Owners' Equity
Self-Assessment Questions
3.5 Basic Concepts Underlying Preparation of Balance Sheet
3.5.1 Business Entity Concept
3.5.2 Money Measurement Concept
3.5.3 Going Concern Concept
Self-Assessment Question
3.5.4 Cost Concept
3.5.5 Dual Aspect Concept
Self-Assessment Question
Activity
3.6 Statement of Profit and Loss
Self-Assessment Questions
3.1 Basic Concepts
3.7.1 Accounting Period Concept
3.7.2 Conservatism Concept
Self-Assessment Questions

NMIMS Global Access - School for Continuing Education


3.7.3 Realization Concept
3.7.4 Matching Concept
Self-Assessment Questions
3.7.5 Consistency
Self-Assessment Question
3.7.6 Accrual Concept
3.7.7 Materiality
Activity
3.8 Summary
Key Words
3.9 Descriptive Questions
3.10 Answer Key
Self-Assessment Questions
E-References
3.11 Suggested Books and
FINANCIAL STATEMENTS 37

INTRODUCTORY CASELET

ANALYSIS OF FINANCIAL ASPECTS

A wealthy person is looking to invest money in a company that will pro-


vide a reasonable return on his investment on a regular basis both by
way of dividend and capital appreciation. At the same time, he wants the
investment to be not too risky. How can he pick such a company? Fromn
where, can he get the necessary information to make the investment
decision? One important source of information that the investor is look-
ing for is the financial statements of a company.
Companies convey vital information about their performance, finan
cial position and cash flows through financial statements comprising of
statement of profit and loss, balance sheet and cash flow statement. Thee
statement of profit and loss reveals the profit earned or loss incurredd
by the company in an accounting period. The balance sheet shows the
financial position of the company at the end of an accounting period.
The cash flow statement presents the sources and uses of cash during
an accounting period.

&QUESTION
1. What are the other possible sources of information that the
investor is looking for? (Hint: Websites of investment advisors,
stock exchanges, etc.)
LEARNING OBJECTIVES

will be able to:


After reading this chapter, you
purpose of balance sheet.
Understand the nature and
contents ofa balance sheet.
Understand the format and preparation
accounting principles that underlie the on o
Explain the
a balance sheet. loo
nature and purpose of statement of profit and
Understand the loes
form and contents of statement of profit and
Understand the preparation
principles that underlie the
Explain the accounting
statement of profit and loss.

INTRODUCTION
3.1 information is provided in the form-
Information to users of accountingassets, liabilities, revenue and
expensee
financial statements that arrange
generally prepares three finan
different ways. Every business enterprise flows
sheet and statement of cash
statements: income statement, balance

3.1.1 INCOME STATEMENT


loss account) reports the resulm
The income statement (or the profit and
period. It matches the expen
business operations during the accounting
and reports the res=
for the accounting period with the revenues earned,
ing net income (profit or loss). The income
statement is of particular us=
investors, lenders and creditors. Investors use the past net
income as a b=
decisia
to predict the future net income and to make their investment statem-
Lenders to the business use information provided by the income
to form an opinion about the ability of the business to repay loans
and
opn
pay interest on time. Creditors use the income statement to form an
about the ability of the business to pay their dues on time.

3.1.2 BALANCE SHEET


The balance sheet reports the financial position of the business at a paru
lar point of time, generally at the end of the accounting period. It shows
amount of assets owned by the business and the claims on these assets.
claims on the assets belong to the providers of resources to buy the ass
i.e., the owners and creditors/lenders. As the business cannot spend mor
buying assets than the resources it has, the following relationship (also cau
the accounting equation) always holds.
Assets = Owners' Capital + Liabilities
Creditors and lenders analyze the balance sheet to understand the api
the business to repay their dues. The proportion of owners' capital in relat
to outside liabilities also serves as an indicator of the financial streng
the business
3.1.3 STATEMENT OF CASH FLOW
The statement of cash flow provides information about the cash receipts
and cash payments during an accounting period. Particularly, it shows the
sources from which cash is received, the uses to which cash is put and the
change in the cash balance during the accounting period.

The presentation of financial statements for external users depends on the


type of business organization. A business can be organized as sole-
proprietorship (having a single owner), a partnership (with two or more
owners or partners) or a company (having a large number of owners
or shareholders). Only companies are required by law to keep the pre-
scribed set of books and to present their financial statements in the pre-
scribed format. Other forms of business organization need to maintain
their accounts in a manner that enables determination of their income for
income tax purposes.
Normally companies are required to prepare annual and quarterly finan-
cial statements. The quarterly financial statements are also known as interim
financial statements. Certain business organizations are, however, not
required to prepare interim financial statements for external reporting

3.2 BALANCE SHEET


A balance sheet reveals the financial position of an entity. It sets out the QUICK TIP
assets, liabilities and owners' capital of an entity as on a certain date. Assets
are economic resources controlled by an entity which provide future cash A balance sheet shows the
flows to the entity. These economic resources are in the form of land and financial position of an entity
building; plant and machinery; furniture and fixtures; investments; inven- on a particular date.
tories; receivables; cash balances; ete. Liabilities represent the claims of
persons other than owners on these assets or the amount of money pro-
vided by them for acquisition of assets. Capital represents the claims of
owners on the assets or the amount of money invested by the owners to
acquire the assets.
It is prepared on a particular date and is true only on that date because
even a single transaction will affect the assets or liabilities and, there-
fore, the owners' capital shown in the balance sheet drawn on that date.
It is prepared only after preparing the profit and loss account as the net
income revealed by the profit and loss account is added to the owners' cap-
ital. The two sides of the balance sheet must have the same total because
capital is always equal to the difference between assets and liabilities, and
the amount of capital is independently arrived at by the capital account.
The non-agreement of the two sides indicates the presence of some error
in the preparation
QUICK TIP
Balance sheet is useful to both investors and lenders, Investors analyze the
balance sheet to form an opinion about the financial strength of the business. Balance Sheet provides useful
Lenders use the balance sheet to understand the capacity of the entity to information to both investors
repay the borrowed money, and lenders.
Balance sheet of a typical non-corporate entity in horizontal form
sented in Table 3.1.
TABLE 3.1 BALANCE SHEET OF X AS ON MARCH 31, 201
(HORIZONTAL FORM)
Amount Amour
Liabilities and Capital (Rs.) Assets (Rs.
Capital 600,000 Fixed assets
Long-term debt 300,000 Land 200,000
Building 300,000
Current liabilities
50,000 Equipment 100,000
Creditors
Accrued 40,000 Furniture 80,000
expenses
90,000 Patents 60,000
740,0
Investments 60,0

Current assets
Cash 50,000
Debtors 40,000
Inventories 80,000
Prepaid 20,000
expenses
190,00

990,000 990.0

In the vertical form of the balance sheet, capital and liabilities are liste
the top. The vertical form of the balance sheet given in Table 3.1 is preset
in Table 3.2.

TABLE 3.2 BALANCE SHEET OF XAS ON


MARCH 31, 2016 (VERTICAL FORM)
Sources of Funds
Capital and Liabilities Rs.
Capital 600,000
Long-term debt 300,000
Current liabilities
Creditors 50,000
Accrued expenses 40.000
90,000
990,000
Application of funds
Fixed assets
Land 200,000
Building 300,000
Equipment 100,000
TABLE 3.2 BALANCE SHEET OF
XAS ON MARCH 31, 2016
(VERTICAL FORM)-CONTINUED
Applications of Funds
Fixed Assets Rs.
Furniture 80,000
Patents 60.000
740,000
Investments 60,000
Current assets
Cash 50,000
Debtors 40,000
Inventories 80,000
Prepaid expenses 20,000
190,000
990,000
The various elements of the balance sheet are explained in the next section.
IMPORTANT CONCEPT
3.3 ASSETS Fixed assets are acquired for
long-term use and not for the
3.3.1 FIXED ASSETS (NON-CURRENT ASSETS) purpose of resale.
Fixed assets are meant for a long-term use and are not acquired for the
purpose of resale. Fixed assets are of two types: tangible and intangible.
Tangible fixed assets have a physical existence while intangible assets do QUICK TIP
not. Goodwill, patents, copyrights, trademarks, brands, etc. are examplesof
Non-current assets include
intangible assets. Land, Buildings, Plant and Machinery and Furniture are fixed assets and other
examples of tangible fixed assets. Fixed assets are shown at their net value
long-term assets such as
after accounting for accumulated depreciation. investments.
3.3.2 INVESTMENTS
QUICK TIP
Investments refer to money invested outside the business in the form of
shares, bonds or other instruments. Investments made for a period of more Tangible fixed assets have a
than one year are called long-term investments. Investments made for a physical existence. Intangible
period of less than one year are called current investments or marketable assets have no physical
securities, While long-term investments are referred to as non-current assets, existence.
short-term investments are included in current assets.

3.3.3 CURRENT ASSETS QUICK TIP

Current assets are either in the form of cash or are meant to be converted Long-term investments are
into cash or other current assets during the accounting period or the operat- those that are made for a
ing eycle of the business, whichever is longer. The operating cycle is the time period of more than 1 year
period between two points. The first point is the time of payment by an entity Investments made for a period
for purchase of raw materials. The second point is the time of realization of of less than a year are called
cash from customers for sale of finished goods that are converted from the current investments.
raw material. Cash, marketable securities, debtors (accounts receivable) and

NMIMS Global Access-School for Continuing Education


QUICKQUICK TIP inventories of raw material and finished goods are examples of current
Current assets reflect the ability of the business to pay its short-term liaht
bili
Current assets consist of cash
and other assets that are
3.3.4 ORDER OF PRESENTATION OF ASSETS
expected to be converted into
cash during the accounting Assets can be presented either in the order of permanence or in then
.
period or the entity's operating of liquidity. In the former case, the asset used over the longest time
cycle, whichever is longer. is listed first and the others are listed in that order. For example, Gor
is listed first followed by Land, Buildings, Plant and Machinery, Furmi
Patents, Vehicles, Investments, Stocks, Debtors and Cash balance. In
latter case, exactly the reverse order is followed as liquidity refers to the.
with which an asset can be converted into cash. It must, however, be
that there is no legal requirement relating to the order in which assets
liabilities may be presented in the case of non-corporate entities. Howe
in the case of a company, the order in which assets are to be shown is
seribed by the Companies Act, 2013. Companies have to prepare theiri
cial statements in accordance with Schedule III of the Companies Act, 2

3.4 LIABILITIES
The liabilities to outsiders can either be short term or long term.

3.4.1 LONG-TERM LIABILITIES


Long-term liabilities include borrowings from banks or financial institutir
for a period of more than one year. These may be secured or unsecured
the case of secured loans, some assets of the firm serve as collateral for
loan. Long-term liabilities also include bonds and debentures, which gen
ally have a maturity of more than one year.
QUICK TIP
3.4.2 SHORTTERM LIABILITIES
ng-term liabilities are for a
eriod of more than one year. Short-term or current liabilities are those that must be settled within
Short-term liabilities are meant year, for example, creditors (accounts payable), outstanding expenses, etd
to be settled within a period of
one year. 3.4.3 OWNERS' CAPITAL OR OWNERS' EQUITY
For a non-corporate entity, owners' capital consists of the capital orign
contributed by the owner and adjusted for subsequent profñtslosses a
drawings (withdrawals of money or goods by owners for their personal use
the case of a corporate entity, owners' capital or shareholders' equity cons
of share capital, retained earnings, securities premium and other reserves

SELF-ASSESSMENT
QUESTIONS
1. Sources of funds for an enterprise are reflected on the
a. income side of profit and loss account
b. expense side of profit and loss account
C. asset side of the balance sheet
d. liability side of the balance sheet
2 Which of the following is incorrect about a company's balance sheet?
a. It displays the sources and uses of cash.
b. It displays the sources and uses of funds.
C. It is an expansion of the basic accounting equation: Assets =
Liabilities + Owners' Equity.
d. It is also referred to as a statement of the financial position.
3. The balance sheet
a. Summarizes the changes in retained earnings for a specific
period of time.
b. Reports the changes in assets, liabilities and stockholders'
equity over a period of time.
c. Reports the assets, liabilities and stockholders' equity at a
specific date.
d. Presents the revenues and expenses for a specific period of time.

3.5 BASIC CONCEPTS UNDERLYING


PREPARATION OF BALANCE SHEET
In preparing the balance sheet, certain basic principles or concepts are fol
lowed. Every entity is expected to follow these concepts so that their finan-
cial statements provide reliable information, are consistent and comparable
with those of other entities. These are:
1. Business entity concept
2. Money measurement concept
3. Going concern concept
4. Cost concept

5. Dual aspect concept


3.5.1 BUSINESS ENTITY CONCEPT
5usiness entity concept requires that the business enterprise and the owners
be treated as two independent entities. The affairs of the business should
not
be mixed up with the personal affairs of the owners. The implication
of the
entity concept is that personal transactions of the owner are not recorded in the
books of the business. This concept helps determine the profit or loss
made by
the business. Personal assets of the owner are also not included in determining
the business assets. The business is liable to the owner for the capital
invest
ment made by the latter. Any amount withdrawn by the owner for personal use
is treated as a reduction of the capital and not as an
expense of the business.
3.5.2 MONEY MEASUREMENT CONCEPT
Only those transactions that can be measured in terms of money are
to be
recorded in the books of accounts. Many aspects of business such as quality of
management, level of customer satisfaction, etc., which cannot be expressed
in terms of money are not recorded. The
reason for using this concept is that
a common unit of measurement is needed for preparing
the financial state-
ments of business that report the operating results and financial position of
the business. Money serves as a common denominator in which t
different items such as a piece of land, a piece of equipment and r
can be expressed. A further assumption made by accountants is thas
of money does not change with the passage of time. This is a imitatig
this concept as it is well known that the purchasing power of mon
during periods of rising prices and rises during periods of falling
eices
3.5.3 GONG CONCERN CONCEPT
Financial statements are prepared on the assumption that
the entera
would continue to exist for an indefinite period of time. The entere
no intention of liquidating the business in the near terprise
future. Followine
going concern concept, thOse expenditures which are expected
to pr
future benefits are treated as assets rather than expenses. For examn
armple
expenditure on purchasing a machine is treated as a fixed asset.
The
the machine is spread over its useful life in the form of depreciation
is set-off against revenue in the income statement of
different accom
periods over the life of the machine. At the end of each accountingpe-
the value of the machine is shown at its acquisition cost minus accumu
depreciation, and it represents the cost applicable to its remaining us
life. In the absence of the going concern assumption, the
cost of the mac
shown in the books will have a different value (e.g. current market
p
Similarly, the amount of prepaid expenses and inventories are carried
ward at the end of an accounting period to be charged against the rere
of future periods. The going concern concept provides a sound basis for
measurement of income and motivates investors by ensuring the conti
of returns from investments.
An implicit assumption is made that the accounts have been prepared on a g
concern basis. If this is not the case, then the fact has to be explicitly dise
and the basis on which accounts have been prepared also needs to be discl=

SELFASSESSMENT
QUESTION 4. The going concern concept implies that
a. the business will continue to be profitable
b. the business will continue to exist in the foreseeable future
C. the owners are concerned about the success of the business
d. all of the above

3.5.4 COST CONCEPT


ac
The value of an asset shown in the balance sheet is the price paid its
sition and not its current market value. For example, a machine ac
1or Hs, 100,000 is shown in the books at a value of Rs. 100,000. This
easily determinable, objective and free from bias. Recording the asm
1Us current cost presents a problem because it may change every
also not be easily determinable because exactly the same asset
uet
it nly wi
available. Similarly, the realizable value of an asset can be knowi
it is sold. Following the cost concept, an asset will not be shown bothe b
n
of accounts if the entity has not paid anything for acquiring assel.
example, an entity can show goodwill as an asset in its balance
tneet
0

when it has purchased that goodwill for a price.


The problem with using this concept is that it loses its relevance when infla-
tion affects the price of an asset. For example, a piece of land purchased for
Rs. 1 million ten years earlier may cost Rs. 5 million now. If the cost of the land
is shown in the books at Rs. 1 million, the accounts will not reflect the true
position of the capital used in the business. Secondly, this concept results in
loss of comparability. Two assets acquired at different points of time at differ-
ent costs may give equal cash flows. The old asset would appear to be more
efficient as it is shown at a lower cost. However, a different conclusion may be
drawn if the current cost of that machine is taken into account. Thirdly, many
assets such as human assets do not have any acquisition cost. Such assets,
though important to an organization, do not get recognized under the cost
concept. Finally, when the cost principle is followed, the balance sheet does
not reveal the current worth of the business.
In some cases, an exception is made to the cost principle. Highly liquid assets
that are expected to be shortly converted into cash (e.g. short-term invest-
ments, accounts receivables, etc.) are shown at their net realizable value. The
net realizable value is the amount expected to be realized when the asset is
converted into cash. Similarly, a business may make investments in other
enterprises, which it intends to sell in the near future. Such investments are
shown at their current market value in the balance sheet.

3.5.5 DUAL ASPECT CONCEPT


Every transaction or event has two aspects. It affects two items of the
accounting equation simultaneously in one of the following ways:
1. It increases one asset and decreases another asset; or
2. It increases an asset and a liability simultaneously; or
3. It decreases one asset and increases another asset; or
4. It decreases an asset and simultaneously decreases a liability; or
5. It increases one liability and decreases another liability; or
6. It inereases a liability and increases an asset; or
7. It decreases a liability and increases another liability; or
8. It decreases a liability and decreases an asset.
For example, if a machine is purchased for cash, it results in an inerease
of one asset (machine) and decrease of another asset (cash). If the machine
15 purchased
on credit, it results in an increase of an asset (machine) and
increase of a liability (creditor).
The dual aspect concept leads to the basic accounting equation:
Equity (Capital) + Liabilities = Assets
which always holds true.
The basic accounting equation implies that the assets of a business are
always equal to the claims of owners and outsiders to these assets. The
owners claims are termed as capital and outsiders' claims are termed
as liabilities.
the business. Money serves as a common denominator in which the valua.
different items such as a piece of land, a piece of equipment and raw materia
can be expressed. A further assumption made by accountants is that the val
of money does not change with the passage of time. This is a limitation
this concept as it is well known that the purchasing power of money decin
during periods of rising prices and rises during periods of falling prices.

3.5.3 GOING CONCERN CONCEPT


Financial statements are prepared on the assumption that the enterpriho
would continue to exist for an indefinite period of time. The enterprise
Following th
no intention of liquidating the business in the near future.
going concern concept, those expenditures which are expected
to provide
future benefits are treated as assets rather than expenses. For example, c
th
a asset. The
expenditure on purchasing a machine is treated as fixed cost
whic-
useful life in the form of depreciation
the machine is spread over its accountine
is set-off against revenue in the income statement of different
period
periods over the life of the machine. At the end of each accounting
the value of the machine is shown at its acquisition cost minus accumulate
depreciation, and it represents the cost applicable to its remaining usei
life. In the absence of the going concern assumption, the cost of the
machin
shown in the books will have a different value (e.g. current market
price
Similarly, the amount of prepaid expenses and inventories are carried
for-

ward at the end of an accounting period to be charged against the revenu=

of future periods. The going concern concept provides a sound basis for
the

measurement of income and motivates investors by ensuring the continuit


of returns from investments.
An implicit assumption is made that the accounts have been prepared on a
going
disclosed
concern basis. If this is not the case, then the fact has to be explicitly
and the basis on which accounts have been prepared also needs to be disclosed

SELFASSESSMENT 4. The going concern concept implies that


QUESTION
a. the business will continue to be profitable
b. the business will continue to exist in the foreseeable future
cC. the owners are concerned about the success of the business
d. all of the above

3.5.4 COST CONCEPT


The value of an asset shown in the balance sheet is the price paid for its aequ
sition and not its current market value. For example, a machine acqure
for Rs. 100,000 is shown in the books at a value of Rs. 100,000. This value
easily determinable, objective and free from bias. Recording the asseis
its current cost presents a problem because it may change every day. It Ine
also not be easily determinable because exactly the same asset may not
available, Similarly, the realizable value of an asset can be known only w
it is sold. Following the cost concept, an asset will not be shown in the boo
of accounts if the entity has not paid anything for acquiring the asset.
example, an entity can show goodwill as an asset in its balance sheet onl ou
when it has purchased that goodwill for a price.
The problem with using this concept is that it loses its relevance when infla-
tion affects the price of an asset. Por example, a piece of land purchased for
Rs. 1 million ten years earlier may cost Rs. million now. If the cost of the land
5
chown in the books at Rs. 1
million, the accounts will not reflect the true
DOsition of the capital used in the business. Secondly, this concept results in
loss of comparability. Two assets acquired at different points of time at differ-
ent costs may give equal cash flows. The old asset would appear to be more
efficient as it is shown at a lower cost. However, a different conclusion may be
many
drawn if the current cost of that machine is taken into account. Thirdly,
assets such as human assets do not have any acquisition cost. Such assets,
though important to an organization, do not get recognized under the cost
concept. Finally, when the cost principle is followed, the balance sheet does
not reveal the current worth of the business.
In some cases, anexception is made to the cost principle. Highly liquid assets
that are expected to be shortly converted into cash (e.g. short-term invest
ments, accounts receivables, etc.) are shown at their net realizable valhue. The
net realizable value is the
amount expected to be realized when the asset is
converted into cash. Similarly, a business may make investments in other
enterprises, which it intends to sell in the near future. Such investments are
shown at their current market value in the balance sheet.

3.5.5 DUAL ASPECT CONCEPT


Every transaction or event has two aspects. It affects two items of the
accounting equation simultaneously in one of the following ways:
1. It increases one asset and decreases another asset; or
2. It increases an asset and a liability simultaneously; or
3. It decreases one asset and increases another asset; or
4. It decreases an asset and simultaneously decreases a liability; or
5. It increases one liability and decreases another liability; or
6. It increases a liability and increases an asset; or
7. It decreases a liability and increases another liability; or
8. It decreases a liability and decreases an asset.
For example, if a machine is purchased
for cash, it results in an increase
of one asset (machine) and
1S purchased
decrease of another asset (cash). If the machine
on credit, it results in an increase of an asset (machine) and
increase of a liability (creditor).
The dual aspect concept leads to the basic accounting equation:
Equity (Capital) + Liabilities = Assets
which always
holds true,
e basic accounting equation
iways equal to
implies that the assets of a business are
the claims of owners and outsiders to these assets. Thne
owers claims are termed as capital and
as liabilities. outsiders' claims are termed
SELF-ASSESSMENT
QUESTION
5. The dual aspect concept means that.
a. when a transaction is recorded in the accountinging system,
system. there
t
are at least two effects on the accounting equation
b. both parties to a transaction have to record the transaction
affected
C. both the income statement and the balance sheet are ted
by
the transaction
d. one account increases and the other
account decreases a
Sa
result of the transaction

of the concept that :


Match the accounting concept with the description is
ACTIvITY 1 given in the following table:

Concept Description of the Concept


A Business Entity A Every transaction or event has two aspects 0

B Money B Financial statements are prepared on the


Measurement assumption that the business entity will continue
to exist for an indefinite period of time.
C Cost C Books of account record only those transactions
that can be measured in terms of money
D Going Concern D The value of an asset shown in the balance
sheet is not its current market value but is the
price paid for its acquisition
E Dual-aspect E The business enterprise and its owners are
independent entities

3.6 STATEMENT OF PROFIT AND LOSS


to show
Statement of profit and loss or the income statement is prepared
period.
IMPORTANT CONCEPT the amount of profit earned or loss suffered by an entity during a i
! shows the various items of income and expenditure, grouped under
differeni
Gross proft is the difference difierent
heads, relating to an accounting period. It is generally prepared in
between sales revenue sections. For a trading firm, the first section measures the gross profit,
whiet
and the cost of goods sold is simply the difference between sales revenue and the cost of goods
sole
and operating profit is the operat
In the next section, the operating profit is determined by deducting normau
difference between the
ing expenses from the gross profit. Operating expenses relate to the
gross profit and operating geu
expenses, operating activities of the business such as administrative, selling and
non-operatng
eral expenses. Finally, the net profit is determined by adjusting
erpenses (such as interest expense, loss on sale of fixed assets) and non-0peun
ating income (such as interest income, profit on sale of fixed assets) from
operating profit.
expen
In a manufacturing concern, the cost of goods sold also includes all
incurred in the factory for producing the goods such as wages, powe
fuel and rent of factory premises. A typical multi-step statement of proit
loss is presented in Table 3.3.
TABLE 3.3 MULTI-STEP STATEMENT OF PROFIT AND
LOSS OFA FOR THEYEAR ENDED MARCH 31, 2016
Particulars Amount (Rs.)
Net sales 5,000,000
Less: Cost of goods sold 3,800,000
Gross profit 1,200,000
Less: Selling. general and administrative 400,000
expenses
Operating profit 800,000
Interest expense (50,000)
Interest income 30,000
Net profit 780,000

rom the trading account, the cost of goods sold, Rs. 3,800,000, can be worked
ut as follows:

Beginning inventory 350,000


Add: Purchases (net of returns) 3,150,000
Carriage on purchases 150,000
Wages 300,000
Cost of goods available for sale 3,950,000
Less: Ending inventory 150,000
Cost of goods sold 3,800,000

The selling, general and administrative expenses of Rs. 400,000 in Table 3.3
is the sum of the following expenses:

Rs.
Depreciation 150,000
Insurance 40,000
Printing expenses 25,000
Carriage on sales 27,000
Salaries 130,000
Bad debts 28,000
400,000

Thedivision of the statement of profit and loss under different sections


provides more information to the users leading. to better decisio making.
Omparison of current gross profit rate with past rates and that of other
irms in the industry reveals the effectiveness of a firm's purchasing and pric-
ng policies. Similarly, non-operating income may form a significant portion
OUne total income, External users of financial statements focus more on the
operating income as they consider this income to be sustainable in the future
and non-operating income to be non-recurring
SELFASSESSMENT 6. The income statement shows
QUESTIONS a. Cash balance at the end of the period
b. Contributions by the owner during the period
c. Revenues earned during the period
d. Profit earned or loss incurred during the period
7. Which of the following statements is incorrect?
by an entity for a period of time
a. Net income is reported
b. Net income increases the owner's
capital
expenses
cC. Net income is equal to revenue minus
d. Net income is equal
to revenue minus the sum of expens
and drawings

CONCEPTS
3.7 BASIC
of the balance sheet, certain basic principles or cOmeawl
cepisa
As in the case and loss also to
followed in the preparation of statement of profit t
Secu
consistency and comparability with other entities. The basic
reliability,
of statement of profit and loss imel
cepts underlying the preparation
(i1) realization, (iv) matching (tm
(i) accounting period, (ii) conservatism,
sistency, (vi) accrual and (vii) materiality.

3.7.1 ACCOUNTING PERIOD CONCEPT


and its exact profit or loss g
A business is expected to have a long-life,
up. Measuring the pertie
be determined oly when the business is wound
because no corrective ste
mance over a long period of time loses value expectations. To tad
can be taken to improve it if it is below the owner's
position firom time
the business performance and to measure its financial
of time, usualy a yez
time, its life is divided into relatively small intervals
a financial yeur.
The accounting period is called an accounting year or
is also influenced
The adoption of accounting year for reporting purposes
companies, a compa
certain legal requirements. Under the law relating to
The law relang
is required to submit annual reports to its shareholders.
taxable income
taxation of income requires determination of annual
a calendar je
accounting period usually adopted by a business is eitherListed compane
(January-December) or a financial year (April-March).
are also required to prepare quarterly income statements.
identihcatou
The adoption of the accounting period eoncept requires tions
For some transacu
transaction that relate to a specifie accounting period.
the identification is easy and straightforward. For many
transactions, su a
ch
alocr
yeal, au
acquisition of a fixed asset that affect more than one accounting partie
to to determine the consumption of the asset in & P
tions have be made
ular accounting period.

3.7.2 CONSERVATISM CONCEPT


houldh
In preparing financial statements of a business, an accountant sti 7sactn
treatment to a tra
conservative. He/she should apply that accounting
that results in the lowest (most conservative) estimate of the income. He/she
should not anticipate incomes and should provide for all possible losses. An
income should be recognized only when it has been realized. Further, when
there are many alternative values of an asset, an accountant should choose
the method that leads to a lesser value. Following the conservatism concept.
the rule 'cost or market value, whichever is lower' is followed in valuing
inventories. Following this rule, the value of inventories is written down to
its market value if it declines below its cost. However, the value of invento-
ries is not revised upward if its market value goes above its cost. There are
many other instances where conservatism is applied, for example, making
provision for doubtful debts; marking investments to market to reflect their
current market value, etc.

What is the underlying concept that supports


SELF-ASSESSMENT
8. the immediate QUESTIONS
recognition of an estimated loss?
a. Substance over form
b. Consistency
C. Matching
d. Conservatism
9. A businessman purchased goods for Rs. 2,500,000 and sold 80% of
such goods during the accounting year ended March 31, 2016. The
market value of the remaining goods was Rs. 400,000. He valued
the closing stock at Rs. 500,000. He violated
a. Money measurement concept
b. Accounting standard for Revenue Recognition
C. Accounting standard for valuation of inventory
d. Periodicity concept

3.7.3 REALIZATION CONCEPT


According to the realization concept, revenue should be recognized when
t is realized. Revenue from sales or service transactions is considered to
be realized only when certain requirements relating to performance (such
as transfer of property or transfer of risks and rewards of ownership) are
satistied, and at the time of performance there is no significant uncertainty
regarding the ultimate collection of revenue.
evenue may be recognized even when the payment for a transaction is yet
o received. As a result, even credit sales are recognized by a business as
De
revenue. or example, a business receives an order from a customer for the
Supply of a custom-made machine in the year 2013. The business supplies the
achine to the customer in 2014, and the customer makes the payment in
401. In this case, the condition for recognition of revenue is satisfied in the
year 2014, and hence revenue should be recognized in
2014.

he realization concept also has another interpretation. According to this


nterpretation, any change in the value of goods is to be recorded only when
the business realizes it, that is,
when the goods are sold.
3.7.4 MATCHING CONCEPT
accounting period, revenues earned
To determine the income of an expenses incurred to h
the accounting period are matched with the
revenues. The first step in the matching of revenues and expenses ist
an accounting period. After determint
mine the revenue earned during all expenses incurred to earn that8 th
revenue for an accounting period, determine the income of that
acoeny
the revenue to
are deducted from
recognition of revenue is deferred on the basis that ititin
period. If the to such revenue must also be
defer
earned, all expenses pertaining periodicity concoh
because of accrual and
question of matching arises
1.4.2014 to 31.320
chairs Rs. 1,000 each during the period cha
EXAMPLE1 X buys 500 period, he has paid Rs. 450,000 to the supplier of
During the accounting to Dav h
400 of these chairs @ Rs.1,200 each. His customers are yetper him
He sells 1,000
2015. He hires an employee @Rs. onth
Rs. 20,000 as on March 31, of March 2015 is yet to be paid
the month
The salary of the employee for
X will recognize the revenue for the
Following the matching concept, bal
sold and treat the cost of 400 chairs as an expense. The revenm
400 chairs
as stock in hand. Accordingly, the
ance 100 chairs will be shown Rs. 400,000
period will be Rs. 480,000, from which expenses of
for the Rs. 12,000 on account of salary
(for
account of purchase cost of chairs and
concept) will be deducted. X will showa
12 months following the accrual
profit of Rs. 68,000 for the year.
the customers and the actual amount
The actual amount received from What is
for measuring profit or loss.
paid to the supplier are not relevant expenses incurred during the account
relevant is the revenue earned and
amount received or paid.
ing period, irrespective of the
to revenue that are directly associated
It is easy to match those expenses
example is the cost of goods sold.
For
with the earning of revenue. An
some expenses, subjective judgement is
required to apply the matching
concept. An example is the cost of fixed
assets. which provide benefits
number of accounting periods. There are other expenses such as
over a
with particular goods
administrative expenses that cannot be associated as
or services sold. Such expenses, known as
period costs', are treated
incurred. Expenses
expenses of the accounting period in which these are
costs.
related to products or services are known as product

4/5th of the goods for


SELF-ASSESSMENT A purchased goods for Rs. 1,500,000 and sold
& QUESTIONS
10.
Rs. 1,800,000 and met expenses amounting to Rs.
250,000 during tne
Which of the
year 2015. He counted the net profit as Rs. 350,000.
accounting concepts was followed by him?
a. Entity b. Periodicity
C. Matching d. Conservatismn
I1. The determination of expenses for an accounting period is baseu
the principle of.
a. Objectivity b. Materiality
C. Matching d. Periodicity
3.7.5 CONSISTENCY
Consistency implies that the same accounting policies and
followed by an enterprise in preparing its accounts procedures are
from one accounting
period to another. Accounting standards provide for equally
accounting alternatives in respect ot certain matters. For acceptable
example, an enter-
nrise can value its inventories using either the First-in, First-out (FIFO)
method or the Last-in, First-out (LFO) method.
Under the FIFO method,
the cost of older inventories is used to determine the profit or loss, while the
ending inventories represent the most recently purchased items. Under thee
LIFO method, the cost of most recently purchased items is used to
determine
the profit or loss and the ending inventories represent items purchased
ear
ier in time. Change in the method from one accounting period to another
can have a significant impact on the amount of expense recognized and the
value of inventory in hand at the end of the accounting period.
Similarly, several alternative methods, such as the straight-line
method,
written-down-value method, sum-of-the-years-digits method, etc. are avail
able to provide for depreciation on fixed assets. Again, change in the method
of providing depreciation can have a significant impact on the amount of
depreciation expense recognized and the value of fixed assets in hand at the
end of the accounting period.
The consistency principle helps to achieve comparability of financial state-
ments of an enterprise through time. It is not that the accounting policies,
once adopted, cannot be changed. Accounting standards allow change in
accounting policies under certain circumstances. Whenever a change is
made, the accounting standards require that the enterprise should make a
full disclosure of the change and also of the rupee effect of the change on the
reported income and financial position of the enterprise.

12. An enterprises follows the written-down-value


SELFASSESSMENT
method of
QUESTION
depreciating machinery year after year due to
a. Reliability b. Convenience
c. Consistency d. All of the above

3.7.6 ACCRUAL CONCEPT


Under the acerual concept, revenues are recognized when they are earned
and expenses are recognized when the related goods or services are used.
ne tirning of receipt of revenues and payment of expenses is immaterial. The
acerual concept facilitates measurement of income for a particular accounting
period Applying appropriate tests, the revenue pertaining to an accounting
pernod is recognized first. Then the expenses incurred to earn that revenue
re recognized. Income for the accounting period is then determined as the
alerence between the revenue recognized and the matched expenses.

Or example, X buys goods worth Rs. 500,000, paying a cash of Rs. 350,000
dsells the
revenue
goods for Rs. 650,000, of which customers pay only Rs. 400,000.
S would be Rs. 650,000, his expense is Rs. 5,00,000 and his profit
2sed on the accrual concept will be Rs. 150,000. Cash receipt of Rs. 500,000
and cash payment
of Rs. 350,000 do not enter the calculation of profit.
The alternative to accrual accounting is the cash basis
accounting p.
may not be realized in cash. Cash may be received simultaneously Revenue
the revenue is created or after the revenue is created. The same
expenses. Pure cash basis accounting is not appropriate is the case
e for measuring
me wit
profitability of economic activities carried out during the accounting the
period
3.7.7 MATERIALITY
The term 'materiality' refers to the relative importance of an
item event
An item or event is considered material if its knowledge is
likely to affee
the decisions of the users of financial statements. Accountants
should en
that all material items are properly reported in the financial statero
determining the materiality of an item, they need to compare
vale s
information with the cost of providing such information. The the lue
value a
exceed the cost. For immaterial items, accountants can use mus
estimates inste
ofkeeping detailed records and can also disregard certain accountingn steam

ciples. Professional judgment is required to assess the materiality an prin


of iter-
For example, the cost of small value items such as stationery, lighting
may not be treated as an asset and may be written off as expenses. Ignorinmateri-

the matching principle, utility bills may be charged as expenses when


bhi
are received rather than when services are rendered.

ACTIVITY 2 Match the accounting concept with the description of the concept that
is
given in the following table:

Concept Description of the Concept


A Accounting Period A Same accounting policies and procedures
are
followed in preparing accounts year after year
B Conservatism B Revenue earned during an accounting period
is matched with expenses incurred to earn
that revenue
C Realization C Life of the business is divided into
smal
intervals of time to measure performance and
financial position
D Matching D An accountant should not
anticipate incomes
and should provide for all possible losses.
E Consistency E Accountants ensure that all material items are
properly reported in the financial statements
Accrual Revenue is recognized when it is earned and
not whenm it is actually received
G Materiality G Revenue is recognized
when it is realized.

3.8 SUMMARY
O Understand the nature and purpose
of balance sheet. The balance shee
reveals the financial position of an
entity. It is prepared on a partieua
date, and is true only on that date. It is
of the profit and loss account. The two
prepared only after the preparano
sides of the balance sheet must havs
the same total.

NMIMS Global Access -School for Continuing Education


Understand the format and contents of a balance sheet. It sets out the
assets, liabilities and owners' capital of an entity as on a certain date. The
balance sheet can be prepared in a horizontal or vertical form.

1. Accounting period is a small interval of time, usually a year out of KEY WORDS
the life of business, determined to track the business performance
and to measure its financial position.
2. Accrual basis of accounting implies that revenues are recognized
when these are earned and expenses are recognized when these are
incurred. The timing of receipt of revenues and payment of expenses
is immaterial.
3. Consistency means that the same accounting policies and proce-
dures are followed by an enterprise in preparing its accounts from
one accounting period to another.
4. Conservatism is the non-anticipation of incomes and making
provision for all possible losses.
5. Cost concept is the concept on which the value of an asset is
determined on the basis of its acquisition cost, which is the most
objective basis.
6. Cost of goods sold is the cost of that part of goods available for
sale (beginning inventory + purchases), which is sold during the
accounting period. It is caleulated as the cost of goods available for
sale minus the cost of ending inventories.
7. Current assets are assets, which are either in the form of cash or are
meant to be converted into cash or other current assets during thee
accounting period or its operating cycle, whichever is longer.
8. Current liabilities are liabilities that must be settled within one yea.
9. Dual aspect concept states that every transaction or event has two
aspects. The impact of a transaction is such that the accounting
equation: Assets = Liabilities +Owners' Capital always holds.
10. Entity concept is a concept in which the affairs of business are
distinguished from the personal affairs of the owners.

3.9 DESCRIPTIVE QUESTIONS


In what order are assets listed on a balance sheet of a sole proprietor?
2 At the instance of the management, you want to show the good quality
of management in financial statements. As an accountant, which
accounting concept will you be violating?
3. A company wants to:
a. Treat goods drawn from the business by the owner as his/her
personal expense.
Bgnore the increase in the price of some inventory items.
54 FINANCIAL ACCOUNTING AND ANALYSIS

State the accounting concept the company would need to follow in the
above cases.
4. Why is a business treated as a separate entity for accounting purposes?
5. Name the two main forms in which a balance sheet can be prepared.

6. Why should the two sides of a balance sheet always match?


7. How are fixed assets different from current assets?
8. What are intangible fixed assets? Give some examples.

9 How is the going concern assumption applied in the presentation of


financial statements?
10. State the problems that arise in the application of the cost concent t
the preparation of balance sheet.

3.10 ANSWER KEY


SELFASSESSMENT QUESTIONS
Topies Q. No. Answers
Liabilities 1. d. liability side of the balance sheet
2. a. It displays the sources and uses
of cash.
3. c. Reports the assets, liabilities and
stockholders' equity at a specific
date.
Basic Concepts Underlying 4. b. the business will continue to exist
Preparation of Balance Sheet in the foreseeable future
5. a. when a transaction is recorded in
the accounting system, there are
at least two effects on the account
ing equation
Statement of Profit and Loss 6. d. Profit earned or loss incurred
during the period
7. d. Net income is equal to revenue
minus the sum of expenses and
drawings
Basic Concepts 8. d. Conservatism
9. c. Accounting standard for
valuation of inventory
10. c. Matching
11. c. Matching
12. c. Consistency
3.11SUGGESTED BOOKS AND E-REFERENCES
SUGGESTED BOOKS
a Narasimhan M. S. (2016). Financial Statement and Analysis. Cengage
Limited;
Learning India Private First edition.
a Financial Accounting Bssentials You Always Wanted To Know: 4 (Self
Learning Management). Vibrant Publishers, 2017.

E-REFERENCES
a Food and Agriculture Organisation, Statistical Database, Various years,
http:/faostat. fao.org accessed on 30 April, 2011.
a Accountingtools.com Financial Statement Analysis.
-
CONTENTS

4.1 Introduction
4.2 Trial Balance
Activity
4.3 Relationship between Profit and Loss Account and Balance Sheet
Self-Assessment Questions
4.4 Preparation of Profit and Loss Account
4.4.1 Gross Profit
4.4.2 Sales Revenue
4.4.3 Sales Returns and Allowances
4.4.4 Goods and Services Tax
4.4.5 Cost of Goods Sold
4.4.6 Operating Profit
4.4.7 Net Profit
4.4.8 Income Tax
Self-Assessment Questions
Activity
4.5 Preparation of Balance Sheet
4.6 Adjustment Entries
4.6.1 Prepaid Expenses
4.6.2 Depreciation and Amortization
4.6.3 Income Received in Advance or Unearned Income
4.6.4 Outstanding (Accrued) Expenses
4.6.5 Outstanding or Accrued Income
4.6.6 Provision for Bad and Doubtful Debts
Self-Assessment Question
4.7 Adjusted Trial Balance
4.7.1 Closing Entries
4.7.2 Post-Closing Trial Balance
INTRODUCTORY CASELET

MODERN COFFEE HOUSE

Achok and Ramesh who had set up Modern Coffee House on January2,
2018 (refer Chapter 1) carried on the business till December 31, 2018. As
on December 31, 2018, their accounting records revealed the following
balances:

Rs. Rs.
Materials purchased 1,500,000 Miscellaneous 850,000
expenses
Sale proceeds & collections 4,100,000 Salaries 510,000
Expenses on eatables 580,000 Rent 360,000
Capital 400,000 Cash and bank 900,000
balance
Suppliers 200,000

They noted that as of December 31, 2018, they had yet to pay Rs. 35,000
to their workers. On the other hand, they had paid rent for a year on
July 1, 2018. Materials costing Rs. 100,000 were still at hand on
December 31, 2018. They did not know how to determine the profit and
loss of the business for the year just ended taking into account these
items and their financial position as on December 31, 2018.

QUESTIONS
1. Calculate the amount of material consumed by Modern Coffee
House during the year. (Hint: Deduct stock of materials at the
end of the year from the amount of materials purchased.)
2. Caleulate the amount of rent expense that pertains to the
accounting year January to December 2019. (Hint: Calculate
rent for the 6-month period of July to December, 2018.)
LEARNING OBJECTIVES

After reading this chapter, you will be able to:


Understand the relationship between profit
and loss accon
balance sheet. ntand
Prepare profit and loss account and balance
sheet from the
trial balance without accounting for any adjustment e

entrioven
Understand how to make adjustments for
accruals, deferrals
other items. and
Prepare profit and loss account and balance sheet
after accotn
unting
for adjustment entries.
Prepare closing entries and post-closing trial balance.

4.1 INTRODUCTION
Ashok and Ramesh can determine the profit or loss made by
Modern Cof
House during the year by preparing a Statement of Profit and
Loss and thei
financial position at the end of the year by preparing a
Balance Sheet.The
balance sheet shows the amount of assets and liabilities of
the businesS t
the close of an accounting period. These two statements
are closely relate
to each other. However, they need to first prepare
the trial balance as on
December 31, 2018, which will form the basis of preparation
financial statements. of the other

4.2 TRIAL BALANCE


After the transactions are posted in the ledger,
a statement showing the
accounts with debit and credit balances
separately is prepared. This state
ment is called the trial balance. It serves as a
the ledger. It has two columns. The debit
summary of the contents of
balances are listed in the left-hand
column and the credit balances are listed in
the right-hand column.
The trial balance is prepared on a
particular date, which is mentioned atthe
top of the trial balance. The general
Table 4.1.
format of the trial balance is shownin

TABLE 4.1 TRIAL BALANCE


AS ON
Debit Balance Credit Balance
S. No. Ledger Account L.E (Rs.) (Rs.)

The totals of the debit and credit


metical errors in the accounting
balances must agree if there are no aru
th-

system all debits and credits takenprocess because under the double-enuy
together must be equal. Instead ofusis
balances of ledger accounts, the trial
totals of the debit and credit sides of all balance may be prepared using
ledger accounts.
The trial balance agreement implies that the accounting work is free from
lerical errors, even though other errors may still be present. Some entries
to the wrong ledger account, but on the
may havee been omitted or posted
sorrect
corre side. Mistakes in posting on the debit side may have been offset by
mistakes in posting on the credit side.
TEthe debit and credit totals of the trial balance do not agree, one or more of
the following errors might have been committed:
1. A debit amount is posted as a credit amount or vice-versa.

2. Arithmetic mistakes in determining account balances.


3. Error in carrying the amount from the ledger account to the trial
balance or listing the account balance in the wrong column of the trial
balance.
4. Errors in calculating totals of the trial balance.
The work of preparing financial statements starts after establishing the
agreement of the trial balance. This is because it is desirable to ensure that
the total of accounts with debit balances is equal to the total of accounts with
credit balances. Preparation of financial statements becomes difficult in the
absence of an agreed trial balance.

RECTIFICATION OF ERRORS
Agreement of the total of debit balances and credit balances in the Trial
Balance does not mean absence of errors in the books of account. Agreement
of the Trial Balance simply means that for every debit, there is an equivalent
credit entry. For example, the Trial Balance may agree even though a trans-
action is not entered at all in the books of account.

TYPES OF ERRORS
There could be four types of errors in the books of account.
1. Errors of Omission
2. Errors of Commission
3. Errors of Principle
4. Compensating Errors
Errors of Omission occur when a transaction is omitted to be entered in the
DOoks of account. For example, a credit purchase might not be recorded at
all in the books. Even then, the Trial Balance will agree. Such an error is
etected when statements of account are received from creditors or sent to
debtors.

Tors of Commission occur when the balancing or totaling of an account is


Correct or an amount is wrongly posted or the balance of an account that is
aried forward to the next period is not correct, etc. For example, an amount
oT KS, 2,000
received from a debtor may be posted to his account as a eredit
31 S)Isodap pue Jeuojsno oyj uuoij sraAoDa.I 1®|[[Os oyi jeyi xej 128.1Iput
ua
1 "sajes Ssoiš UOjonpap e se uMoys si (LSD) XEL saoth.los
pue spoe
uo.J
XVI SAOIAYIS ANV SaOoD FE

qunoooes
pue 1110.Id ayi uI onuaAa.I sajes ayi urouj uojonpaPE SE UMOus are pue or
Sa[es oj 1unodDe anuaAal JaJunoae aIe saoUEMOIIE pue
OAOI suInjai san

10ajap rO 3ðeuep 'uojeaJIoads ut ašueyo JoJ ua


31esuduos oj aouEMOJ[E ue uaAI3 ae pue spoos asayi uje13.I SlauIojs
Spooš yons šujuinja Jo pea1su! 'saugauros '3AgoaJop 1o pašeuep Jo 'suc
Aq pau.An
-BOYIDOds ayi yiiM 3Uep.00De uI 1ou a1OM Koyi se stouojsn3
spooB Jo anjea oyi suasaIdaI qeyi anuaAaI SaJPs Jo 1ed 4eyi are su.inja.i saj
SaONVMOTTV GNV SNANLaN SITVS ST

sao[OAUt sajes jO SISEq ayi Uo paaJt


a1e suo1Oesueq APP-o1-Aep yoIym u 9unoooe sajes ay1 uoij uONPI
sejes
poLrod suunosoe ue 1oJ anuanaI sajes jejo oyL *anuaAaI saJes Jo SujurE
anuana
saziušoaaI uIy e youm ge auij ayi urOAO3 sejdiouiid uorgtušo3aI
aNNAAIASITVST7
"ses[uard Aao1oej JO ueI pue jony pu
JOMOd 'sašBM Se yons spoo3 Suonpo.Id Joj AOJOeJ ayi uI painout sasuadx
osje P[os spoo Jo so ayi 'ureouoo Sujinjoejnueu e uJ
A^n
PIOS spoo5 jo ]s0 a42 ITe sapnou!
pue anuanaI sajes ayj uaaMjaq suojsno pue To.o *aðe91e 'saseyoind uo pIed y3tay a.Ie sasuadxa yon
aDuajayip 3yi SI ayoid ssoi Jo sejduexg 'uoipuoo pue uoeoj 9UasaId atayi oj spoo3 oyi šuišuIq t
sasuadxo eyno 1e pue (SY001s u eseo.loap O SeaIOU IOJ pOJSTlP
dllXoIno paLnou
soseyoind Jo sissuoo uy Supea e Jo oseo oyn u Pos spooš Jo 1s00 uI PIC
spoos JO gso ayi pue onuaAaI SOJes ayi ueaMIaq aoualaJJIP aYj sI 1LJOId ss015
LIJONd Sso¥ IF
LNNOOOV SSOT INV
LIJONd JO NOILvAVdaHd V

puepIAa P
soLIejeS
asuodxa ue se juouaejs
auoou! ayj ui pojIodo. 1ou SI Sue! uIMOJjOJ oyi Jo auO yoIYM
sßuuieo paujejar Jo Juouojeis P uauojejs MoI YSe
uouajejs awoouj E
&1sy poedord
SNOLLSANo
Aeaouas s squourojejs jejoueuy äumon9J OYn Jo auo yoiM I
JNAWSSASSVIT4s

29 SLNANaLVIS TVIONVNIA O NOLIVAVATNd


-o1a sosuOdx ALIndas 'sisoÐ 9 ueuojuteu opnjpui sasuadxa [e-IOUOD
p peq 'sasUOdx
PDUEInsuI 'saIPIYOa KiaaIjopJo aoueuaueuI pue uIUtna
O sasuad>O
2 sosuadx
S
suHOEd
p o o'saSuadxe
*SaTEs Uo
aSUIIIeD pue ytoAJ 'sasuadxa sUIsn
quuÐuasIgioape uoIssuIuuoD Pue
uousa[US PTIPUsEISOdSOsu®d>XO UOIJnqiIJSIP pue
sO ET
Oydotas pue sasuadXa ee[
BuIIIas '22a 'eaj 3Upne 'sosIEUP
sasuadxaSuiyii SuIPINA BB
u3.I saaKOTdua UIO 01 PIed saLIETes opnjput sosuedxa 3A!}E.IJSIt P
*saSuadxe [eiua pue sasuadxa
ue ssauisnq uI
ZuiIIas 'ÐAIGEIJS!uIuPE PPT jyoId ssoJ6 se
JO SuOINEIdo Teuiiou o] paejoi oJe sasuedxa uIeIOd0 6ugeiado snuju
sasuadxa sUIIEIOdo snuIu jUOJd ssoi se poqe[nnores SI 1LyoJd uIIeISdo UL PaJejn>jes si 1yOid 6uneiado
LIA0¥d DNILVAddO 9VV dIL HOind

-apIAo1d 1eyi 3UnoDDE ou sI 313uL


KxojuoAu!
KIOJueAuI
anea au
BuIpus ay Jo
po1uod sununo>o ue JO Pua e spooz prosunIuPua:Arojusau
aInii9SUO
äupua
oyi

ss3ua Jo 1soo aua o1 pappe st s3osse paxy


oe uat u UOI[oru
o2sosnouoIeM PIEd sosem xOnDmoH PIOS spooB Jo 3s02 Jo 3.1ed
wioj osIe Pue saxoJs ui sieqIom oj pred saseM :sošeM
PIOs Spoo Jo 1soo Jo uoI4enoTe»
10J spoo oui JO S»LId sseuoInd aua oi pappe s Pue 1sos aseyand Jo
Jed e I Su21 AIOuaAu JO 3seyoind uo jqs1eiA :sasEqond uo qyšaiI
3unoose ssoI PUE yOId aui ui nšy saseyoInd ayi uo.y uononpape se
umoys SI PUE Uno>SE SUInjai saseyoind eui ui paaenunooe st sxÐ1es
o1 pauInjaI SI 9eYI SUIaU PSEYSInd JO 1sos auL :suingol saSEyOInd £
aoiId DsEyPIna aui UOLI P®JSnpap osIe 3JE saouEmoIIe yOns AÐSEYOInd
au Jo SpipurS ATEnb aYi 29au 1Ou Op eYi spooš 1OJ 3SIId 9SIOAUI
aui ui uOIIOnpa.I JO AEA Aq saouvaoiNp aAIS SIE SISIIddns *SauiIauoSS
gunouue payIDÐds e Jo sseoxe uI ÐSIPueyoJauu O KI1uenb e šuISEYOInd
1oJ 1aI[ddns aya sq pamoIue 9unoosip apvij
Aue Bu1onpap 13Je aneA 2au
e umoys aJe saseyoina 'saoIOAUI SEYOand J sIseG Yi UO
PaT®UO A.
suonoesue aseyoInd Aep-o1Aep qÐIYm ur junooae saseyond ay9
uoJ
uoye sI pofisd ununosov aui xoJ oanzy soseuond [eao ouL soseyoana *z
oauereq eiu eui uoij uoE? s{ anšy
stqL pouad suaunosse snotaoidšuIUUtsSq
aui Jo KiouSAUI šusoo
AIoIuSAu!
a SI PoLOd
šuuutšoA I
uIurnosse UE JO KIOJUSaut auL
PIOS Spoos JO 1sos Jo uoienoES Sq9 JO 9Ed uIOJ Suaii 3ULmOIIOJ aYL
dTOS SaOOD A0 LSOD STV

seIes 1eN

LSD
suinaeI sa[es :ssaT
sa]es ssoJD

Se pajenoIea SI aInšy saIes eN


SmoIIOJ

SISXTVNV aNV DNIINnODov TVIONVNII T9


Net sales figure is calculated as follows:

Gross sales
Less: sales returns
GST

Net sales

4.4.5 COST OF GOODS SOLD


The following items form part of the calculation of cost of goods sold.
1. Beginning inventory: The beginning inventory of an accounting
period the closing inventory of the previous accounting period. This
is
figure is taken from the trial balance.
2. Purchases: The total purchases figure for the accounting period is taken
from the purchases account in which day-to-day purchase transactions
are entered on the basis of purchase invoices. Purchases are shownat
net value after deducting any trade discount allowed by the supplier for
purchasing a quantity of merchandise in excess of a specified amount.
Sometimes, suppliers also give allowances by way of reduction in the
invoice price for goods that do not meet the quality standards of the
purchaser. Such allowances are also deducted from the purchase price.
3. Purchases returns: The cost of purchased items that is returned to
sellers is accumulated in the purchases returns account and is shown
as a deduction from the purchases figure in the profit and loss account.
4. Freight on purchases: Freight on purchase of inventory items is a part
of purchase cost and is added to the purchase price of the goods for
calculation of cost of goods sold.
5. Wages: Wages paid to workers in stores and warehouses also form a
part of cost of goods sold. However, wages paid in relation to an item o
fixed assets is added to the cost of that asset.
6. Ending inventory: Unsold goods at the end of an accounting pero
constitute the ending inventory. There is no account that proviae
the value of the ending inventory.

QUICK TIP 4.4.6 OPERATING PROFIT


nses.
Operating profit calculated
is The operating profit is caleulated as gross profit minus operating expens
au
Operating expenses are related to normal operations of the business
as gross profit minus operating
expenses. include administrative, selling and general expenses.
Administrative expenses include salaries paid to office employees, rent
office building, lighting expenses, legal expenses, postage and teleptn
charges, audit fee, etc. Selling and distribution expenses include salesm
salaries and commission, advertisement expenses, packing expenses, wan on
housing expenses, freight and carriage on sales, export duties, expense
running and maintenance of delivery vehicles, insurance expenses, bad a
etc. General expenses include maintenance costs, security expenses, etc:

NMIMS Global Access- School for Continuing Education


4.4.7 NET PROFIT
Net profit is calculated by adjusting the
operating profit for non-operating
revenues, non-operating expenses, gains and losses.
Non-operating 1tems
are not related to the main business operations and include such
items as
interest income, dividend income, interest expense, profit or loss on disposal
of fixed assets, etc.

4.4.8 INCOME TAX

For companies, income tax is treated as a separate business expense. In the


case of sole-proprietorship, income tax is treated as a personal expense and
is adjusted in the owners' capital account.

SELF-ASSESSMENT
3. Which one of the following items will not appear on a firm's income
QUESTIONS
statement?
a. Rent expense b. Salaries
C. Insurance expense d. Purchase price of furniture

4. A firm received a rental income of Rs. 50,000 during a year. It,


however, wrongly recorded it as a rental expense. What is the effect
of this error on the firm's income?
a. -50,000 b. +50,000
C. +100,000 d. -100,000

From the information given below, determine the amount of gross profit,
ACTIVITY 2
operating profit and net profit.
Sales Rs.5,000,000 Cost of goods sold Rs. 3,750,000

Salaries Rs. 300,000 Rent Rs. 200,000

Interest paid Rs. 100,000 Interest received Rs. 50,000

4.5 PREPARATION OF BALANCE SHEET


either in a horizontal or
Assets, liabilities and owners' equities are arranged
9.
vertical format, as described in Chapter

Illustration 4.1
books of Naveen Brothers for the
From the balances extracted from the
4:2), prepare a trading and profit and
year ended March 31, 2016 (Table
loss accounts, and balance sheet.

FROM THE BOOKS OF NAVEEN


TABLE 4.2 BALANCES
YEAR ENDED MARCH 31, 2016
BROTHERS FOR THE
(Rs.) (Rs.)

50,000 Plant and machinery 249,200


Opening stock
472,000 Purchase returns 55,200
Sales
(Continued)

NMIMS Global Access- School for Continuing Education


TABLE 4.2 BALANCES FROM THE BOOKS OF
NAVEEN BROTHERS FOR THE YEAR ENDED
MARCHI 31, 2016-CONTINUED
(Rs.) (Rs.)
Depreciation 26,680 Cash in hand 37,680
8,440 Salaries 30,000
Commission received
Insurance 15,200 Accounts receivable 185,400

Carriage on purchases 12,000 Discount allowed 13,120


26,800 Wages 63,560
Furniture
Printing expenses 19,240 Sales returns 66,360

Carriage on sales 8,000 Bank overdraft 160,000


369,120 Purchases 347,160
Capital
92,840 Bad debts 7,200
Accounts payable

Closing stock as on March 31, 2016 is Rs. 148,000.


Solution: See Tables 4.3 and 4.4.

TABLE 4.3 TRADING AND PROFIT AND LOSS ACCOUNTS OF


NAVEEN BROTHERS FOR THE YEAR ENDED MARCH 31, 2016
(Rs.) (Rs.)

To opening 50,000 By Saless 472,000


stock
Purchases 347,160 Less: Returns 66.360
Less: Returns 55,200 405,640
291,960 Closing stock 148,000
Wages 63,560
Carriage on 12,000
purchases
Gross profit c/d 136,120
553.640 553.640
To depreciation 26,680 By grosS 136,120
profit b/d
Insurance 15,200 Commission 8,440
Printing 19,240
expenses
Carriage on 8,000
sales
Salaries 30,000
Discount 13,120
Bad debts 7,200
Net profit 25,120
144.560 144.560

NMIMS Global Access- School


for Continuing Edueation
OF NAVEEN BROTHERs
TABLE 4.4 BALANCE SHEET
AS ON MARCH 31, 2016
Assets (Rs.)
(Rs.)
Liabilities
Plant and machinery 249,200
Capital 369,120
Furniture 26,800
25,120
Add: Net profit
394,240 Closing stock 148,000

92,840 Accounts receivable 185,400


Accounts
payable
160,000 Cash 37,680
Bank overdraft
647,080
647.080

4.6 ADJUSTMENT ENTRIES


Some business activities affect revenues and expenses of more than one
accounting period. Some adjustments are required at the end of the
accounting period to ensure proper measurement of income for an account
ing period and to give a true picture of the state of affairs of the business at aUICKQUICK TIP
the end of the accounting period. Adjustment entries apply both the realiza-
tion and matching principles to transactions affecting two or more periods. Adjustment entries affect both
the income statement and the
Adjustment entries affect both the income statement and the balance sheet.
balance sheet.
This is because the adjustment entries relate to recognition of revenue and
expenses causing a change in the owners' capital. As already known, the
basic accounting equation always holds. Therefore, a change in the owners'
capital is accompanied by a change in assets or liabilities.
These adjustments usually relate to the following:
1. Adjustments needed to convert assets into expenses:
(a) Prepaid expenses
(b) Depreciation and amortization
2. Adjustments needed to convert liabilities into revenue:
(a) Income received in advance or unearned income
3. Adjustments needed to accrue unpaid expenses and
revenue uncollected
(a) Outstanding expenses
(b) Outstanding or accrued income
4. Adjustments needed to account for expected
future expenses:
(a) Provision for bad debts

4.6.1
PREPAID EXPENSES
For some items such as
insurance and rent, payments are made in
These payments may benefit advance.
nay be some unexpired part more than one accounting period, and there
of such expenditure at the end of
Ing year. At
the time of payment, such expenditure is the account-
treated as an asset.

NMIMS Global Access- School


for Continuing Education
received in advance by publishers of a magazine or insurance premium ma may
ho
be received in advance by an insurance company. Such receipts cannot
servic
treated as revenue until the related goods have been supplied
or the ices
Till such time, these receipts are treated as liabilities
have been rendered.
part of the liabilityto
The purpose of the adjustment entry is to transfer that to
during the accounting period.
revenue that has been earned
Part of the payment received, which has not been earned at the end of
unearnod
the accounting year, is known as income received in advance or
income. To account for such income, relevant income account
is debited and
effect of this entr
the income received in advance account is credited. The lia.
is that the amount of income for the accounting period is reduced and a
Income
bility account in the form of income received in advance is created.
from the related head of income
received in advance is shown as deduction in

the profit and loss account and recognized as a liability in the balance sheet.

If the unearned income appears in the trial balance, it implies that the amount
of income earned during the accounting period has already been reduced
by
income will not be deducted
the unearned income. In such a case, the unearned
from the amount of income earned when shown in the trading account or profit
and loss account. This will only be shown as a liability in the balance sheet.

Illustration 4.4
an
An insurance company receives an annual premium of Rs. 50,000 on
insurance policy whose coverage period extends till the mid of the next
accounting year.
The insurance company will show Rs. 25,000 as an unearned income by
way of deduction from the premium income of Rs. 50,000. The unearned
income of Rs. 25,000 will also be shown as a liability in the balance sheet.

4.6.4 OUTSTANDING (ACCRUED) EXPENSES


Outstanding expenses are expenses that are not paid till the end of the
accounting year. For example, salaries payable to the employees of the firm
in the last month of the year are paid in the first month of the next account:
ing period. These expenses relate to the previous accounting year and should
be part of that year's expense.
To account for such expenses, the relevant expense account is debited and the
outstanding expenses account is credited. The effect of this entry is that the
amount of expense for the accounting period is increased anda liability account
in the form of outstanding expenses account is created. The unpaid amounto
expenses is added to the amount of such expenses actually paid during the
accounting year, and the total amount is shown on the debit side of the trading
account orthe profit and loss account. To complete the dual effect, the amount
of outstanding expenses is shown on the liability side of the balance sheet.
If the outstanding expenses appear in the trial balance, it implies that the
amount of expenses paid during the accounting period has already bee
increased by the outstanding expenses. In such a case, the outstandins
expenses will not be added to the amount of expenses paid
the trading account or profit and loss account.
when shown
These will only be shown as a
liability in the balance sheet.
NMIMS Globul Access-School for
Continuing Education
Illustration 4.5
A firm has a monthly salary
year ended on March 31, 2015, expense of Rs. 100,000.
For the accounting
it has paid Rs. 1,100,000
on account of salary. The salary during the year
20115. for the month of March 2015 is paid in
April
The salary expense for the accounting
debited in the profit and loss account willyear ended on March 31, 2013
paid plus Rs. 100,000 outstanding). Rs. 100,000be Rs. 1,200,000 (Rs. 1,100,000
will also be shown as a
liability in the balance sheet.

4.6.5 OUTSTANDING ORACCRUED INCOME


Sometimes interest on securities or deposits is earned, and it is accumulated
over time, but is not due for collection by the firm till the end of the accounting
year. The firm may be entitled to receive dividends declared on its investments,
which are not received by the firm during the accounting year. A firm may have
rendered services, which have not been billed and collected by the end of the
accounting year. All these are examples of accrued or outstanding income. Such
incomes must be accounted for to report the correct amount of income.
To account for such an income, the accrued income account is debited, and
the
the relevant income account is credited. The effect of this entry is that
amount of income for the accounting period is increased and an asset account
loss account, the
in the form of accrued income is created. In the profit and
the accounting year
amount of the relevant income actually received during of accrued
and the amount
is increased by the amount of accrued income,
income is shown as an asset in the balance
sheet.
balance, it implies that the amount
If the accrued income appears in the trial
period has already been increased
of income earned during the accounting
by the accrued income. In such a
case, the accrued income will not be added
shown in the trading account or profit
to the amount of income earned when
shown as an asset in the balance sheet.
and loss account. This will only be

Illustration 4.6
in debentures of a company that bear
A firm has invested Rs. 100,000 on June 30 and December 31 every
14% interest. The interest is
paid
31.
year. The accounting period of the firm ends on March
Rs. 7,000 every sixX months on June
30
The firm is entitled to receive 31, the firm earns an interest for only
and December 31. As on March 6 months between December
31 and Junee
3 months out of the period of
30. The firm will show Rs.
3,500 (50% of Rs. 7,000) as an accrued interest
profit and loss account for the year ended on March 31 by
income in the erediting interest account. The accrued
debiting accrued interest and as an asset in the balance sheet.
interest of Rs. 3,500 will be shown
on June 30 next year, it will debit cash
When the firm receives Rs. 7,000 interest by Ks. 3,500 and credit inter
account by Rs. 7000, credit accrued
est account by Rs. 3,500.
4.6.6 PROVISION FOR BAD AND DOUBTFUL DEBTS
Bad debts are losses that result from debts that default on their obligat.
to pay. Bad debts account is debited and the debtors account is creci dited
The balance in the bad debts account is transferred to the debit ofthe
profit and loss account. The balance in the debtors account is reduced ed by
the amount of bad debts. When the amount of bad debts is given in
trial balance, it means that the balance of debtors (accounts receivahl
account has already been reduced by the amount of bad debts. No furthe rther
adjustment is required in the amount of debtors when it is carried to the he
balance sheet.
There are some debts that have not yet become bad but their recovery ery is
uncertain. As the amount of bad debts is uncertain, a provision is created for
bad and doubtful debts by debiting the profit and loss account and crediting
the provision for bad and doubtful debts account. If there is any balance in
the provision account at the beginning of the accounting year, the same is
reduced from the amount of provision to be maintained at the end of the
accounting year and only the remaining amount is debited to the profit and
loss account.
If the amount of provision to be maintained at the end of the accounting yeat
is less than the balance in the provision account at the beginning of the yean
the difference is credited to the profit and loss account.
In the balance sheet, the provision for bad and doubtful debts is showna
deduction from the balance in the debtors account.

Illustration 4.7
The trial balance of a firm shows a balance of Rs. 100,000 in the debtors
account and a balance of Rs. 8,000 in the bad debts account. The firm
wants to create a provision for bad and doubtful debts equal to 5% of the
balance in the debtors account.
In this case, the balance of Rs. 8,000 in the bad debts account will be
transferred to the debit side of the profit and loss account. The profit and
by
loss account will be further debited by Rs. 5,000 (5% of Rs. 100,000)
giving credit to provision for bad and doubtful debts. The entries in the
balance sheet appear as shown in Table 4.7.

TABLE 4.7 BALANCE SHEET


Liabilities Assets (Rs.) (Rs.)
Debtors 100,000
Less: Provision for bad 5,000
and doubtful debts
95,000
Illustration 4.8
The following balances appear in the trial balance
2015. of a firm as on March 31,

(Rs.)
Bad debts 8,000
Debtors 100,000
Provision for bad and doubtful debts (as on April 1, 2014) 5,000

It is proposed to maintain a provision of 10% on debtors for bad and


doubtful debts.
In this case, Rs. 8,000 of bad debts will be debited to the profit and loss
account. The required provision is Rs. 10,000 (10% of Rs. 100,000). The
existing provision is Rs. 5,000. An additional provision of Rs. 5,000 will
pro
be created by debiting the profit and loss account and crediting the
vision for bad and doubtful debts.
SELFASSESSMENT
intangible assets is called QUESTION
5. The charge for the expiry of benefits from

a. depreciation b. depletion
C. amortization d. none of the above

4.7 ADJUSTED TRIAL BALANCE balance


QUICK TIP

in the ledger, an adjusted trial Ending inventory and provision


After posting of adjustment entries updated account balances.
a summary of for bad debts are not reflected
15 prepared that carries in the adjusted trial balance

Illustration 4.9
bad debts are not reflected in the
Closing stock and provision for because their double entry takes
4.8)
adjusted trial balance (Table and loss accounts. For closing stock,
place in the trading and profit closing stock and to credit the trading
the
the journal entry is to debitaccount. For bad debts provision, profit and
account or profit and loss provision for bad debts is credited
loss account is debited and
TRIAL BALANCE
TABLE 4.8 ADJUSTED
Debit Amount (Rs.) Credit Amount (Rs.)
550,000 1,040,000
Purchases and sales 30,000
Sales returns 18,000
Purchase returns 24,800
Freight on purchases 117,200
Wages and salaries 4,400
Miscellaneous expenses (Continued)
TABLE 4.8 ADJUSTED TRIAL BALANCE-(CONTINUED)
Debit Amount (Rs.) Credit Amount (Rs
Rent 24,000
Insurance 4,000
Audit fees 2,400
Debtors/creditors 226,600 128,600
Printing and advertising 11,000
Commission 2,800
Opening stock 72,000
Cash in hand 25,600
Cash at bank 53,600
Bank loan 40,000

Interest on loan 6,000


Capital 500,000

Drawings 30,000
Property, plant and 540,000
equipment
Depreciation 60,000

Commission outstanding 800


2,000
Rent received in advance
3.000
Outstanding interest
1,758400 1.758.400

balance
financial statements prepared on the basis of adjusted trial
The 4.10.
are presented in Tables 4.9 and
FOR
AND LOSS ACCOUNT
TABLE 4.9 TRADING AND PROFITMARCH 31, 2015
THE YEAR ENDED ON Amount
Amount (Rs)
(Rs.) Particulars
Particulars 1,040,000
72,000 By Sales
To opening stock
Less: 30,000
550,000
Purchases Returns 1,010,000

Less: Returns 18,000 120,000


532,000 Closing
stock
117,200
Wages &
salaries
24,800
Freight on
purchases 1,130,000
384,000
Gross profit c/d
1,130,000 (Continued

Education
Access School for Continuing
NMIMS Global
TABLE 4.9 TRADING AND PROFIT AND LOSS ACCOUNT FOR
THE YEAR ENDED ON MARCH 31, 2015-(CONTINUED)
Amount Amount
Particulars (Rs.) Particulars (Rs.)
To Insurance 4,000 By gross 384,000
profit b/d
Audit fee 2,400 Rent 26,000
Printing and 11,000 Less:
advertising Received
Interest on loan 3,000 in advance 2,000
Add: 3,000 24,000
Outstanding 6,000
Commission 2,000
Depreciation 60,000 Add: 800
Outstanding 2,800

Provision for 11,330


bad debts
Miscellaneous 4,400
expenses
Net profit 311.670
410.800 410.800

31, 2015
TABLE 4.10 BALANCE SHEET AS ON MARCH
Amount Assets Amoumt
Liabilities (Rs.)
Rs.)
Property, plant 600,000
Capital 500,000
and equipment
311.670 Less: Depreciation 60.000
Add: Net profit
540,000
Debtors 226,600
811,670
Less: Drawings 30,000 Less: Provision for 11,330
bad debts
781,670 215,270

Bank loan 40,000


Closing stock 120,000
Add: Outstanding
Cash in hand 25,600
interest 3.000
43,000 Cash at bank 53,600
128,600 Commission 800
Creditors
outstanding
Rent received in 2,000
advance
955,270 955,270
4.7.1 CLOSING ENTRIESS
as these are not
Revenue and expenses accounts are temporary accountS Car.
accounting year. At the end ot the accounting Der
ried forward to the next
to the profit anel
these accounts are closed by transferring their balances
these accounte
account. Revenue accounts have credit balances. To closeand loss account,they
are debited with a corresponding credit to the profit
Similarly, expense accounts
amount being equal to the balance in the account. and loss
a corresponding debit to the profit
account,
are credited with
accounts have been transfere
After the balances in revenue and expense profit and loss account
erred
the
to the profit and loss account, the balance in is more
will
Hh
(when the sum of credit balances than
either show the net profit the sum of debit balances is
sum of debit balances) or the net loss (when
more than the sum of credit balances).
transferring the balance to tha
The profit and loss account is also closed by
of companies).
Owners capital or retained earnings (in the case

4.7.2 POST-CLOSING TRIAL BALANCE


closing the revenue and
The post-closing trial balance is prepared after accounts that
expense accounts. This trial balance consists of only those balance is to
trial
appear in the balance sheet. The purpose of preparing this
as the starting
check the accuracy in posting of closing entries. It also serves
period.
point for recording transactions in the next accounting

4.8 SUMMARY
OUnderstand the relationship between profit and loss account andinter
bal-

ance sheet. Both the profit and loss account and balance sheet are to the
related. A cost relating to the operations of an accounting period or
beyond
revenue earned during the period whose benefits do not extend
that period is shown in the profit and loss account.
Prepare profit and loss account and balance sheet from the given trial
balance, without accounting for any adjustment entries. First, h
tne
gross profit is determined by deducting the cost of goods sold from
sales revenue.
Operating profit is calculated as gross profit minus operating expen
Net profit is calculated by adjusting the operating profit for non-opera
ing revenues, expenses, gains and losses.
owner
Income-tax and drawings are treated as personal expenses of the owne
and are, therefore not shown in the profit and loss account.
Understand how to make adjustments for accruals, deferrals and a
other items. Some business activities affect revenues and expenhe
more than one accounting period. Some adjustments are
end of the accounting period to ensure proper measurement ot
requirea
of
e
for an accounting period and to give a true picture of
the business at the end ofthe accounting the state aueally
of
period. Adjustments
made relating to outstanding expenses, ared of
prepaid expenses, outSta ding
accrued income, income received in advance or unearned income, depr
ciation and amortization, and provision for bad debts.
Prepare profit and loss account and balance sheet after accounting for
adjustment entries. The amounts in the trial balance get adjusted with
the amount of adjustments made. Following the same principles that
apply in the absence of adjustments, the profit and loss account and the
balance sheet are prepared on the basis of the adjusted trial balance.
Prepare closing entries and post-closing trial balance. At the end of
the accounting period, revenue and expense accounts are closed by trans-
fer to profit and loss account. The balance in the profit and loss account
is transferred to owners' capital account or retained earnings account.
Thereafter, post-closing trial balance is prepared that consists of only bal-
ance sheet accounts.

KEY WORDS
1.Adjusted trial balance After posting of adjustment entries in the
ledger, an adjusted trial balance is prepared that carries a summary
of updated account balances.
2. Closing entries At the end of the accounting period, expense and
revenue accounts are closed by transferring their balances to the
profit and loss account. The profit and loss account is also closed by
transferring the balance to the owners' capital or retained earnings
(in the case of companies).
3. Cost of goods sold is equal to the cost of goods available for sale
(beginning inventory + net purchases + direct expenses) minus
ending inventory.
4. Gross profit is the difference between the sales revenue and the
cost of goods sold.
5. Perpetual inventory system keeps a detailed record of each
inventory purchase and sale. The inventory that should be on hand
is available perpetually from these records.
6. Periodic inventory system does not keep a detailed record of
inventory on hand. The value of the ending inventory is determined
by taking a physical inventory count.
7. Operating profit is calculated as gross profit minus operating
expenses.
operations of the
8. Operating expenses are related t0 normal general expenses.
selling and
business and include administrative,
profit for non-
9. Net profit is calculated by adjusting the operating
and losses.
operating revenues, expenses, gains
4.9 DESCRIPTIVE QUESTIONS
1. How is gross profit measured?
2. What are allowances in relation to sales revenue?
3. How is GST treated in accounts?

4. How is the cost of goods sold calculated?


5. What expenses are classified as operating expenses?
6. Define operating profit.
financial
7. State any five adjustment entries made in the preparation of
statements at the end of the accounting period.
8. Differentiate between the terms depreciation and amortization.

4.10 ANSWER KEY


SELF-ASSESSMENT QUESTIONS
Topics Q. No. Answers
Relationship Between Profit and 1. a. Income statement
Loss Account and Balance Sheet
2. d. Dividend
Preparation of Profit and Loss 3. d. Purchase price of furniture
Account
4. d. -100,000
Adjustment Entries 5. c. amortization

4.11 SUGGESTED BOOKS AND E-REFERENCES


SUGGESTED BOOKS
DBhattacharyya, A.K. (2014). Financial Accounting for Business
Managers, Prentice Hall of India.
Anthony, R.N., Hawkins, D.E. and Merchant, K.A. (2015). Accounting
Text and Cases, Tata MeGraw Hill.

E-REFERENCES
Khan M.Y. and Jain, PK. (2010). Management Accounting: Text, Problem
and Cases, Tata McGraw Hill (KJ).
Horngren C.T., Sundem G.L. and Elliot J.A. (2013). Introduction
Financial Accounting, Pearson Education.
CONTENTS

5.1 Introduction
5.2 Accounting Standards Board
Self-Assessment Questions
Activity
5.3 Constitution of Accounting Standard Board of India
5.4 Procedure for Issuing Accounting Standards
Self-Assessment Questions
Activity
5.5 Compliance with Accounting Standards
Self-Assessment Questions
5.6 Implementation of Accounting Standards in lnda
Self-Assessment Questions
5.7 Convergence of Indian Accounting Standards with 1FRS
5.7.1 Applicability of Ind AS
Self-Assessment Questions
5.8 Summary
Key Words
5.9 Descriptive Questions
5.10 Answer Key
Self-Assessment Questiona8
5.11 Suggested Books and E-Referenees
LEARNING OBJECTIVES

to:
Alter reading this chapter, you will be able
standar
Understand the meaning and importance of accounting
in bringing
Understand the role of Accounting Standards Board
out new accounting standards.
howi1s
Understand how new accounting standards are issued and
compliance with accounting standards ensured.
Standards in
Understand the current structure of Accounting
India.

5.1 INTRODUCTION
by accounting bodies spec-
Accounting standards are pronouncements made
ifying the accounting requirements for
recognition, measurement, presen-
events. Entities prepare
tation and disclosure of different transactions and
standards. Financial state
their financial statements based on accounting to make a fair presen-
ments based on accounting standards are expected
financial position and cash flows
tation of an entity's financial performance,
Accounting standards also bring
to different users of financial statements.
financial statements of dif
about uniformity in financial reporting and make
ferent entities comparable.
accounting bodies spec
Accounting standards are pronouncements made by
ifying the accounting requirements for different
transactions and events.
for developing and
Accounting bodies in different countries are responsible
implementing Accounting Standards in their respective countries.

5.2 ACCOUNTING STANDARDS BOARD


formulating the
Accounting bodies in different countries are responsible for
accounting stan
accounting standards applicable to that country. In India,
Accountants
dards are formulated by the Council of the Institute of Chartered objec
of India (ICA) through its Accounting Standards Board (ASB). The
practice
tive of the ASB is to standardize different accounting policies and
so that financial statements prepared by different entities are
reliable and
comparable.

1. The purpose of Accounting Standards is to


a. make a fair presentation of an entity's financial performance,
financial position and cash flows
b. bring about uniformity in financial reporting
make financial statements of different entities comparab
d. All of the above
2 ASB stands for
a. Accounting Standards
Bureau
b. Accounting Standards Bulletin
c. Accounting Standards Board
d. None of the above

Refer to the website https://www.icai.org/post.html?post 3ACTIVITY 1


prehend the objectives and functions of the id=379 and com
accounting standards board
(ASB. Make a note on the Subjects on
which new Accounting Standards
are under preparation.

5.3 CONSTITUTION OF ACCOUNTING


STANDARD BOARD OF INDIA
Some members of the ASB are nominated by ICAI. Other members of the
ASB consist of the following:
1.Nominee of the central government representing the Department of
Company Affairs on the council of the ICAI.
2. Nominee of the central government representing the office of the
Comptroller (Controller) and Auditor General of India on the council of
ICAI
3. Nominee of the central governmment representing the Central Board of
Direct Taxes on the council of ICAI.
Representative of the Institute of Cost and Works Accountants of India.
5. Representative of the Institute of Company Secretaries of India.
6. Representative of Industry Association from "Associated Chambers of
Commerce and Industry (ASSOCHAM)," from Confederation of Indian
Industry (CI) and from Federation of Indian Chambers of Commerce
and Industry (FICCI).
7. Representative of Reserve Bank of India (RB).
8. Representative of Securities and Exchange Board of India (SEBI).
9. Representative of Controller General of Accounts.
10. Representative of Central Board of Excise and Customs.
. Representative of academie institutions from universities and from
Indian institutes of management,
12. Representative of financial institutions.
13. Bminent professionals co-opted by ICAI.

NMIMS Global Access-School for Continuing


Education
82 FINANCIAL ACcOUNTING AND ANALYSIS

or
the Exper
and chairman
14. Chairman of the Research Committee members
if they are not otnerwise of
Advisory Committee of the ICAI,
the Accounting Standards Board
considered appropriate by the
15. Representative of any other body, as
ICAI.
ACCOUNTING
PROCEDURE FOR ISSUING
NOTE 5.4 STANDARDS
| ASB is acommittee by the Council of the Institute
accounting standards are formulated Thereafter, these
under the Institute of In ndia, (ICAI) through its AsB.
Accountants of India
Chartered Accountants of of Chartered Financial Keporting
standards are considered by the National Standards
India (CAlI), which consists accounting Government may prescribe the
Authority (NFRA). The Central the ICAI, in
of representatives from
addendum thereto, as recommended by
government department of Accounting or any recommendations made by
with and after examination of the
academicians, and other consultation
professional bodies (viz., ICAI, the NFRA.
CIL, FICCL, etc.).
ASSOCHAM,

accounting
the following is instrumental in formulating the
SELFASSESSMENT 3. Which of policies and
ditferent accounting
QUESTIONS standards that standardize
practices?
a. Board of Direct Education
b. Accounting
Standards Board (ASB)
C. Board of Secondary Education
d. Corporate Affairs
Board
the Central Government on the
4. Which among the following advises
policy and accounting
formulation and laying down of accounting
standards for adoption by companies.
a. International F'inancial
Reporting Standards (1FRS)
b. Ministry of Corporate
Affairs (MCA)
(NFRA)
c. National Financial Reporting Authority
d. Indian Accounting Standard
(Ind-AS)

In the Indian context, check out the procedure


for issuing accounting
2ACTIVITY considers tne
standards (AS). Note when the council of the institution
final draft.

COMPLIANCE WITH ACCOUNTING


5.5 STANDARDS
Accounting Standards become mandatory from the date specified in
Accounting Standards.

The 1CAI enforces compliance with Accounting Standards through the aud
ing process. It is the duty of the auditors to check whether requirements
the Accounting Standards are complied
case of any deviation, such deviations
have
with or not complied with. In
e
to be reported in the audit reporus
to bring it to the attention of the users of financial
statements.
Compliance with Accounting Standards is also
enforced through the proV
sions of the Companies Act, 2013 in the following manner
1. Sub-section 1 of Section 129 of the Companies Act, 2013 provides that
the financial statements have to comply with the accounting standards
notified under section 133 of the Companies Act, 2013.
Further, the financial statements are required to be in the form or
forms as specified in Schedule III of the Act and the items contained
in such financial statements are required to be in accordance with the
Accounting Standards.
2. Sub-section 5 of Section 129 provides that where the
financial
standards
statements of a company do not comply with the accountingto make the
referred to in sub-section (1), the company is required
following disclosures:
a. Deviation from the accounting standards.
b. The reasons for such deviation.
c. The financial effects, if any,
arising out of such deviation.
Responsibility
directors are required to attach Directors' before
3. The of Directors that is placed
Statement' to the report of Board Section 134
company in general meeting under sub-section 3 of to state that
the
2013. This statement is
required
of the Companies Act, annual accounts, the applicable accounting
of the
in the preparation followed along with proper explanation relating to
standards had been
material departures;
SELFASSESSMENT
disclosures does a company need to make if its QUESTIONS
5. Which of the
following the accounting standards?
do not comply with
financial statements accounting standards
the
a. Deviation from deviation from the accounting standards
b. The reasons for from accounting standards
financial effects of deviation
c. The
d. All of the
above. the forms in
of the Companies Act, 2013 prescribes
prepared?
6. Which Schedule statements are required to be
the financial
which b. Schedule VI
a. Schedule II d. None of the above QUICKQUICK TIP
c. Schedule III
Indian Accounting Standard
ACCOUNTING (abbreviated as Ind-AS) is the
IMPLEMENTATION OF Accounting standard adopted
STANDARDS IN INDIA
5.6 by companies in India and
commonly known as the issued under the supervision
standards, also
standards), have been notie
accounting of Accounting Standards
Currentlv, the following accounung ified: (ASB)
Board
(generally accepted which was constituted
as
Indian GAAP a body in the year 1977.
Accounting Policies
Disclosure of
1.AS 1:
FR Sehool for Contin
2. AS 2: Valuation of Inventories (Revised)
3. AS 3: Cash Flow Statement
. AS 4: Contingencies and
Date (Revised)
Events Occurring he Balance Sheet
atter the

5. AS 5: Net Profit or Loss for the Period,


Prior Period Items and Changes
Chas

in Accounting Policies
6. AS 7: Construction Contracts
7. AS 9: Revenue Recognition
(Revised)
8. AS 10: Property, Plant and Equipment
Exchange Rates
9. AS 11: The Effects of Changes in Foreign
10. AS 12: Accounting for Government Grants

11. AS 13: Accounting for Investments (Revised)


12. AS 14: Accounting for Amalgamations (Revised)
13. AS 15: Employee Benefits (revised
2005)

14. AS 16: Borrowing Costs

15. AS 17: Segment Reporting


16. AS 18: Related Party Disclosures
17. AS 19: Leases
18. AS 20: Earnings Per Share

19. AS 21: Consolidated Financial Statements (Revised)


20. AS 22: Accounting for Taxes on Income

21. AS 23: Accounting for Investments in Associates in Consolidated


Financial Statements
NOTE 22. AS 24: Diseontinuing Operations
Generally Accepted Accounting 23. AS 25: Interim Financial Reporting
Principles (GAAP) are basic
accounting principles and 24. AS 26: Intangible Assets
guidelines which provide the
framework for more detailed 25. AS 27: Financial Reporting of Interests in Joint Ventures
accounting
and comprehensive other 26. AS 28: Impairment of Assets
rules, standards and
industry-specific accounting 27. AS 29: Provisions, Contingent Liabilities
and Contingent Assets (Reviseu
practices.
SELF-ASSESSMENT
QUESTIONS
7. Ind AS stands for
a. Indian Financial Reporting Standards
b. International Financial Reporting Standards
c. International Accounting Standards
d. Indian Accounting Standards
8. IFRS stands for
a. International Financial Reporting Standards
b. Indian Financial Reporting Standards
c. Indian Federal Reporting Standards
d. International Financial Report Structure

5.7 CONVERGENCE OF INDIAN ACCOUNTING


WITH IFRS
STANDARDS
for accounting and financial reporting
Every country has its own set of rules stan-
prepared on the basis of accounting
As a result, financial statements financial statements pre-
prevailing in a country are not comparable to imposes
dards country. This
standards in the othercourntry
pared on the basis of accounting enterprises in a to raise funds !IMPORTANT CONCEPT
a limitation on the ability of business accounting stan-
Such a problem will not arise if the a set
from other countries. or with
consistent with each other standards International Financial
two countries are either
dards in the
can either adopt global
accounting Reporting Standards (IFRS)
of global standards. Countries standards with
convergence of their accountingdepartures from set common rules so that
as they are or can achieve may be financial statements can
In convergence, there
global accounting standards. respect of some accounting treatments. be consistent, transparent
in
global accounting standards accounting standards and comparable around the
financial reporting and benefits: world.
Convergence of a country's following
standards has the
with the global accounting capital inflows in a
growth of international business and
1. It leads to
country.
investors from other countries to better
ability of country.
2. It improves the opportunities in that
understand investment at a lower cost.
raise capital from foreign markets
8. Companies can pro-
Standards Committee (IASC) that represents 1973
International Accounting countries was set up in
of more than 75 many account-
tessional accounting bodies accounting standards.
lt released
to develop international period 1973 to 2001. LASC was restructured 2001,
and
the (IASB) in The
ing standards during Accounting Standards Board Finaneial
renamed as International are referred to as International ac. STUDY HINT
pronouncements made by IASB by LASC are referred to
while those made
ieporting Standards (IFRS) Standards (TAS). The convergence of accounting
nternational Accounting by LASC, internrot eta- standards refers to the goal
issued by LASB, LAS IsSued IFRRS of establishing a single set of
comprises of IFRS Committee (SIC) and
RStlons issued by the
Standards Interpretations establishes broad principles
oe accounting standards that will
of IASB. IFE accountin be used internationally.
nterpretation Committee directions relaung tO Speclne treatment of
accounting rather than

e Sahool fop Contini


bransactions. Different countries are in the process of either adopting
ifying their own or mo.
nod.
accounting standards to make them consistent withIFRS
The latter process is
referred to as Convergence.
The Institute of Chartered Accountants of India (ICA), in consultation
wi
he Giovernment of India, decided against outright adoption of IFRS. Se ome
departures have been made from IFRS which reflect difference in the eco
nomic environment of India and the economic environment that underlies
IFRS.
In a move towards convergence with IFRS, in 2007, the ICAI commenea
the process of developing a complete set of accounting standards that ar
converged with" IFRS. These are known as Indian Accounting Standarde
or Ind AS.
lmplementing the new accounting standards, the Mlinistry of Corporata
Affairs (MCA) of India has notified The Companies (Indian Accounting
Standards) Rules, 2015. The following Ind AS have been notified under the
rules:
1. Indian Accounting Standard First-Time Adoption of Indian
(Ind AS) 101 Accounting Standards
2. Indian Accounting Standard Share-Based Payment
(Ind AS) 102
3. Indian Accounting Standard Business Combinations
(Ind AS) 103
4. Indian Accounting Standard Insurance Contracts
(Ind AS) 104
5. Indian Accounting Standard Non-Current Assets Held for Sale
(Ind AS) 105 and Discontinued Operations
6. Indian Accounting Standard Exploration for and Evaluation of
(Ind AS) 106 Mineral Resources
7. Indian Accounting Standard Financial Instruments:
(Ind AS) 107 Disclosures
8. Indian Accounting Standard Operating Segments
(Ind AS) 108
9. Indian Accounting Standard Financial Instruments
(Ind AS) 109
10. Indian Accounting Standard Consolidated Financial
(Ind AS) 110 Statements
11. Indian Accounting Standard Joint Arrangements
(Ind AS) 111
12. Indian Accounting Standard Disclosure of Interests in Other
Ind AS) 112 Entities
13. Indian Accounting Standard Fair Value Measurement
(Ind AS) 113
14. Indian Accounting Standard Regulatory Deferral Accounts
(Ind AS) 114
15. Indian Accountig Standard Revenue from Contracts with
(Ind AS) 115 Customers
16. Indian Accounting Standard Leases
(Ind AS) 116
17. Indian Accounting Standard
(Ind AS)1 Presentation of Financial
18. Indian Accounting Standard Statements
(Ind AS)2 Inventories
19. Indian Accounting Standard
(Ind AS) 7 Statement of Cash Flows
20. Indian Accounting Standard
Accounting Policies, Changes in
(Ind AS)8 Accounting Estimates and Errors
21. Indian Accounting Standard Events after the Reporting Period
(Ind AS)10
22. Indian Accounting Standard Income Taxes
(Ind AS) 12
23. Indian Accounting Standard Property, Plant and Equipment
(Ind AS) 16
24. Indian Accounting Standard Employee Benefits
(Ind AS) 19
25. Indian Accounting Standard Accounting for Government Grants
(Ind AS) 20 and Disclosure of Government
Assistance
26. Indian Accounting Standard The Effects of Changes in Foreign
(Ind AS) 21 Exchange Rates
27. Indian Accounting Standard Borrowing Costs
(Ind AS) 23
28. Indian Accounting Standard Related Party Disclosures
(Ind AS) 24
29 Indian Accounting Standard Separate Financial Statements
(Ind AS) 27
30. Indian Accounting Standard Investments in Associates and
Ind AS) 28 Joint Ventures
31. Indian Accounting Standard Financial Reporting in
(Ind AS) 29 Hyperinflationary Economies
32. Indian Accounting Standard Financial Instruments:
(Ind AS) 32 Presentation
33. Indian Accounting Standard Earnings per Share
(Ind AS) 33
34. Indian Accounting Standard Interim Financial Reporting
(Ind AS) 34
35. Indian Accounting Standard Impairment of Assets
(Ind AS) 36
36. Indian Accounting Standard Provisions, Contingent Liabilities
(Ind AS) 37 and Contingent Assets
Intangible Assets
37. Indian Accounting Standard
(Ind AS) 38
38. Indian Accounting Standard Investment Property
(Ind AS) 40
Agriculture
39. Indian Accounting Standard
(Ind AS) 41
ESSMENT 9. What among the following
of a country's accounting standards with
NS global standards leads to more investment opportunities, growth of
international capital flows, and lower cost of capital.
a. Convergence b. divergence
C. discrepancy d. variations
10. ICAI and MCA decided to
b. recreate the AS
a. adopt the IFRS
C. converge with IFRS d. differ on their mandates

5.7.1 APPLICABIILITY OF IND AS


for companies whose
April 1, 2016, Ind AS have been made mandatory listing on
From or are in the process
of any
equity and/or debt securities are listed worth of Rs. 500 crore
India and having a net
stock exchange in India or outside
or more. having a net worth
April 1, 2016, all listed and unlisted companies accounting standards.
From to follow the new
above Rs. 500 crore are required applicable to other entities
having a net worth
The deadline was also made joint venture or associate
500 crore or more and to holding, subsidiary,
of Rs. of entities.
companies of these two classes for:
have been made mandatory
From April 1, 2017, Ind AS securities are listed or are in
equity and/or debt a net
1. Companies whose or outside India having
-

being listed within India


the process of crore.
worth of less than Rs. 500 250 crore
that are unlisted having a net worth of Rs.
2. Other companies, than Rs. 500 crore.
or more but less companies of these entities
venture or associate
Holding, subsidiary, joint this deadline.
have also to comply with or
on SME
in the process of listing continue
securities are listed
Companies whose required to apply Ind AS. Such
companies shall
exchanges are not
accounting standards unless they choose other-
existing
to comply with the
from
wise.
to follow Ind AS norms on a voluntary basis the
Companies were also allowedthese companies cannot subsequently
revoke
April 1, 2015. However,
JICK TIP norms. otne
venture and acco
company's overseas subsidiary, associate, jointstatements in
n
voluntarily An Indian financial
orporate IND AS similar entities can prepare their stand-alone jurisdiction. However, such co
of the concerned
s for accounting dance with requirements to prepare their consolidated financial statement*
ning on or after panies are required
accordance with the Ind AS. Ind AS can
llow
continue to fol
not required to follow Standar
Companies which are as prescribed in Companies (Accounting
accounting standards accounting standards formulated by ing
are the earlier
to banking, insurance and non-ban
Rules, 2006. These
applicable
Ind AS are also not
finance companies

DIMIMSs Global Access-School for Continuing Education


5.8 SUMMARY
n Understand the meaning and
Accounting standards are importance of accounting standards
pronouncements made by accounting
bodies specitying the
accounting requirements for different transac
tions and events. Accounting standards also bring about uniformity 1n
financial reporting and make financial statements of different entities
comparable.
Understand the role of Accounting Standards Board in bringing
out new accounting standards. Accounting Standards Board (ASB) is
instrumental in formulating the accounting standards that standardize
different accounting policies and practices so that financial statements
prepared by different entities are reliable and comparable.
Understand how new accounting standards are issued and how is com
pliance with accounting standards ensured. The Central Government
thereto,
may prescribe the Standards of Accounting or any addendum
with and after examina-
as recommended by the ICAI, in consultation
Financial Reporting
tion of the recommendations made by the National
Authority (NFRA).
Standards in India.
Understand the current structure of Accounting
Indian companies are required
Except for some categories of companies,accounting standards that are con-
follow the new 'Ind AS'. These are
to departures
means that there are certain
verged to IFRS. Convergence to follow the existing
companies are required
from IFRS in Ind AS. Other known as the Indian GAAP (generally
accounting standards commonly
accepted accounting principles).
KEY WORDS
are pronouncements made by accounting
1. Accounting Standards
accounting requirements for different
bodies specifying the
transactions and events. is the report of
Board of
Statement annual accounts,
Directors' Responsibilityin the preparation of the along with
Directors that states that been followed
accounting standards haddepartures;
the applicable relating to material
proper explanation principles,standards,
a common set of and
Standards) is financial accounting
AS (Accounting basis of
that define the
3.
and procedures
policies and practices. accounting principles) is a collection
accepted standards for financial
GAAP (generally rules and
4. commonly-followed accounting
of accounting
reporting. Standards (IAS) are olderStandards Board
Accounting Accounting
5. national International
the international standard-setting body
based
issued by
standards independent Financial
International
(LASB), an replaced in 2001 by Int
were
in don. The IAS (IFRS).
Reporting Standards
6. IFRS (International Financial Reporting
Standards) are accountin
standards that are issued by the International
Board (IASB) with the objective of Accounting Standard
providing a common accountin
language to increase transparency in
the presentation of finanei
information. al
7. Indian Accounting Standard (Ind-AS) is the
Accounting standard
adopted by companies in India and issued under
the supervisionaf
Accounting Standards Board (ASB) which was
constituted as a bod
ody
in the year 1977.
8. International Accounting Standards Board (TASB) is an independent.
private-sector body that develops and approves International Financial
Reporting Standards (TFRSs).

5.9 DESCRIPTIVE QUESTIONS


1. What are Accounting Standards? Why are these necessary for the
preparation and presentation of financial statements?
2. What is the objective of the Accounting Standards Board?
3. Explain the procedure of issuing Accounting Standards in India.
4. How is compliance with Accounting Standards enforced in India?
5. How does 'Directors' Responsibility Statement' secure companies
compliance with Accounting Standards?
6. What advantage can Indian companies derive by preparing and
presenting their financial statements in accordance with IFRS?
7. Are Ind AS exactly similar to IFRS? Comment.
8. Some categories of companies have been exempted from adoption of
Ind AS. Name these categories.

5.10 ANSWER KEY


SELFASSESSMENT QUESTIONS
Topics Q. No. Answers
Accounting Standards 1. d. All of the above
Board
2. C. Accounting Standards Board
(ASB))
Procedure for Issuing 3. b. Accounting Standards Board
Accounting Standards
4. c. National Financial Reporting
Authority (NFRA)
Compliance with 5. d. All of the above
Accounting Standards
6. c. Schedule III

-
NMIMS Global Access School for Continuing Education
Topics Q. No. Answers
Implementation of 7. d. Indian Accounting
Accounting Standards Standards
in India
8. a. International Financial
Reporting Standards
Convergence of Indian a. Convergence
Accounting Standards
with IFRS
10. C. Converge with IFRS

5.11 SUGGESTED BOOKS AND E-REFERENCES


SUGGESTED BOOKS
Raiyani J.R. and Lodha G. (2012). International Financial Reporting
Standards ((FRS) and Indian Accounting Practices. Ingram.
Epstein B.J. and Mirza A.A. (2006). Wiley IFRS 2006: Interpretation and
Application of International Financial Reporting Standards. John Wiley
&Sons.
P and Bhalla K. (2019). Scanner Cum Compiler
Financial
Sharma
Reporting (CA-Final). Taxman.
Reporting with Applicable
Rawat D.S. (2019). Students' Guide to Financial
Ind ASs (CA-Final). Taxman.

E-REFERENCES
Accounting Standards (Ind AS): An Overview (revised
2019).
DIndian
https://resource.cdn.icai.org/55845indas45234a.pdf
http://taxclubindia.com/simple/2013-14/IFRS%
IFRS Pocket Guide 2013.
%2BGuide_2013_PWC.pdf
2BPocket
Accountants of India (2019). Indian Accountina
DThe Institute of CharteredOverview (Revised2019). https://resource.cdn.
Standards (IND AS): An
icai.org/55845indas45234a.pdf

Advanced Corporate Accounting. http://www.universityofcalicut.info/


SDE/advanced_corporate_accounting_onl3April2016.pdf
6
CHA PT E R

FINANCIAL REPORTING STANDARDS II

CONTENTS

6.1 Generally Accepted Accounting Principles


Self-Assessment Questions
6.2 International Financial Reporting Standards
6.2.1 Advantages of Adopting IFRS
Self-Assessment Questions
6.2.2 Indian Accounting Standards
6.2.3 Comparisons of Indian GAAR IFRS and Ind AS
Self-Assessment Question
6.3 Summary
Key Words
6.4 Descriptive Questions
6.5 Answer Key
Self-Assessment Questions
6.6 Suggested Books and E-References
LEARNING OBJECTIVES

After reading this chapter, you will be able to:


Understand the concept of generally accepted accounting princi.
ples (GAAP).
Understand the structure of International Financial Reporting
Standards (IFRS) and advantages of adopting them.
Understand the key differences between Indian GAAP IFRS and
Indian Accounting Standards (Ind AS) with respect to important
accounting transactions and events.

6.1 GENERALLY ACCEPTED ACCOUNTING


PRINCIPLES
Generally accepted accounting principles (GAAP) are a set of conventions,
! IMPORTANTCONCEPT
rules and procedures that define the accepted accounting practice at a par
Generally accepted ticular time. These result from a broad agreement on the theory and practice
accounting principles (GAAP) of accounting at a particular time. These principles are "generally
accepted"
are a set of conventions, rules because an authoritative body has set them or the accounting profession
and procedures that define widely accepts them as appropriate. The purpose of GAAP
is to ensure that
the accepted accounting the information provided in the financial statements is
reliable
standable to the users. The users should be able to meaningfully and under
practice at a particular time. compare the
current performance of a business entity with its past
performance of other business entities. The GAAP performance and the
to time as the circumstances or the information keep changing from time
needs of the users change.
In India, the sources of GAAP include the
accounting standards and the pronouncementsCompanies Act, 2013, Indian
Companies can also voluntarily adopt of the accounting profession-
Standards (IFRS) for International Financial Reporting
financial reporting.

SELFASSESSMENT 1. GAAP stands for


QUESTIONS
a. generally accepted accounting
b. generally agreed accounting principles
c. protocols
general accounting accreditation
d. generally accepted pool
accounting protocols
2. IFRS stands for
a. Indian Financial
Reporting Standards
b. International Fund
Reporting Standards
c. International Financial
d. Indian Financial Reporting Standards
Reportage Standards

NMIMS Global Access


School for Cantin
6.2 INTERNATIONAL FINANCIAL REPORTING
STANDARDS
As the business world becomes closer in its financial and trade ties, many
countries are moving towards International Financial Reporting Standards
TFRS). IFRS are common accounting rules for financial reporting. IFRS
comprise the following:
1. Two series of standards those explicitly called International Financial
-
NOTE
Reporting Standards and the older series of International Accounting
Standards (LAS) and IFRS are common accounting
rules for financial reporting.
2. Two series of interpretations those issued by the former Standing
Interpretations Committee (SIC) and those issued by the existing
International Financial Reporting Interpretations Committee (TFRIC)
of the International Accounting Standards Board.

6.2.1 ADVANTAGES OF ADOPTING IFRS


STUDY HINT
Adoption of IFRS has many advantages. Investors can compare financial
statements of companies located in different countries and decide where to International Financial
invest money. It becomes easier for companies to raise money outside their Reporting Standards (IFRS)
home country and for countries to attract foreign investment. As IFRS are can be adapted to specific
principle-based rather than rule-based, these can be adapted to specific business conditions in a
business conditions in a country. country.

3. IFRS are common accounting rules for SELF-ASSESSMENT


a. marketing budget QUESTIONS
b. financial reporting
C. Indian firms
d. BSE and NSE
4. Based on which among the followings, investors can compare
financial statements of companies located in different countries and
decide where to invest money?
a. Ind AS
b. ICAI rules
c. Indian GAAP
d. IFRS

QUICK TIP

6.2.2 INDIAN ACCOUNTING STANDARDS Indian Accounting


Standard (abbreviated as
RS has many advantages, and hence many countries have adopted it as Ind-AS) is the Accounting
elr national standard. India has decided to converge its accounting stan standard adopted by
aards with IFRS instead of adopting IFRS. In convergence companies in India and
a move
n rRS, in 2007 the ICAI commenced the process towardsof developing a com- issued under the supervision
PACLe set of Accounting Standards Board
of accounting standards that are "converged with" IFRS. These
(ASB) which was constituted as
a body in the year 1977.
AS. Ind AS have certa:
are known as Indian Accounting Standards or Ind tain
modifications to IFRS to reflect "Indian conditions"
NOTE
6.2.3 COMPARISONS OF INDIAN GAAP IFRS AND IND AS
Indian GAAP differs but those are not IFRS
accounting principles
from Lnd AS norms are converged with the IFRS,
the old accountine
auditing standards with
and equivalent. Further, some companies in India will follow
interest
prospective investors may
which standards (Indian GAAP), while others follow Ind AS. It would be of
familiar in other be to see the key differences between the requirements of Indian GAAP IFRS
countries, such
as U.S. GAAP and IFRS. and Ind AS. These differences in respect of some important types of account
ing transactions and events are presented in Table 6.1.

SELFASSESSMENT
QUESTION 5. Instead of adopting, India has decided to converge its accounting
standards with
a. IFRS
b. ICAI rules
C. International Accounting Standards (IAS)
d. International Accounting Standards Board (IASB)

NMIMSGlobal
Access School for Continuing
Eduo
for Continuing Education
NMIMS Global Access- School
IND AS
TABLE 6.1 COMPARISON OF INDIAN GAAR IFRS AND
Presentation
of Financial IFRS Ind AS
Statements Indian GAAP A complete set of financial
A complete set of financial
Components The requirements for the statements comprises of a statements under Ind AS 1
of financial presentation of financial statements statement of financial position, comprises of a balance sheet at
statements are set out in Statutes that govern a statement of comprehensive
the end of the period (including
the entity. For instance, statement of changes in equity
income, a statement of changes which is presented as a part of the
Schedule VI to the Companies in equity, a statement of cash
Act sets out financial statement flows and notes to the financial
balance sheet), statement of profit
requirements in case of companies; statements including summary of and loss, a statement of cash flows
Schedule III to the Banking accounting policies. and notes including summary
Regulation Act, 1949 (for banks) sets of accounting policies and other
out financial statement requirements explanatory information.
in case of banks.
Only illustrative formats for Ind AS 1 does not include
Formats Schedule VI prescribes mandatory any illustrative formats for
formats for presentation of presentation of financial
statements have been given. the presentation of financial
balance sheet and statement of statements though Ind AS 27
profit and loss. However, the samne does set out the form in which
stand modified, if any change is consolidated financial statements
required for compliance with the are to be presented.
requirements of the Companies Act
including Accounting Standards.
Omissions or misstatements Similar to IFRS.
Definition of Financial statements should disclose
all "material" items, the knowledge are material if individually or
material" collectively they could influence
of which might influence the
decisions of the user of the financial the economic decisions that users
statements. take on the basis of financial
statements.
(Continued)
TABLE 6.1 COMPARISON OF INDIAN GAAR IFRS AND IND AS
Presentation
of Financial IFRS Ind AS
Statements Indian GAAP
A complete set of financial A complete set of financial
Components The requirements for the statements under Ind AS 1
presentation of financial statements statements comprises of a
of financial statement of financial position, comprises of a balance sheet at
statements are set out in Statutes that govern the end of the period (including
the entity. For instance, a statement of comprehensive
income, a statement of changes statement of changes in equity
Schedule VI to the Companies in equity, a statement of cash which is presented as a part of the
Act sets out financial statement flows and notes to the financial balance sheet), statement of profit
requirements in case of companiesS; statements including summary of and los, a statement of cash flowss
Schedule III to the Banking accounting policies. and notes including summary
Regulation Act, 1949 (for banks) sets of accounting policies and other
out financial statement requirements explanatory information.
in case of banks.
Only illustrative formats for Ind AS 1 does not include
Formats Schedule VI prescribes mandatory
presentation of financial any illustrative formats for
formats for presentation of
balance sheet and statement of statements have been given. the presentation of financial
profit and loss. However, the same statements though Ind AS 27
stand modified, if any change is does set out the form in which
required for compliance with the consolidated financial statements
requirements of the Companies Act are to be presented.
including Accounting Standards.
Definition of Financial statements should disclose Omissions or misstatements Similar to IFRS.
"material" all "material" items, the knowledge are material if individually or
of which might influence the collectively they could influence
decisions of the user of the financial the economic decisions that users
statements. take on the basis of financial
statements.
(Continued)
AS-CONTINUED
TABLE 6.1 COMPARISON OF INDIAN GAAR IFRS AND IND
Presentation
of Financial Ind AS
Statements Indian GAAP IFRS
Entities are not permitted to Similar to IFRS.
Extraordinary Extraordinary items are disclosed
items separately in the statement of profit present any item of income or
and loss and are included in the expense as extraordinary.
determination of the net profit or loss
for the period.
A statement of changes in equity A statement of changes in equity
Statement of A statement of changes in equity is
is presented showing: (a) the total ispresented as part of the balance
changes in equity not required.
comprehensive income for the sheet. The statement of changes in
Movements in share capital, retained (b) effects of retrospective equity contains information which
earningS and other reserves are to be period,
application or restatement of is similar to that under IFRS.
presented in the notes to accounts.
each component of equity, (c)
for each component of equity, a
reconciliation between opening
and closing balances, separately
disclosing changes resulting from:
(i) profit or loss, (ii) each item of
other comprehensive income and
(iii) transactions with owners in
their capacity as owners, showing
separately contributions by and
distributions to owners and
changes in ownership interests in
subsidiaries that do not result in a
loss of control.
Presentation
of Financial IFRS Ind AS
Statements Indian GAAP
AS 1 does not require an entity to Requires disclosure of information Similar to IFRS.
Capital the management of capital
disclose information that enables users about
financial statements to evaluate and compliance with capital
of its
the entity's objectives, policies and requirements.
processes of managing capital.
Changes in depreciation method Similar to IFRS.
Change in Requires retrospective
re-computation of depreciation. are considered as change in
the method of accounting estimate, and applied
depreciation Any excess or deficit on such re-
computation is required to be adjusted prospectively.
in the period in which such change is
effected. Such a change is treated as
a change in accounting policy and its
effect is quantified and disclosed.
IFRS Ind AS
Revenue Indian GAAP to IFRS.
Revenue is the gross inflow of Similar
Definition Revenue is the gross inflow of cash,
receivables or other consideration economic benefits during the
arising in the course of ordinary period arising in the course of
entity
activities from the sale of goods, from ordinary activities of an in
the rendering of services, and use by when these inflows result
other than
others of enterprise resources yielding increases in equity,to contributions
interest, royalty and dividends. increases relating
from equity participants.
On AS 30 becoming effective, there
will be no difference between AS 9
and IAS 18. (Continued)
Presentation
of Financial IFRS Ind AS
Statements Indian GAAP
entity to Requires disclosure of information Similar to IFRS.
Capital AS 1 does not require an
disclose information that enables users about the management of capital
evaluate and compliance with capital
of its financial statements to
the entity's objectives, policies and requirements.
processes of managing capital.
Changes in depreciation method Similar to IFRS.
Change in Requires retrospective
re-computation of depreciation. are considered as change in
the method of accounting estimate, and applied
depreciation Any excess or deficit on such re-
computation is required to be adjusted prospectively.
in the period in which such change is
effected. Such a change is treated as
a change in accounting policy and its
effect is quantified and disclosed.
IFRS Ind AS
Revenue Indian GAAP Similar to IFRS.
Revenue is the gross inflow of cash, Revenue is the gross inflow of
Definition economic benefits during the
receivables or other consideration
arising in the course of ordinary period arising in the course of
activities from the sale of goods, from ordinary activities of an entity
the rendering of services, and use
byy when these inflows result in
others of enterprise resources yielding increases in equity, other than
interest, royalty and dividends. increases relating to contributions
from equity participants.
On AS 30 becoming effective, there
9
will be no difference between AS
and IAS 18. (Continued)
TABLE 6.1 COMPARISON OF INDIAN GAAR IFRS AND
IND AS-CONTINUED
Revenue Indian GAAP IFRS
Measurement Ind AS
Revenue is recognized at the nominal Fair value of revenue
from Similar to IFRS.
amount of consideration receivable. sale of goods and
services
On AS 30 becoming effective, there when the inflow of cash and
will be no difference between AS 9 cash equivalents is deferred is
and IAS 18. determined by discounting all
future receipts using an implied
rate of interest. The difference
between the value and nominal
amount of consideration is
recognized as interest income
using the effective interest
Services rendered method.
No specific guidance in AS 9. Fair value of servieces provided
However, the Guidance Note on Similar to IFRS.
is measured with reference to
Accounting for Dot.com compamies non-barter transactions that
provides guidance for advertising occur frequently, representing a
barter transactions which is similar substantial number of transactions.
to IFRS. Consideration involves cash or
other securities that has a reliable
measure of fair value and do not
involve transactions with the
same counterpart to the barter
transaction.

TABLE 6.1 COMPARISON OF INDIAN


Related Party GAAR IFRS AND IND AS-CONTINUED
Diselosures Indian GAAP
Items to be IFRS Ind AS
Generally disclosed in aggregate.
disclosedd The amount of transactions with
Disclosure of the volume of Similar to IFRS.
related parties and the amount of
transactions with related parties, either outstanding
balances including
as an amount or as an commitments.
appropriate
proportion and amounts or appropriate
proportions of outstanding items.
Earnings Per
Share Indian GAAP IFRS
Extraordinary EPS with and without extraordinary There is no concept
Ind AS
items of Similar to IFRS.
items should be presented. extraordinary item,
Mandatorily No specific requirement, Ordinary
convertible shares to be issued
on coaversion of a mandatorily
Similar to IFRS.
instrument
convertible instrument are ineluded
n the caleulation of basie EPS from
he date the conlract is ntered ntO.
Ses
Indiann
R
GAAP Ind AS
segnen
sz On of
Recuires an enterprise to identilyancl erting seE22Onts are identifled Similar to FRS.
LwosSets ol segmentS ness
(buts1 assoc on the tinaneial information
apB
or inte Gn
ing rislks and rowards
yLom
that s rexularly reviewed by the
ieooratinK docision maker in
management personnel servin only ss ad in aoai periormamee
e Lrting poini for he identification
Related Party Ind AS
IFRS
Disclosures Indian GAAP Similar to IFRS.
The amount of transactions with
Generally disclosed in aggregate. related parties and the amount of
Items to be
disclosed Disclosure of the volume of balances including
either outstanding
transactions with related parties, commitments.
as an amount or as an appropriate
appropriate
proportion and amounts or
proportions of outstanding items.

Ind AS
Earnings Per IFRS
Share Indian GAAP Similar to IFRS.
extraordinary There is no concept of
Extraordinary EPS with and without extraordinary item.
items should be presented. Similar to IFRS.
items
Ordinary shares to be issued
Mandatorily No specific requirement. on conversion of a
mandatorily
convertible convertible instrument are included
basic BPS from
instrument in the calculation of
entered into.
the date the contract is
Ind AS
IFRS Similar to IFRS.
Segments Indian GAAP Operating segments are
identified
identify
Determination of Requires an enterprise to information
based on the financial
segments (business and reviewed by the
segments two sets of that is regularly
rewards
geographical), using risks and chiefoperating decision maker in
enterprise's system resources
approach, with the deciding how to allocate
reporting to key performance.
of internal financial serving only as and in assessing
management personnel
for the identification
the starting point
of such segments.
TABLE 6.1 COMMPAKISON OI INDIAIN GAA; L

Related Party
IFRS Ind AS
Disclosures Indian GAAP
Generally disclosed in aggregate. The amount of transactions with Similar to IFRS.
Items to be
disclosed
related parties and the amount of
Disclosure of the volume of
Oustanding balances including
transactions with related parties, either
commitments.
as an amount or as an appropriate
proportion and amounts or appropriate
proportions of outstanding items.

Earnings Per
Share Indian GAAP IFRS Ind AS

Extraordinary EPS with and without extraordinary There is no concept of Similar to IFRS.
items items should be presented. extraordinaryitem

Mandatorily No specific requirement. Ordinary shares to be issued Similar to IFRS.


convertible on conversion of a mandatorily
instrument convertible instrument are included
in the calculation of basic EPS from
the date the contract is entered into.

Segments Indian GAAP IFRS Ind AS


Determination of Requires an enterprise to identify Operating segments are identilied Similar to lFRS.
segments two sets of segments (business and based on the financial information
geographical), using risks and rewards that is regularly reviewed by the
approach, with the enterprise's system chief operating decision maker in
of internal financial reporting to key deciding how to allocate resources
management personnel serving only as and in assessing performance
the starting point for the identification
of such segments.
6.3 SUMMARY
Generally accepted accoun
OUnderstand the concept of GAAP conventions, rules and procedureg
nting

principles (GAAP) are a set of at a particular time.


that

define the accepted accounting practice


Financial Repori.
OUnderstand the structure of International
of adopting them. IFRS
orting
Standards (IFRS) and advantages prise
adon
two series of interpretations. By
two series of standards and statements of compani
ing IFRS, investors can compare financial to invest money
located in different countries and decide where
GAAR IFRS and Ind
Understand the key differences between Indian
AS with respect to important accounting
transactions and evente
There are major differ
Ind AS are more or less converged with IFRS.
respect to the presenta
ences in the Indian GAAP and the Ind AS with
presentation of cash
tion of financial statements, inventory accounting,
flows, revenue recognition, etc.

set
KEY WORDS 1. GAAP or generally accepted accounting principles are a
accepted
of conventions, rules and procedures that define the
accounting practice at a particular timne.
2. IFRS are common accounting rules for financial reporting developed
by International Accounting Standards Board (LASB).

6.4 DESCRIPTIVE QUESTIONS


1. Explain the term 'Generally Accepted Accounting Principles'.
2. What all is included in Tnternational Financial Reporting Standards
(IFRS)?
3. What advantages can companies derive by adopting IFRS?
4. How is convergence with accounting standards different from adoption
of accounting standards?

6.5 ANSWER KEY


SELFASSESSMENT QUESTIONS

Topics Q. No. Answers


Generally Accepted 1. a. generally accepted accounting
Accounting Principles principles
2. C. International Financial Reportine
Standards
International Financial 3. b. financial reporting
Reporting Standards
4. d. IFRS
5. a. IFRS
STANDARDS I 103
INANCIAL REPORTING

G.6 SUGGESTED BOOKS AND E-REFERENCES


SUGGESTED BOOKs
n Sekar G., Saravana Prasath B. (2018). Padhuka's Students' Guide on
Financial Reporting (For CA Final New Syllabus). 13th Edition, Wolters
Kluwer India Pvt. Ltd.
n Welkins S.K. (2019). Financial Reporting Made Easy. Bharat Law House
Pvt. Ltd.

E-REFERENCES
a The Institute of Chartered Accountants of India (2019). Indian Accounting
Standards (IND AS): An Overview (Revised 2019). https://resource.cdn.
icai.org/55845indas45234a.pdf
a Advanced Corporate Accounting. http://www.universityofealicut.info/
SDE/advanced_corporate_accounting_on13April2016.pdf
7 R
C H AP T E

CORPORATE FINANCIAL STATEMENTS

CONTENTS

7.1 Introduction
7.2 Books of Accounts to be Kept by a Company
7.3 Financial Statements
7.3.1 Consolidated Financial Statements
7.3.2 Statement of Changes in Equity
7.4 Assets
7.4.1 Non-Current ASsets
7.4.2 Current Assets
Activity
7.5 Equity
7.5.1 Equity Share Capital
7.5.2 Preference Shares
Self-Assessment Question
7.6 Other Equity
7.6.1 Share Application Money Pending Allotment
7.6.2 Capital Reserve
7.6.3 Securities Premium Reserve
7.6.4 Retained Earnings
7.6.5 Revaluation Surplus
7.7 Liabilities
7.7.1 Non-Current Liabilities
7.7.2 Current Liabilities
Self-Assessment Questions
7.8 Contingent Liabilities and Commitments
7.8.1 Statement of Profit and Loss
7.9 Revenue from Operations
7.9.1 Revenue Recognition
Activity
7.10 Other Income
Activity
7.11 Expenses
7.11.1 Cost of Materials Consumed
7.11.2 Purchases of Stock-in-Trade
Work-in-Progress and
7.11.3 Changes in Inventories of Finished Goods,
Stock-in Trade
7.11.4 Employee Benefits Expense
7.11.5 Finance Costs
7.11.6 Depreciation and Amortization Expenses
Activity
7.11.7 Other Expenses
7.12 Profit Before Exceptional Items and Tax
7.13 Exceptional Items
Activity
7.14 Tax Expense
7.15 Profit (Loss) for the Period from Continuing Operations
7.16 Discontinued Operations
7.17 Profit (Loss) for the Period
7.18 Other Comprehensive Income
Activity
7.19 Earnings per Share
7.19.1 Basic Earnings per Share
7.19.2 Diluted Earnings per Share
7.20 Income Taxes
7.20.1 Advance Tax
7.20.2 Provision for Tax
7.21 Dividend
7.21.1 Interim Dividend
7.21.2 Final Dividend
7.21.3 Accounting Treatment of Dividends
7.22 Summary
Key Words
7.23 Deseriptive Questions
7.24 Answer Key
Self-Assessment Questions
7.25 Suggested Books and E-References

NMIMS Global Access-School for Continuing Education


STATEMENTS 107
CORPORATE FINANCIAL

INTRODUCTORY CASELET

MODERN COFFEE HOUSE

Ashok and Ramesh of Modern Coffee


House desired to have a nanon
wide presence of the coffee house. They
knew they will need large
financial resources to do so and that the presentthat
form of their business
organization, 1.e., partnership, was not suitable for carrying out large-
scale business. They could not garner money from public at
their business and at the same time they had unlimited liability large
To
for busi
ness debts.
They decided to convert their partnership firm into a limited company.
They would have access to public money,
their liability for business debts
would be limited and they could freely transfer their shares. However
the company will have to comply with strict regulatory requirements
relating to governance and financial reporting.

QUESTION
1. What is the implication of unlimited liability for partners in a
partnership firm'? (Hint: Personal assets of partners can also be
used to satisfy the liabilities of the business.)
LEARNING OBJECTIVESs

After reading this chapter, you will be able to are required to keep and
Explain the books of account companies
required to prepare.
the financial statements they are
corporate financial statements,
Bxplain the form and contents of
Prepare corporate financial statementS.

7.1 INTRODUCTION which financial stato


in the manner in entities,
There is no fundamental difference and non-company Such as sola
ments are prepared by companies follow the same basie principles. Soma
ne
Owners and partnerships. All entities are as follows:
Special features of company financial
statements
Companies Act and
requirements of the statements.
1. Companies have to follow the
their financial
other applicable laws in preparing use by interester
2. Financial statements of
companies are published for
parties; these are public documents. figures of the
carry comparative
3. Financial statements of companies
previous accounting period.
provisions relating to the books of
This chapter explains the statutory and contents of financial
companies; form
accounts to be maintained by statement of profit and loss); and year-end
statements (balance sheet and which are specifically applicable to
accounting adjustment entries, some ofprovided to facilitate understanding,
companies. Many solved problems are Chapter 9.
The statement of cash flow is covered
in

7.2 B0OKS ACCOUNTS TO BE KEPT


OF
BY A COMPANY
every company to prepare
Section 128(1) of the Companies Act, 2013 requires
office books of account and other relevant books
and keep at its registered year, which give a
and papers and financial statements for every financial including that of its
true and fair view of the state of affairs of the company,
branch office or offices, if any, and explain the transactions effected both at
office and its branches. Such books shall be kept on an accrual
the registered
basis and according to the double entry system of accounting.
According to Section 2(13) of the Companies Act, 2013, "books of account"
include records maintained in respect of:
1. all sums of money received and expended by a company and matters
in relation to which the receipts and expenditure take place
2. all sales and purchases of goods and services by the company;
3. assets and liabilities of the company; and

NMIMS Global Access School for Continuing Education


4. the items of cost as may
be prescribed se o
a company which
belongs to any classunder Section 148
of companies spec under
that section.

7.3 FINANCIAL
STATEMENTs
Section 129 of the Companies
Act,
neeting of a company, the Board of2013 requires that at every annual gene ral
Directors of the company shall lay re
uch meeting the financial statements for the financial year
According to Ind AS 1 Presentation of Financial Statements, a complete seu
inancial statements comprises:
1. a balance sheet as on the end of the period;
2. a statement of profit and loss for the period
3. statement of changes in equity for the period;
4. a statement of cash flows for the period;
5. note, comprising a summary of significant accounting policies and
other explanatory information;
6. comparative information in respect of the preceding period; and
7. a balance sheet as on the beginning of the preceding period if the
company has applied an accounting policy retrospectively, or made a
retrospective restatement of items in its financial statements, or has
reclassified items in its financial statements.
Section 129 of the Companies Act, 2013 further requires that the finaneial
statements shall give a true and fair view of the state of affairs of the com
pany or companies, comply with the accounting standards notified under
Section 133 and shall be in the form or forms as may be provided for different
lass or classes of companies in Schedule IIl.
QUICK TIP
7.3.1 CONSOLIDATED FINANCIAL STATEMENTS
Where a company has one or more subsidiaries, it shall also prepare a consol- Consolidated financial
idated financial statement of the company and of all the subsidiaries. Where statements are required to
a company is required to prepare Consolidated Financial Statements, that is, be prepared by a company if
consolidated balance sheet, consolidated statement of changes in equity and it has one or more subsidiary
same
consolidated statement of profit and loss, the company shall follow the
prepara-
companies.
the
requirements of Schedule III as are applicable to a company in of profit
tion of balance sheet, statement of changes in equity and statement
and loss.

NMIMS Global Access- School for Continuing Education


o Financia
formof
Schedule III of the Companies Act, 2013 which provides the
Statements is given as follows:
Schedule III
Part I-Balance Sheet

Name of the Company


Balance Sheet as on
(Rupeesin
Figures as on
Figures as on the end of the previor
end of the current reporting period
Particulars Note No. reporting period 4
2 3

Assets
1. Non-current assets
(a) Property, plant and equipment
(b) Capital work-in-progress
(c) Investment property
(d) Goodwill
(e) Other intangible assets
() Intangible assets under development
(g) Financial assets
) Investments
(ii) Trade receivables
(iii) Loans
iv) Others (to be specified)
(h) Deferred tax assets (net)
(i) Other non-current assets
2. Current assets
(a) Inventories
(b) Financial assets
i) Investments
(ii) Trade receivables
(ii) Cash and cash equivalents
(iv) Bank balances other than (iii) abov
(v) Loans
(vi) Others (to be specified)
(c) Current tax assets
(net)
(d) Other eurrent assets
Total Assets
Equity and Liabilities

Equity
(a) Equity share capital
(b) Other equity
Liabilities
1. Non-current liabilities
(a) Financial liabilities
(i) Borrowings
ii) Trade payables
in item (D,
(ii) Other financial liabilities (other than those specified
to be specified)
(b) Provisions
(c) Deferred tax liabilities (net)

(d) Other non-current liabilities


2. Current liabilities
(a) Financial liabilities
(i) Borrowings
(ii) Trade payables
specified in
financial liabilities (other than those
(iii) Other
item (c))
(b) Other current liabilities
(c) Provisions
liabilities (net)
(d) Current tax
Liabilities
Total Equity and

March 31, 2017 is


sheet of Asian Paints as on
or reference, the balance
7.1.
oresented in Exhibit

DEducation
School for Continuing
NMIMS Global Access
EXHIBIT 7.1 Balance sheet of Asian Paints Ltd. as on March 31, 2017
(Rupees in Millions
As on As on
Notes 31.03.2017 31.03.2016
Assets
Non-Current Assets
Property, plant and
equipment 2 25,120.1 25,329.7
Capital work-in-progress 2,197.6 927.9
Goodwill 3A 353.6 353.6
Other intangible assets 3B 573.1 606.6
Financial Assets
Investments 14,545.5 13,196.4
Loans 5 702.7 610.7
Others Financial Assets 6 1980.5 305.4
Current tax assets (Net) 7 364.8 151.5
Other non-current 8 2003.9 350.1
assets
47,841.8 41,831.9
Current Assets
Inventories 9 21,940.9 1,610.12
Financial assets
Investments 4 13,154.0 1,477.00
Trade receivables 10 9,946.3 759.06
Cash and cash 11A 613.4 76.75
equivalents
Other balances with 11B 1,439.3 84.03
banks
Loans 5 135.5 9.65
Other financial assets
6 4,744.3
Assets classified as held 306.27
for sale
12
5.7 0.96
Other current assets
8
2319.4 217.92
Total Assets 54,298.8 45.417.6
102.140.6 87.249.5
Equity and Liabilities
Equity
Equity share capital
13
Other equity 959.2
14 959.2
68.550.6
58.298.1
69.509.8
59,257.3
(Continued)

NMIMS Global
Access - School
for Continuing
Eluentin
As on As on
Notes 31.03.2017
Liabilities 31.03.2016
Non-Current Liabilities
Financial liabilities
Borrowings
15 292.7
Other financial 103.8
16 16.8
liabilities 23.1
Provisions 17 1,0984 942.3
Deferred tax liabilities 18C
(Net) 2,611.7 2,171.7

Other non-current 19
liabilities 36.5 18.2

3.873.5 3441.7
Current Liabilities
Financial liabilities
Borrowings 15 268.3
Trade payables
Due to micro and
small enterprises 20 265.9 179.5
Due to others 0 16,446.7 13,152.0
Other financial 16 8,798.0 8,234.7
liabilities
Other current liabilities 19 2,063.2 1,982.3
Provisions 17 362.00 363.5
Current tax liabilities 21 553.2 638.00
Net)
28,757.3 24.550.5
Total Equity and 102.140.6 87.249.5
Liabilities

(Source: Asian Paints Annual Report 2016-17)

7.3.2STATEMENT OF CHANGES IN EQUITY


The statement of changes in equity is prepared in two parts. Part A depicts
the changes in equity share capital and part B depicts changes in other equity.

Statement of Changes in Equity

Name of the Company


Statement of changes in Equity for the period ended
(Rupees in.

NMIMS Global Access- School for Continuing Education


A. Equity Share Capital
Balance at the Changes in Equity
Balance at the end of
Beginning of the Share Capital during the Reporting Period
Reporting Period the Year

Limited ie
he statement of changes in equity share capital of Asian Paints
presented in Exhibit 7.2.

Asian Paints Ltd. for


EXHIBIT 7.2 Statement of Changes in Equity Share Capital of
the Year ended 31 March, 2017
in (Rupees Millions)
As on 31.03.2016
Equity Share Capital As on 31.03.2017
959.2 959.2
Balance at the beginning of the
reporting year
Changes in equity share capital
during the year
959.2 959.2
Balanceat the end of the
reporting year

informa-
Part B of the statement of changes in equity requires the following
tion to be presented.
following
Each component of the balance sheet is discussed in detail in the
sections.

7.4 ASSETS
QUICK TIP Assets are economic resources controlled by an entity whose cost (or fair
An economic resource is a value) at the time of acquisition could be objectively measured. A resource is
resource that provides future an economic resource if it provides future cash flows to the entity. An asset
cash flows to the entity. can be: (i) cash or something convertible into cash (e.g. accounts receivable),
(i) goods expected to be sold and cash received from them and (iii) items to
be used in future activities that will generate cash flows.
A basic classification of assets is between current assets and non-current assets.
An asset is classified as current when it satisfies any of the following criteria:
1. It is expected to be realized in, or is intended for sale
or consumption
in, the company's normal operating cycle. An operating cycle is the time
between the acquisition of assets for processing and their realization in
cash or cash equivalents. Where the normal operating cycle cannot
be
identified, it is assumed to have duration of 12
months.
2. It is held primarily for the purpose of
being traded.

NMIMS Global Access School for


Continuing Education
sh or cashBe realized within
or used
ed to settle equivalent
a valent unless 12 months after
1l other assets liability it is restricted the reporting date.
for atleast 12 restricte
are classified mont frombeing excn
as non-current. 1onths after the reportingnged
4.1 NON-CURRENT ate.
on-current ASSETS
assets are
further classified
1. Property,
plant and as:
equipment
2. Capital work-in-progress
3. Investment
property
4. Goodwill
5. Other intangible
assets
6. Financial
assets
(a) Investments
(b) Trade receivables
(c) Loans
(d) Others (to be specified)
7. Deferred tax assets
(net)
8. Other non-current assets

PROPERTY, PLANT AND EQUIPMENT


Ind AS 16 defines property, plant and equipment as tangible items that
(i) are held for use in the production or supply of goods or services, for rental
to others, or for administrative purposes and (ii) are expected to be used
during more than one period.
property,
The following classification is required to be given in respect of
plant and equipment
1. Land
2. Buildings
3. Plant and equipment
4. Furniture and fixtures
5. Vehicles
6. Office equipment
nature) lease by a com-
7. Others (specify that is held under
equipment to show a
property, plant and Companies are also required
Any item of separately specilied. amounts of each class of assets at the
pany needs to be net carrying additions, disposals, acquisi-
gross and showing
reconciliation of the reporting period other adjustments.
Depreciation and
of the separately
beginning and end combinations and assets are to be disclosed
business to these
tions throughlosses/reversals related reduction of its
capital or
on a have
impairment
value of assets assets, the company will
reduces the revaluation of
t a companyvalue of assets on Continuing Education
increases the Access- School for
DIMIMS
Global
amount and date of the
to show the reduced or increased figure and also the
first 3 years subsequent
reduction or increase in every balance sheet for the
to the date of such reduction or increase.

CAPITAL wORK-IN-PROGRESS
a
Capital work-in-progress is the amount
invested in construcun8 tangible use
yet complete and ready for its intended
non-current asset that not
is this heac
paid as advance to suppliers of such assets also fall under
Amounts

INVESTMENT PROPERTY
long-term
held by a company for
nvestment property is that property that is is not OCcupied by
or both and that
rental income or capital appreciation ot the gross and
to show a reconciliation
the group. Companies are required
property at the beginning and end of
net carrying amounts of each class of disposals, acquisitions through busi.
the reporting period showing additions, and the related depreciation and
ness combinations, and other adjustmentsdisclosed separately.
impairment losses or reversals shall be

GOODWILL
reconciliation of the gross and net
Companies are also required to show a
beginning and end of the reporting period
carrying amount of goodwill at the
disposals and other adjustments.
showing additions, impairments,

OTHER INTANGIBLE ASSETS


the following classification:
Other intangible assets are shown under
1. Brands/trademar
2. Computer software
3. Mastheads and publishing titles
4. Mining rights
5. Copyrights, patents and other intellectual
property rights, services and
operating rights
6. Recipes, formulae, models, designs and
prototypes
7. Licenses and franchise
8. Others (specifying nature)
to
In a manner similar to that for tangible assets, companies are required
show reconciliation of gross and net carrying amount at the beginning and
end of reporting period for each class of other intangible assets. The reconcil-
iation should show additions, disposals, acquisitions through business com-
binations and other adjustments. The related amortization and impairment
losses or reversals need to be disclosed separately
Provisions relating to disclosure of reduction or increase pursuant to reduce-
tion of capital or revaluation of assets are similar to those for tangible assets.

FINANCIAL ASSETS
According to Ind AS 32 Financial Instruments: Presentation, a financial asset
is any asset that is:
1. Cash or,
2. An equity instrument of
another entity or,
3. A contractual right: (i) to
receive cash or financial asset iro
another entity; or (ii) to exchange financialanother financial liabilities
assets or
with another entity under conditions that
are potentially favorable to
the entity.
inancial assets are further classified as:
1. Investments

2. Trade receivables
3. Loans
4. Others (to be specified)
VESTMENTS

vestments are amounts of money invested outside the business in stocks, other
invest
curities, tirms, subsidiary companies and other assets. Non-current
ents are long term in nature and are not expected to be sold within a year

hese are required to be classified as:


1. Investments in Equity Instruments;
2. Investments in Preference Shares;
3. Investments in Government or trust securities;
4. Investments in debentures or bonds;
5. Investments in Mutual Funds;
6. Investments in partnership
firms; or
7. Other investments (specify nature).
RADE RECEIVABLES
amount due
receivable if it is in respect of the business.
receivable is classified as a trade normal course of
account of goods sold or services rendered in the
n
further classified as:
rade receivables are required to be
1. Secured, considered
good;
good; and
2. Unsecured considered
3. Doubtful.
OANS directors or out-
its officers,
by a company to
0ans include loans extended
ide parties.
classified as:
be further
0ans are required to
1. Security deposits, thereof), and
parties (giving details
2. Loans to related
loans (specifying their nature).
3. Other

School for Continuing Edueation


NMIMS Global Access
as:
r Sub-classification of the above types of loans is required
1. Secured, considered good,
2. Unsecured, considered good, and
3. Doubtful.

OTHER FINANCIAL ASSETS

Some examples of other financial assets are


*Bank deposits with more than 12 months of original
maturity

2 Subsidy receivable from government


.Term deposits kept as margin money by the bank against guarantees
given by the bank
Cash and bank balances not available for immediate use, for example.
Dalance in the account representing unpaid dividend
DEFERRED TAX ASSETS
Deferred tax assets and deferred tax liabilities are netted off against each other
An amount against deferred tax assets will appear in the balance sheet when
he amount of deferred tax assets exceed the amount of deferred tax liabilities,

OTHER NON-CURRENT ASSETS


Other non-current assets are required to be classified as:
1.Capital advances and
2. Advances other than capital advances.
Capital advances are moneys given as advance for procurement of non-
current assets and the company does not expect to realize these in the next
12 months or within the normal operating cycle.

Advances other than capital advances should be further classified as:


1. Security deposits,
2. Advances to related parties (giving details thereof) and
3. Other advances (specifying nature).
Examples of advances other than capital advances include security deposits
with port, customs and other statutory authorities.
NOTE
7.4.2 CURRENT ASSETS
Any asset that does not meet
the requirements of the Current assets are liquid assets of the company that are held either in the
definition of current asset
is form of cash or can be easily converted into cash within one accounting
treated as non-current asset. period, usually a year. Schedule II of the Companies Act, 2013 classifies
current assets in the following manner:
1. Inventories
2. Financial Assets
(a) Investments
(b) Trade receivables

NMIMS Global Access School for Continuing Education


-
Loans other than
(Others (to (c) above
3. Current be specified)
tax assets
4 Other current (net)
assets
INVENTORIES
Tnventories IMPORTANT CONCEPT
Drocess of
are
assets
production held
to be consumed for
for suchsale in the ordinary
Bank deposits having a
maturity up to 12 months
Inventories in the sale; or in
production course of business; are treated as current asset
merchandiseencompasS goods the
process or form of materials in the
or supplies and those with a maturity
purchased
held for resale, purchased in the rendering of more than 12 months are
encompass or land by a retailer and and held for resale; of services. treated as non-current asset.
and
finished goods other property held for resale, for example,
by the enterprise held computer software
produced,
and loose tools and imclude materials,or work infor resale. Inventories
progress als0
awaiting use in maintenance supplies,being produced
The schedule of the production process. consumables
inventories
for the year 2016-17 appearing in the
is reproduced annual report of Asian
in Exhibit 7.3. Paints
Inventories of Asian
Paints as on March 31,
2017
EXHIBIT 7.3
Inventories Rs. in Millions
(at
lower of cost and net As on
realizable As on
value) March 31, March 31,
(a) Raw materials 2017 2016
Raw materials-in-transit 5,167.8 4,551.1
811.6 792.3
(b) Packing 5.979.4 5.343.4
materials 363.2
(c)Work-in-progress 401.3
748.0 664.7
(d) Finished goods
12,315.0 7,759.9
Finished goods-in-transit 18.0 24.2
12.333.0 1.784.1
(e) Stock-in-trade (acquired for trading) 1,824.1 1,345.2
Stock-in-trade (acquired for trading) 29.7 5.0
1.853.8 1.350.2
(t) Stores, spares and consumables 661.0 577.5
Stores, spares and consumables in 2.5
transit
663.5 577.5
Total 21.940.9 16.101.2
INVESTMENTS
Current investments vertible
aresshort-term securities that are easily conve into
Cash. Companies usually
invest liquid assets in excess ot transaction cadd
eeds in such securities to generate better returns. In a company's balane ash
Sheet, current investments are required to be classified as: e
. Investments
in equity instruments
2 Investment in preference shares
Investments in government or trust securities
4. Investments in debentures or bonds
5. Investments in mutual funds
6. Investments in partnership
firms
1, Other investments (specify nature)

TRADE RECEIVABLES

Trade receivables are required to be further classified as:


1. Secured, considered good;
2. Unsecured considered good; and
3. Doubtful.

CASH AND CASH EQUIVALENTs

Cash and cash equivalents include cash in hand, cheques and drafts in hand
pending their deposit in the bank account, and balance in bank accounts
(current accounts and time deposit accounts).

LOANS

Loans are required to be further classified as:


1. Security deposits;
2. Loans to related parties (giving details thereof) and
3. Other loans (specifying their nature).

Further sub-classification of the above types of loans is required as:


1. Secured, considered good;
2. Unsecured, considered good; and
3. Doubtful.

oTHER CURRENT ASSETS


This is an all-inclusive heading, which incorporates current assets that do not
fit into any other asset categories. Other current assets are to be clasifedas

1. Advances to suppliers
2. Advances to employees

3. Export benefits receivable


4. Prepaid expenses
e classification
State the
Schedule of financial
III of the Companies assets
Act, 2013. to be prov
ovided according to 3ACTIvITY 1
7.5 EQUITY
Dauity generally
(shareholders). refers to the amount invested
assets of an 'These are also in an enterprise by
enterprise. used the owners
of creditors and The claims of to refer to the claim of owners
lenders. These owners to the
get only what is leit are also calledto assels are secondary to Uhos
in equity occur after all obligations the resicdual claims as owners
when (i) new to outsiders
shares are bought back anmd shares are issued by have been paid. Changes
operations or incurs (ii) the business the company or existing
is divided into losses from earns income
two parts: equity unprofitable operations of from profitable
share capital and business. Equity
other equity.
7.5.1 EQUITY SHARE
CAPITAL
The capital of a company
is
can have two classes of divided into small units called shares. QUICKTIP
share capital. Equity sharesshare capital, equity Companies
or share capital and preference
shares. These shares have ordinary shares are the Bonus shares are shares
basic types
to receive dividend and a
the usual rights of ownership: right to of equity that are allotted to existing
residual claim on the assets vote, right shareholders without any
case of liquidation. of the company in the consideration being received
in cash.
7.5.2 PREFERENCE SHARES
Preference shares have a preference
payment of dividend and over equity shares in the matter of
repayment of capital at the time of liquidation.
Preference shares are entitled to a fixed IMPORTANT CONCEPT
not carry any voting rights. The rate of dividend and normally do
holders of preference shares do not have a company generally decides
guarantee that they will receive the fixed A
dividend. It is only when the board to buy back its shares when
of directors declare a dividend that the
company has an obligation to pay its management feels that the
dividend to its shareholders. In such a case, dividend
is first paid to holders shares are undervalued and
of preference shares.
the company has surplus cash.

AUTHORIZED SHARE CAPITAL


Authorized share capital is the value of shares that the company is authorized
to issue by its Memorandum of Association. The companies are governed
by the provisions of their constitution which is known as Memorandum of
Association. Authorized capital is the maximum capital which a company can
issue without altering the capital clause of the Memorandum of Association
for an increase in its authorized capital.

ISSUED, SUBSCRIBED AND PAID-UP CAPITAL


Issued capital is the nominal or face value of shares which are offered by the NOTE
company to the public for subscription. It cannot be more than the autho- Shares are said to be issued
rized capital. Subscribed capital is the nominal value of shares taken up by the at a premium when these are
public. Subscribed capital is equal to issued capital if all the shares
offered to
subscribed issued at a price higher than
part of the capital that
the public are taken up by the public. That capital. Paid-up capital their par value.
is called called-up
has been called up by the company

NMIMS Global Access School for Continuing


Education
ACTrvITY 1
State the classification of financial assets to
Schedule III of the Companies be provided according to
Act, 2013.

7.5 EQUITY
Equity generally relers to the amount invested in an enterprise by the owner
(shareholders). These are also used to refer to the
assets of an enterprise. The claims of claim of owners to tn
owners to assets are secondary to Uno
of creditors and lenders. These are also called the residual claims as owners
get only what is left after all obligations to outsiders have been paid. Changes
in equity occur when (i) new shares are issued by
the company or existing
shares are bought back and (i) the business earns income from profitable
Operations or incurs losses from unprofitable operations of business. Equity
is divided into two parts: equity share capital and other equity.

7.5.1 EQUITY SHARE CAPITAL QUICK TIP

The capital of a company is divided into small units called shares. Companies shares
Bonus shares are
can have two classes of share capital, equity share capital and preference existing
share capital. Equity shares or ordinary shares are the basic types of equity that are allotted to
shareholders without any
shares. These shares have the usual rights of ownership: right to vote, right consideration being received
to receive dividend and a residual claim on the assets of the company in the
in cash.
case of liquidation.

7.5.2 PREFERENCE SHARES


Preference shares have a preference over equity shares in the matter ot IMPORTANT CONCEPT
payment of dividend and repayment of capital at the time of liquidation.
Preference shares are entitled to a fixed rate of dividend and normally do A company generally decides
not carry any voting rights. The holders of preference shares do not have a to buy back its shares when
guarantee that they will receive the fixed dividend. It is only when the board its management feels that the
of directors declare a dividend that the company has an obligation to pay shares are undervalued and
dividend to its shareholders. In such a case, dividend is first paid to holders the company has surplus cash.
of preference shares.

AUTHORIZED SHARE CAPITAL


is authorized
Authorized share capital is the value of shares that the company
companies are governed
to issue by its Memorandum of Association. The
is known as Memorandum of
by the provisions of their constitution which
which a company can
Association. Authorized capital is the maximum capital
clause of the Memorandum of Association
issue without altering the capital
for an increase in its authorized capital.

ISSUED, SUBSCRIBED AND PAID-UP


CAPITAL

Issued capital is the nominal or face value


of shares which are offered by the
cannot be more than the autho-
- -NOTE
company to the public for subscription. It |
Shares are said to be issued
value of shares taken up by the
rized capital. Subscribed capital is the nominal at a premium when these are
capital if all the shares offered to
public. Subscribed capital is equal to issued issued at a price higher than
part of the subscribed capital that
the public are taken up by the public. That their par value.
is called called-up capital. Paid-up capital
has been called up by the company

Continuing Education
NMIMS Global Access School for
been paid up by the sharehol.old-
s that part of the called-up capital which has paid-up capital is equal to
ers. If all the called-up capital has been
received,
ha
called-up capital that has not yet een
ne called-up capital. That part of the unpaa.
received is called calls in arrear or calls

PAR VALUE
value) represents the legal capital
Par value (also called face value or nominal par valuo
the
per share. The shareholders' funds cannot be reduced below
company or by special legal
O1 share capital except by losses suffered by the
that will reduce th
action. The company is not allowed to declare a dividend compa.
Shareholders' funds below the par value of share capital. Majority of
nies in India have par value of Rs. 10 per share. Others have Ks. or Rs. 2 or
5

Rs. as par value of a share.


1

Illustration 7.1

Mars Limited issues 5,000 equity shares of Rs. 10 each at Rs. 20. The
whole amount of Rs. 20 is payable at the time of application. The trans-
action will be recorded in the following manner:
(Rs.)
Bank (Dr.) 100,000
To equity share capital 50,000
To securities premium 50,000

ISSUE OF SHARES FOR CONSIDERATION OTHER THAN CASH


A company may issue shares for consideration other than cash. This may
happen when a company pays for purchase of fixed assets in the form of
shares or when a company acquires another business and pays either the
full or part of purchase consideration by way of its own shares. When a com-
pany issues shares for cash, it debits cash account and credits share capi-
tal account. When a company pays for assets by way of shares, it debits the
vendor or the seller and credits share capital account.

Illustration 7.2

Mars Limited buys a piece of equipment priced at Rs. 60,000.


agrees to accept 300 shares with a par value of Rs. 10 The vendor
each as consideration
for the sale of equipment. These transactions will
be recorded as follows:
(Rs.)
Equipment (Dr.) 60,000
To Vendor
60,000
Vendor (Dr) 60,000
To equity share capital
30,000
To securities premium
30,000

NMIMS Global Access School for Continuing Education


SSUE OF BONUS SHAREs
Bonus shares are the shares allotted
ration being received to existing equity sh
any considera in cash or in kind. shareholders without
IS shares, it converts its accumulated
bonu When a commpany issues
share capital. profits and other reserves into

1, A company issues 300,000 SELFASSESSMENT


equity shares of Rs. 10 face
per share. The company will value at Rs. 15 QUESTION
show in its balance sheet
a. Equity share capital of Rs.
3,000,000
b. Equity share capital of Rs. 4,500,000
c. Equity share capital of Rs.
3,000,000 and securities premium o
Rs. 1,500,000
d. Equity share capital
of Rs. 3,000,000 and retained earnings of
Rs. 1,500,000.

7.6 OTHER EQUITY


Other equity comprises of:
1. Share application money pending allotment
2. Capital reserve
3 Securities premium reserve
4. Retained earnings
5. Revaluation surplus
Companies need to disclose, under each of these heads, additions and
deductions since last balance sheet.

7.6.1 SHARE APPLICATION MONEY PENDING ALLOTMENT


When a company issues its equity shares to the public to raise funds, it receives
the application money from the applicants. However, the equity shares are
allotted to the applicants on a later date. It is only on allotment of shares, that
the application money gets transferred to the equity share capital.

7.6.2 CAPITAL RESERVE


It is a reserve created as a result of capital profits. Capital profits are prof-
its other than those earned from normal business operations. These include
profit on sale of fixed assets, profit prior to incorporation, premium on issue
of shares, premium on issue of debentures, profit on purchase of business.
Capital reserve is not available for distribution of dividend. However, it may
De used to issue bonus shares and setting off of capital losses.

7.6.3 SECURITIES PREMIUM RESERVE


when shares are issued at a price higher than the par value, it is called an
sue of shares at a premium. Excess of issue price over face value is called
une securities premium. It is a capital profit for the company and this profit
has to be credited to a separate account called the securities preminum reserve.

NMIMS Global Access


- School for Continuing Education
7.6.4 RETAINED EARNINGS

Apart of the profit earned by a company may be distributed as dividend. Tho


gets accumulated year
remaining profit is retained within the business and balance Sheet, The bal.
au Ler year and appears as retained earnings in the
uses the amount
ance in retained earnings gets depleted when the company
redemption of preterence shares or
1n this account to issue bonus shares, for
for redemption of debentures.

7.6.5 REVALUATION SURPLUS


property, plant and equipment ara
Revaluation surplus arises when items of A credit 15 given to the
valued at their fair value instead of their book value.
property, plant and equipment
revaluation surplus when the value of an item of is the difference between
s Written up on revaluation. The amount ofofcredit
property, plant and equipment
the fair value and the book value of the item
A company cannot distribute its revaluation
surplus as dividend to shareholders

7.7 LIABILITIES
financal resources from
Liabilities are claims to assets. A business raisesclaims to the assets of the
both its owners and outside parties. Both
have
parties other than owners. Liabilities
entity. Liabilities are claims to assets of
are classified as current liabilities and
non-current liabilities. According as
liability shall be classified
to Schedule III of the Companies Act, 2013, a
current when it satisfies any of the following criteria:
normal operating cycle,
1. It is expected to be settled in the company's
acquisition of assets for
An operating cycle is the time between the equivalents. Where the
processing and their realization in cash or cash is assumed to have a
normal operating cycle cannot be identified,
it
duration of 12 months.
2. It is held primarily for the purpose of being traded.
3. It is due to be settled within 12 months after the reporting
date.
4. The company does not have an unconditional right to defer
settlement
of the liability for at least 12 months after the reporting date. Terms
of a
liability that could, at the option of the counterpart, result in its settlement
by the issue of equity instruments do not affect its classification.
Liabilities other than those classified as current are classified as non-current
liabilities.

7.7.1 NON-CURRENT LIABILITIES


Non-current liabilities are long-term in nature. These usually arise from
major expenditures such as acquisition of non-current assets or acquisition
of another business. Large amounts are generally involved in relatively few
transactions related to non-current liabilities. Schedule III further classifies
current liabilities in the following manner:
1. Financial liabilities
(a) Borrowings
(b) Trade payables

NMIMS Global Access School for Continuing Education


(o)Other financial liabilities (other
be specified) than those specified in
item (b), to
Provisions
2.

3.
Deferred tax liabilities (net)
De
4. Other non-current liabilities
BORROWINGS

cowings having a maturity of more


Borro than 1 year are considered as non
nt liability. These may take one of the
following forms:
1. Bonds or debentures
2. Term loans
(a) From banks
(b) From other parties
3. Deposits
4. Loans from related parties
5. Other loans and advances (specify nature)
BONDS OR DEBENTURES
When a large loan is split into small transferable units, these are called bonds
or debentures. These are interest-bearing instruments which are generally
sold to the investing public but may also be sometimes placed privately with
financial institutions. A major advantage of debentures is that these are flex
ible instruments that offer a wider choice to issuers with regard to maturity,
security, interest rates and other features.

TERM LOANS

Term loans are a form of long-term debt finance provided to companies by


banks and financial institutions for setting up new projects or for expansion
and modernization. These are generally secured by the assets that are financed
by such loans. These loans can be in domestic currency or in foreign curren-
Cies. Interest on term loans is payable at quarterly or half-yearly intervals.

DEPOSITs
Companies are allowed to accept deposits from public as well as its employees.
to 3 years. Deposits
These deposits can have a maturity ranging from months
6

with a maturity of morethan one year are considered as long-term borrowings.


These deposits are unsecured. Deposits are a convenient
source of finance for
Companies as these are unsecured and do not carry
any restrictive covenants.
through deposits.
lowever, companies can raise only limited amounts of funds
TRADE PAYABLES
of the amount due
Apayable is classified as a trade payable if it is in respect
in the normal course of
on
account of goods purchased or services received
business.
OTHER FINANCIAL
LIABILITIES
Some examples of non-current other financial
liabilities are:
1. Retention money
relating to capital expenditure
2. Deposits from
contractors or dealers
3. Amounts payable under derivative contracts

PROVISIONS

A provision is an amount set aside from a company's proits to meet an


expected liability or for the decrease in the value of an asset, but the amount
of the liability is uncertain and requires estimation. Important liabilities for
which provisions are required are employee pensions and product warran
ties. Schedule II of the Companies Act, 2013 requires provisions to be clas
sified as provision for employee benefits and other provisions, specifying
their nature.

DEFERRED TAX LIABILITIES


These arise due to difference between the profit as per statement of profit
and loss and the taxable income calculated under the Income Tax Act.
A deferred tax liability is created when the tax on accounting income is more
than the tax payable under the Income Tax Act. This happens when account
ing income is more than taxable income. A deferred tax asset is created when
the tax on accounting income is less than the tax payable under the Income Tax
Act. This happens when accounting income is less than the taxable income.

Deferred tax assets and deferred tax liabilities are netted off against each other.
An amount against deferred tax liabilities will appear in the balance sheet when
the amount of deferred tax liabilities exceeds the amount of deferred tax assets.

OTHER NON-CURRENT LIABILITIES

Other long-term liabilities include liabilities other than those mentioned


under specific heads. These are further classified as:
1. Advances
2. Others
Examples of other non-current liabilities are:
1. Deferred income arising firom government grants
2. Annuity payable to VRS optees
3. Advances from customers
4. Sales tax deferment loan from state government

7.7.2 CURRENT LIABILITIES


We have already defined current liabilities as those liabilities which sa
isfy the requirements specified by Schedule II of the Companies Act
2013. Schedule III further classifies current liabilities in the following8
manner:
Financial Liabilities
1.
(a)
Borrowings
(b) Trade
payables
(c)Other financial liabilities (other than those specified in item (C)

2. Other current iabilities


Provisions
3.
4. Current Tax Liabilities (Net)
BORROWINGS

ewowings shown under the head 'current liabilities' represent short-term


Bor
borrowings. These are to be classified as:
Loans repayable on
demand
1.

(a) from banks


(b) from other parties
2. Loans and advances from related parties
3. Deposits
4, Other loans and advances (specify nature)

OTHER FINANCIAL LIABILITIES


Other current financial liabilitiesare required to be classified as:
1. Current maturities of long-term debt;
2. Interest accrued;
3. Unpaid dividends;
4. Unpaid matured deposits and interest accrued thereon;
and
5. Unpaid matured debentures and interest accrued thereon
6. Others (specify nature).

OTHER CURRENT LIABILITIES


These are classified as:
(a) revenue received in advance;
6) other advances (specify nature) and
(c)others (specify nature)
REVENUE RECEIVED IN ADVANCE
to be sup-
ometimes, a company receives an advance payment for goods
plied or for services to be rendered in future. For example, subscriptions may
magazine or insurance premia may
De received
in advance by publishers of a
company. Such receipts cannot be
received in advance by an insurance supplied or the services
eated as revenue until the related goods have been
Ve been rendered. Till such time, these receipts are treated as current
liabilities.
PROVISIONS
Like long-term provisions, short-term provisions are classified as provision
for employee benefits and others, specifying the nature of other provisions.

SELF-ASSESSMENT 2. Which of the following would not be classified as a long-term liability?


QUESTIONS
a. Current maturities of long-term debt
b. Debentures
C. Bonds
d. Finance lease obligations
3. Which of the following will be classified as a current liability?
a. Debentures
b. Accounts payable
c. Finance lease obligations
d. Bonds

7.8 CONTINGENT LIABILITIES


AND COMMITMENTS
A contingent liability is: (a) a possible obligation that arises from past events and
QUICKTIP whose existence will be confirmed only by the occurrence or non-occurrence
of one or more uncertain future events not wholly within the control of the
hgent liability is not entity; or (b) a present obligation that arises from past events but is not rec-
liability but a potential
ognized because: (1) it is not probable that an outflow of resources embodying
ability that depends on an
uncertain future event. economic benefits will be required to settle the obligation; or i) the amount
of the obligation cannot be measured with sufficient reliability.
A contingent liability is not a real liability but a potential liability that depends
on an uncertain future event. A company is not required to recognize a con-
tingent liability. It needs to just disclose the contingent liability in notes to
accounts. If the possibility of an outflow of resources embodying economic
benefits is remote, no disclosure is required.

7.8.1 STATEMENT OF PROFIT AND LOSS


The Statement of Profit and Loss includes:
1. Profit or loss for the period
and
2. Other Comprehensive Income
for the period.
The sum of (1) and (2) above is "Total
Comprehensive Income'
The form of Statement of Profit and
Loss is provided by Part II of Schedule
III to the Companies Act, 2013. The
and expenditure account, in like
provisions of Part II apply to the income
Profit and Loss. manner as they apply to the Statement o

NMIMS Global Accest


art II Form Statement of Profit
Par -

and L0ss

Name of the Company


Statement of Profit and Loss for
the period ended-
in- (Rupees

Figures for Figures for


the current the previous
Particulars Note reporting reportingg
No. period
I Revenue from operations period
XXX
II Other income XXX
III Total revenue (I + II)
XXX
IV Expenses: XXX
Cost of materials consumed
Purchases of stock-in-trade
Changes in inventories of
finished goods, work-in-
progress and stock-in-trade
Employee benefits expense
Finance costs
Depreciation and amortiza-
tion expense
Other expenses
Total expenses (IV)
V Profit/loss) before exeep- XXX
tional items and tax I-IV)
VI Exceptional items XXX

VII Profit/(loss) before tax XXX

(V-VI
VIII Tax expense: XXX

Current tax
(1)
Deferred tax
(2)
IX Profit (loss) for the period XxX
from continuing opera-
tions (VII- VIII)
Profit (loss) from discon- XXX

tinued operations
XI Tax expense of discontin- XXX

ued operations
XII Profit (loss) from discon- XXX
tinued operations (after
tax) (X-X) (Continued)
XIII Profit (loss) for the period XXx
(IX +XIID
XIV Other Comprehensive KXX
Income
A (i) Items that will not be
reclassified to profit
or loss
(i) Income tax relating
to items that will not
be reclassified to
profit or loss
B (i)
Items that will be
reclassified to profit
or loss
i) Income tax relating
to items that will be
reclassified to profit
or loss
XXX
XV Total Comprehensive In-
come for the period (XIII
+XIV) (Comprising Profit
(Loss) and Other Compre-
hensive Income for the
period)

For reference, the statement of profit and loss of Asian Paints


for the year
ended March 31, 2017 is presented in Exhibit 7.4.

EXHIBIT 7.4
Statement of Profit and Loss of Asian Paints Ltd.
31st March, 2017 for the year ended

(Rupees in Millions)
Year Year
2016-17 2015-16
Revenue from Operations
Revenue from sale of products (includ-
ing excise duty)
22A 141,545.4 131,323.2
Revenue from sale of services
22B 75.9 126.3
Other operating revenues
Other income 22C 1,983.0 1,872.3
Total Income () 23 3,009.0 2,494.3
Expenses 146.613.3 135,816.1
Cost of materials consumed
Purchases of stock-in-trade 24A 67,374.5 58,659.4
24B 6,465.3 5,244.2
(Continued)
NMIMS Global Access-
School for Continuing
Edueation
Year Year
Changes in inventories 2016-17
stock-in-trade and of finished goods, 2015-16
work-in-progress
Excise duty 24C (5,155.8) 1,945.1
Employee benefits expense 17,133.2 15,018.5
Other expenses 25 7,428.3 6,668.3
Total (11) 26 23.644.4 21.017.0
Earning before Interest, 116.889.9 108.552.5
Tax, Deprecia-
tion and Amortization
Finance costs 29,723.4 27.263.6
Depreciation and amortization 27 188.6 234.0
expense 28
Profit before Exceptional 2,954.3 2,345.1
Items and Tax 26,580.5
Exceptional items 24,684.5
45
Profit before Tax 653.5
Tax Expense 26.580.5 24.031.0
18
(1) Current tax
(2) (Excess)/Short 8,172.2 7,437.4
tax provision for
earlier years
(3) Deferred tax (36.0) (33.3)
Total tax expense 413.3 398.8
Profit before Tax 8.549.5 7.802.9
18.031.0 16,228.1
The various components
of the statement of profit and loss
detail in the following are discussed in
sections.

7.9 REVENUE FROM OPERATIONS


Revenue is the gross inflow
of economic benefits during the period
during the ordinary activities of an arising
entity when those inflows result in
in equity, other than
increases relating to contributions from equity increases
participants.
A company has to disclose revenue from
heads in the notes: operations under the following

1. Sale of products (including excise duty or goods and


services tax)
2. Sale of services
3. Other operating revenues
Other operating revenue is the revenue arising from
activities
incidental to the main revenue earning activities of the company,which are
nue from sale of scrap by manufacturing e.g. reve-
a company or revenue from sale
Waste paper or packaging material by a merchandising of
company.
Excise duty (goods and services tax) is an indirect tax payable
to the government. by a company

NMIMS Global Access School for Continuing


Education
7.9.1 REVENUE RECOGNITION

There are two interrelated issues associated with revenue recognition: quan-
tum of revenue and timing of recognition. Revenue is recognized using the
accrual principle.
Revenue firom the rendering of services is recognized by reference to stage of
completion of the transaction at the end of the reporting period. The recognition
OCcurs only when the outcome of the transaction can be estimated reliably.

ACTIvITY2 Name the three main components of revenue from operations for a non-
finance enterprise and for a financial company.

7.10 OTHER INcOME


In addition to income from its regular operating activities, a company may
also generate income from other sources such as income from rent, dividend,
interest, gain or loss on sale of assets or investments. Interest income earned
by a finance company is part of its operating revenue.
Companies are required to report other income classified as:
1. interest Income;
2. dividend Income and
other non-operating income (net of expenses directly attributable
to
3.
such income).

ACTIVITY 3 Name the two main sources of other income.

7.11 EXPENSES
Expenses are matched with revenue to determine the profit or loss made
by a business during an accounting period. An expense is that cost which
relates to the operations of an accounting period (e.g. rent) or to the revenue
earned during the period (cost of goods sold) or the benefits of which do not
extend beyond that period. Expenses, thus, have a relation with the account
ing period and represent that part of the cost of an asset or service that is
consumed during the accounting period. Companies are required to report
expenses under the following heads:
1. Cost of materials consumed
2. Purchases of stock-in-trade
3. Changes in inventories of finished goods, work-in-progress and stoc
in-trade
4 Employee benefits expense
5. Finance costs
6. Depreciation and amortization expense
7. Other expenses
STATEMENTS 135
CORPORATE FINANCIAL

7.11.1 COST OF MATERIALS CONSUMED


loss
mhe purpose of the statement of profit and loss is to caleulate profit or
matching
ade by a business during the accounting period. This is achieved
by
nenses and revenue, The first step in this direction is matching of revenue
expenses
Cost of goods sold represents aggregation of
h cost of goodsto sold.
earning of revenue, i.e. the direct costs of goods that
have
directly related manufactur
cost of materials consumed d other
been sold. This comprises goods
and merchandising cost. Cost of goods sold is calculated as cost of
ing finished goods.
available for sale adjusted for change in the inventory of
= Beginning inventory of
materials +
Cost of material consumed
Material purchased
- Ending inventory of materials

7.11.2PURCHASES OF STOCK-INTRADE
manufacture all the items that they sell. They
Many companies do not finished goods
engage in merchandising transaction and purchase of this head.
also
cost of purchase of such items is reported undergoods sold.
for resale. The second element of the cost
of
Purchase of stock-in-trade is the
GOODS,
OF FINISHED
7.11.3 CHANGES IN INVENTORIES STOCK-INTRADE
WORK-IN-PROGRESS AND dif-
requires adjustment for the cost of
sold accounting
Computation of cost of goods end of the
inventory items at the beginning and inventory
ferent types of usually carry three types of
companies goods.
period. Manufacturing work-in-process and finished
items: inventory of
materials, of mate-
materials is done when the costinventory
for raw represents
The inventory adjustment Work-in-process inventory is a
rial consumed is
calculated. semi-finished items get finished, there the
As these addition to
of semi-finished goods. inventory. This change reflects
work-in-process
change in the
sale.
cost of goods available for materials consumed +
= Cost of
Cost of goods available
for sale cost + Purchase of
Direct manufacturing in inventory of
stock-in-trade + Change
work-in-process

may not be sold. Some part


purchased by a compayup in inventory at the end of
All goods produced and
manufactured and purchased goods may endmay come from the inventorv
of goods sold
A part of the goods available
the accounting period. over from the previous year: Cost of
finished goods to
carried
offinished goods adjusted for change in the inventory of
lor sale need to be
calculate cost of goods sold.
available for sale
sold = Cost of goods
Cost of goods + Change in inventory of
finished goods
and stock-in-trade
7.11.4 EMPLOYEE BENEFITS EXPENSE
Under this head, companies have to show separately expenses incurred on:
o
1. Salaries and wages

2. Contribution to provident and other funds


3. Share based payments to employees
4. Staff welfare expenses

7.11.5 FINANCE COSTS


Finance costs are costs related to the borrowings of the company. These Costs
are to be reported under the following classification:
1. interest;
2. dividend on redeemable preference shares;
3. exchange differences regarded as an adjustment to borrowing costs and
4. other borrowing costs (specifying nature).

7.11.6 DEPRECIATION AND AMORTIZATION EXPENSES


Most of the items of property, plant and equipment have limited useful life.
The cost of an itenm of property, plant and equipment needs to be appropri
ated on a systematic basis over its useful life. This process of appropriation is
called depreciation in relation to tangible assets and amortization in relation
to intangible assets. The appropriation is based upon the matching principle.

SACTIVITY 4 Explain the difference between depreciation and amortization.

7.11.7 OTHER EXPENSES


Expenditure on each of the following items is required to be shown
separately:
1. Consumption of stores and spare parts
2. Power and fuel
3. Rent
4. Repairs to buildings
5. Repairs to machinery
6. Insurance
7. Rates and taxes, excluding taxes on income

7.12 PROFIT BEFORE EXCEPTIONAL ITEMS


AND TAX
Profit before exceptional items and tax is the eve
difference between total i
nue and total expenses.
7.13 EXCEPTIONAL ITEMS
ntional items are those items that occur during the ordinary course
B husiness but need to be disclosed due to their size or incidence.
ofthe
Examples are:
Profit/loss on disposal of surplus properties
Profit/loss on disposal of business/subsidiary.
29 loss on non-current assets of subsidiary companies
Impairment
4.
Restructuring costs
5. Impairment lOss on investments
6. Employee separation cost
to allow users of
The purpose of reporting exceptional items separately is
to generate income
financial statements to assess the ability of the business
from its regular operating activities.
Profit before tax is calculated after
deducting exceptional expenses from
profit before exceptional items and tax.
ACTIVITY 5
Name three items of exceptional nature.

7.14 TAX EXPENSE


income earned during an
company is required to pay income tax on the
in accordance with the provisions
A
accounting period. The tax is calculated for
of the Income Tax Act, 1961. The
tax expense is calculated separately
operations. A detailed treatment of
continuing operations and discontinued
Income tax expense is provided later
in this chapter.

FOR THE PERIOD FROM


7.15 PROFIT (LOSs)OPERATIONS
CONTINUING
continuing operations is calculated after
rofit (loss) for the period from profit/loss
aeducting tax expense from profit
before tax. It measures the
predictions about a company's
in making
Om ongoing operations and helps
future earnings.

OPERATIONS
1.16 DISsCONTINUED
a line of activity or has entered
If the company has
e decided to discontinue
segment of the business, the results of such line of
to sell a loss.
Contract
activ shown separately in the statement of profit and
y or segment are financial statements to better evaluate the perfor
This
s enables the users of operations.
nance of
the mpany's continuing
7.17 PROFIT (LOSS) FOR THE PERIOD
Profit (loss) for the period is the sum of profit (loss) from continuing opera.
tions and profit (loss) from discontinued operations.

7.18 OTHER COMPREHENSIVE INCOME


The performance of a company is reported in the statement of profit and lose
and other comprehensive income. Profit or loss is the total of income les
expenses, excluding the components of other comprehensive income'. Other
comprehensive income (OCI) comprises items of income and expense (includ.
ing reclassification adjustments) that are not recognized in profit or loss as
required or permitted by different accounting standards. Thus, other compre.
hensive income comprises those items that are not reported on the statement
of profit and loss but have an effect on the balance sheet amount of equity.
The components of other comprehensive income include:
1. Changes in revaluation surplus;
2. Actuarial gains and losses on defined benefit plans;
3. Gains and losses arising from translating the financial statements of a
foreign operation;
4. Gains and losses on remeasuring available-for-sale financial assets;
5. The effective portion of gains and losses on hedging instruments in a
cash flow hedge.

ACTIVITY 6
Explain the term 'Other Comprehensive Income' and provide two
examples.

7.19 EARNINGS PER SHARE


Earnings per share (EPS) of equity share capital is an important accounting
statistie widely used by existing and prospective equity shareholders of the
company. Companies are required by accounting standards to present the
EPS on the face of the statement of profit and loss to improve performance
comparisons between different entities in the same reporting period and
between different reporting periods for the same entity.

7.19.1 BASIC EARNINGS PER SHARE


An entity is required to calculate basic earnings per share amounts for proit
or loss attributable to ordinary equity holders of the entity and, if presented
profit or loss from continuing operations attributable to those equity holders.
Basic earnings per share shall be calculated by dividing profit or loss attribut
able to ordinary equity holders of the entity (the numerator) by the weighted
average number of ordinary shares outstanding (the denominator) during
the period.

NMIMS Global Access- School for Continuing Edueation


Profit or loss (profit or loss from
continuing operations)
Attributable to ordinary equity holders
Basic EP'S=
Weighted average number of ordinary
shares outstanding

The weighted average number of ordinary shares outstanding during the


oriod is the number of ordinary shares outstanding at the beginning of the
period, adjuste
Isted by the number of ordinary shares bought back
or issued
during the period multiplied by a time-weighting factor: The time-weighting
aotor is the number of days that the shares are outstanding as a
proportion
of the total number of days in the period.

For example, if the number of ordinary shares outstanding is 100,000 during


the first nine months of the accounting year and 150,000 during the last three
months, the weighted average number of ordinary shares outstanding during
the accounting year is 112,500 (100,000 x 9/12 + 150,000x 3/12).

7.19.2 DILUTED EARNINGS PER SHARE


Companies are also required to report diluted earnings per share in addition
to basic earnings per share. The purpose of reporting the diluted earnings
per share is to inform investors about the potential dilution that might occur
in the earnings per share.
For the purpose of calculating diluted earnings per share, profit or loss
attributable to ordinary equity holders of the entity and the weighted aver-
age number of shares outstanding is adjusted for the effects of all dilutive
potential ordinary shares.
If in the previous example, the company had 50,000 stock options outstand-
ing at the beginning of the accounting period entitling the holders of the
option to get one ordinary share, the weighted average number of ordinary
shares outstanding during the accounting year for the purpose of calculating
the diluted EPS will be 162,500 (150,000 x 9/12 +200,000 x 3/12).

7.20 INCOME TAXES


7.20.1 ADVANCE TAX
Even though the income of the Previous Year is taxable during the Assessment
ear assesses have to pay taxes as they earn, that is, in advance. Whenever a
ompany makes advance payment of income tax, it debits Advance Income
1ax Account and credits Bank Account.
ncome tax paid in advance is shown as tax asset (current or non-current)
in the balance
sheet.
1.20.2
PRoVISION FOR TAX
e end of each accounting year, a company calculates its taxable income
d ncome tax liability in accordance with the Income Tax Act. The com-
pany
makes a provision for income tax by debiting Profit and Loss Account
Crediting provision for Income Tax Account.
Provision
ax is shown as 'current tax liability' in the balance sheet.
for tax
7.21 DIVIDEND
Dividend is the distribution of profits by a company
interim
to
its shareholdere
dividend fina
two types of dividends: and
Ompanies generally pay
dividend.
7.21.1 INTERIM DIVIIDEND
a financial year. Section 123(2
nterinm dividend is paid by a companythatduring
the Board of Directors of a compan.
of the Companies Act, 2013 provides year out of the surplus iin
may declare interim dividend during any financial the financial year in which
the profit and loss account and out of profits of
such interim dividend is sought to be declared.

7.21.2 FINAL DIVIDEND


every annual meeting
general of the company, the shareholders consider
for approval dividend
any that the directors propose to pay to the share.
known as the final
holders for a financial year. This dividend is generaly subject to the require
dividend. The declaration of the final dividend is also
ments of Section 123 of the Companies Act, 2013.

7.21.3 ACCOUNTING TREATMENT OF DIVIDENDS


All dividends paid by the company are shown as deduction from the "Retained
Earnings" under "Reserves and Surplus" in the balance sheet.

7.22 SUMMARY books of account and


Explain requirements relating to corporate
financial statements. Companies Act, 2013 requires every company to
prepare and keep books and papers and financial statements for every
financial year, which give a true and fair view of the state of afairs of the
company.
Explain the form and contents of corporate financial statements.
Financial statements of a company include the balance sheet; profit
and loss account; cash flow statement; statement of changes in equity, if
applicable and explanatory notes annexed to these statements. Financial
statements are required to give a true and fair view of the state of affairs
of the company or companies, comply with the notified accounting stan-
dards anmd shall be in the form or forms as may be provided for different
class or classes of companies in Schedule III.
Prepare corporate financial statements. Income and expense accounts
from the trial balance accounts are carried to the Profit and Loss account.
The net result of the rofit and Loss account represents the net pro
or loss made by the business during the accounting period. Asset and
Liability accounts are transferred to the Balance Sheet along with the
net result of the Profit and Loss account.
1. Current asset is an asset that is expected to be realized
in, or is
KEYWORDS
intended for sale or consumption in, the company's normal operating
cycle; held primarily for the purpose of being traded; expected to be
realized within 12 months after the reporting date; or is cash or cash
equivalent unless it is restricted from being exchanged or used to
settle a liability for at least 12 months after the reporting date.
2, Current liability is a liability that is expected to be settled in the
company's normal operating cycle; it is held primarily for the
purpose of being traded; it is due to be settled within 12 months after
the reporting date; or the company does not have an unconditional
right to defer settlement of the liability for at least 12 months after
the reporting date.
3. Financial statement in relation to a company includes a balance sheet
as on the end of the financial year; a profit and loss account for the
financial year; cash flow statement for the financial year; a statement
of changes in equity, if applicable and related explanatory notes.
4. Non-current asset is an asset other than a current asset.
5. Non-current liability is a liability other than a current liability
6. Operating cycle is the time between the acquisition of assets for
processing and their realization in cash or cash equivalent. Where
the normal operating cycle cannot be identified, it is assumed to
have a duration of 12 months.
7. Other comprehensive income (OCI) comprises those items that are
not reported on the statement of profit and loss but have an effect on
the balance sheet amounts.
8. Trade payable is a payable in respect of the amount due on account of
goods purchased or services received in the normal course ofa business.
9. Trade receivable is a receivable in respect of the amount due on
account of goods sold or services rendered in the normal course of a
business.

7.23 DESCRIPTIVE QUESTIONS


1. A debt that is payable within a year is a current liability. Do you agree?
Explain.
2. What are the requirements relating to the presentation of non-current
liabilities in financial statements?
3. What are the two major components of equity?
4. How do you account for depreciation of fixed assets in financial
statements?
5. What is the difference between interest aeerued and due and interest
accrued but not due?
6. Name the broad heads under which assets and liabilities are classified
in the form of a balance sheet preseribed by Schedule III to the
Companies Act, 2013.
7. What liabilities are classified as current liabilities?
8. What assets are classified as non-current assets ?
9. How would you define an operating cycle?
10. What receivable is classified as trade receivable?

7.24 ANSWER KEY


SELFASSESSMENT QUESTIONS
Topics Q. No. Answers
Equity c. Equity share capital of Rs. 3,000,000 and
securities premium of Rs. 1,500,000
2. a. Current maturities of long-term debt
3. b. Accounts payable

7.25 SUGGESTED BOOKS AND E-REFERENCES


SUGGESTED BOOKS
Text
Anthony R.N., D.E. Hawkins and Merchant K.A. (2015). Accounting
and Cases, Tata McGraw Hill.
C.T, Sundem G.L., and Elliot J.A. (2014). Introduction
to
Horngren
Financial Accounting, Pearson Education.

E-REFERENCES
Ohttp://www.ezinearticles.com/Accounting convention and Accounting
theory; accessed on 25/11/2010.
http://www.Accountingformanagement.com/accountingtheory and con-
cepts; accessed on 25/11/2010.

NMIMS Global Access School for Continuing


Education
CH A
8 PT ER

STATEMENT OF CASH FLOWS

CONTENTS
8.1 Introduction
8.2 Cash and Cash Equivalents
8.3 Purposes of Cash Flow Statement
Self-Assessment Question
8.4 Operating Activities
8.5 Investing Activities
8.6 Financing Activities
Activity
8.7 Reporting Cash Flows from Operating Activities
8.7.1 Direct Method
8.7.2 Indirect Method
Self-Assessment Questions
Activity
8.8 Reporting Cash Flows from Investing Activities
Self-Assessment Question
8.9 Reporting Cash Flows from Financing Activities
Self-Assessment Questions
8.10 Treatment of Special Items
8.10.1 Foreign Currency Cash Flows
8.10.2 Interest and Dividend
8.10.3 Taxes on Income
8.10.4 Non-Cash Investing and Financing Transactions
8.10.5 Components of Cash and Cash Equivalents
8.10.6 Other Disclosures
Self-Assessment Questions
Activity
8.11 Format of Cash Flow Statement (Direct Method)
8.12 Format of Cash Flow Statement (Indirect Method)
8.13 Summary
Key Words
8.14 Descriptive Questions
8.15 Answer Key
Self-Assessment Questions
8.16 Suggested Books and E-References
INTRODUCTORY CASELET

OMAX ELECTRONICS

Omax Electronics produces


and sells computer games. The average sell-
ing price is Rs. 3,850 per
unit, variable cost is Rs. 2,450 per unit and fixed
eXpenses are Rs. 700,000 per month. At the start of the year 2019, the
accounts books revealed the following
balances

Rs. Rs.
Cash 2,500,000 Equity 6,787,500
Inventories 2,450,000
Receivables 1,837,500

he sales during January, February, March and April were 1,000, 1,500,
2,000 and 2,500 units, respectively. The company had a policy of produe
ing the expected
quantity of sales one month prior to the sales. All sales
were on one month's credit. The cash flows for the first three months are
presented below:

January February March


Net Income 700,000 1,400,000 2,100,000
Increase in (1,225,000) (1,225,000) (1,225,000)
inventories
Increase in (612,500) (1,225,000) (1,225,000)
receivables
Net decrease in 1,137,500) (1,050,000) (350,000)
cash balance
Beginning cash 2,500,000 1,362,500 312,500
balance
Ending cash 1,362,500 312,500 (37,500)
balance

The management is puzzled as to why, despite


the company is facing shortage of cash. increasing income levels,

&QUESTION
1. Why should the management
be concerned about
cash balance? (Himt: Negative cash the negative
will not be in a position to balances mean the company
meet its
financial obligations as they
arise without liquidating some
assets.)

NMIMS Global Arrem


LEARNING OBJECTIVES

After reading this chapter, you will be able to


Understand the purpose of preparing the cash flow statement.
Understand the classification of cash flows from different activities.
Understand the difference between direct and indirect methods of
computing cash flows from operating activities.
Understand how to deal with certain special items such as income
taxes, foreign curreney cash flows, cash flows from interest and
dividend, etc., in preparing the cash flow statement.
Understand how to deal with financing and investing activities
that do not involve any cash flow.

8.1 INTRODUCTION cash due to the pres


entity earning handsome profits may face shortage of statement. The entity
n

nce of accruals, deferrals and non-cash items in its income


to meet its obliga
eeds sufficient amount of cash to sustain its operations and
ons towards creditors and investors. Cash flow statement a is statement that
change or movement
hows the flow of cash during a period. Flow here means
are classified as cash inflows, and
cash. Transactions which increase cash cash outflows. Information
ransactions which decrease cash are classified as significance to investors and
ontained in the cash flow statement is of particular
to project future cash flows and form
reditors as they can use past cash flows
to honor its obligations towards them.
n
opinion about the ability of the entity

8.2 CASH AND CASH


EQUIVALENTS
equiv-
demand deposits with bank) and cash are
ash includes cash (cash on hand, highly liquid investments that
alents. Cash equivalents are short-term, insignif-
eadily convertible into known
amounts of cash, and are subject to anperiod of
example, securities with a maturity IMPORTANT CONCEPT
cant risk of changes in value, for acquisition (acquisition of debt or preference
the date of
months or less from bank deposits with a short maturity
period). Cash includes cash on hand,
shares shortly before redemption, demand deposits with bank
Statement" and Indian
standards AS-3 "Cash Flow by the and cash equivalents.
lwo accounting
(Ind AS-7) "Statement of Cash Flows" issued prepa- Cash Equivalents are
iccounting Standard 7 for the
of India, contain guidelines
nstitute of Chartered AccountantsAS-7 is applicable to certain specified com- short-term, highly liquid
ration of cash flow statement. Indoverdrafts that are repayable on demand as investments that are readily
panies. Ind AS-7 includes bank AS-3 is silent on this convertible into cash without
part of cash and cash equivalent, whereas the existing much risk of loss.
aspect.
STATEMENT
PURPOSES OF CASH FLOW
8.3 about the cash flows associated
information
ash flow statement provides financing
from the entity's investing andstatement
and also
with the period's operations standards require that the cash flow firom
dCtivities. Both the accounting classified into cash flows
the period,
shou report cash flows during
operating, investing and financing activities. Using the cash flow statement
shareholders, lenders and other users can assess:
1. Whether the
entity will be able to generate positive cash flows in the future.
2. Whether the entity will be able to meet its obligations and pay dividends.
3. Whether the entity needs to raise more funds.
4. Why there is a difference between the amount of net income and related
net cash flows from operating activities.
5. The effect of entity's investing and financing activities on its cash and
other accounts.
6. The reasons behind change in the beginning and ending balance of
cash and cash equivalents.

SELFASSESSMENT 1. What information would you find in a statement of cash flow that
QUESTION
you would not be able to get from the other two primary financial
statements?
a. Cash provided by or used in financing activities
b. Cash balance at the end of the period
C. Total liabilities due to creditors at the end of the period
d. Net income

8.4 OPERATING ACTIVITIES


Operating activities are the principal revenue producing activities of an
enterprise, and include activities that are not investing or financing activities.
Cash flows from operating activities include cash effects of those transactions
loss on sale
and events that determine the net profit or loss (except profit or
are:
of fixed assets). Some examples of cash flows from operating activities
1. Cash receipts from sale of goods or rendering of services.
2. Cash receipts from royalties, fees, commission and other revenue.
3. Cash payment to suppliers for goods and services.
4. Cash payment to and on behalf of employees.
5. Cash receipts and cash payments of an insurance entity for premiums
and claims, annuities and other policy benefits.
specifically
6. Cash payment or refund of income taxes unless they can be
identified with financing and investment activities.
7. Cash receipts and payments from contraets held for dealing or trading
purposes.

8.5 INVESTINGACTIVITIES
Investing activities include acquisition and disposal of long-term
o
and other investments not included in cash equivalents. Some examples
investing activities are:
1. Cash payments to acquire property, plant and equipment, intangble
and other long-term assets. These payments include those relating

NMIMS Global Access- School forContinuing Education


capitalized development costs and self-constructed
property, plant and
equipment.
2. Cash receipts from sale of property, plant and
equipment, intangibles
and other long-term assets.
3 Cash payments to acquire equity or debt instruments of other
entities and interests in joint ventures (other than payments for those
instruments considered to be cash equivalents or those held for dealing
or trading purposes).
4. Cash receipts from sale of equity or debt instruments of other entities
and interests in joint ventures (other than receipts for those instruments
considered to be cash equivalents and those held for dealing or trading
purposes).
5. Cash advances and loans made to other parties (other than advances
and loans made by a financial institution).
6. Cash receipts from repayment of advanes and loans made to other
parties (other than advances and loans of a financial institution).
7. Cash payments for future contracts, forward contracts, option contracts
and swap contracts except when the contracts are held for dealing or
trading purposes, or the payments are classified as financing activities.
8. Cash receipts from future contracts, forward contracts, option contracts
and swap contracts except when the contracts are held for dealing or
trading purposes, or the receipts are classified as financing activities.

8.6 FINANCING ACTIVITIESS


Financing activities are activities that result in changes in the size and com-
in case of a
position of the owner's capital (including preference share capital
financing activities are:
company) and borrowings of an enterprise. Examples of
1. Cash proceeds from issuing shares or other equity instruments.

redeem the entity's shares.


2Cashpayments to owners to acquire or notes, bonds, mortgages
3. Cash proceeds from issuing debentures, loans,
and other short- or long-term borrowings.
Cash repayments of amounts borrowed.
3. Cash payments by a lessee for the
reduction of outstanding liability
relating to a finance lease.

flows during a month: ACTIVITY 1


Acompany reports the following cash
.Paid salaries of Rs. 1,500,000
5,000,000
Purchased equipment costing Rs.
customers
3, Collected Rs. 3,500,000 from
Rs. 7,500,000
new shares collecting
Issued
.
Obtained a loan of Rs. 2,500,000
from the company's bank
cash flow is related.
ame the cash flow activity to which each

NMIMS Global Accessr School for Continuing Education


8.7 REPORTING CASH FLOWS FROM
OPERATING ACTIVITIES
Cash flows from operating activities can be calculated using either the direet
method or the indirect method.

8.7.1 DIRECT METHOD


In the case of direct method, gross cash receipts and gross cash payments are
shown under major classes. Cash receipts include cash sales, receipts from
cdebtors, commission and fee received and interest. Cash payments include
payments for purchases, payments to and for employees, operating expenses,
interest payments and direct tax payments.
The format for calculating cash flows from operating activities using the
direct method is given as follows:
IMPORTANT CONCEPT
Cash Flows from Operating Activities
Ind AS-7 "Statement of Cash Cash receipts from customers
Flows" encourages entities Cash paid to suppliers and employees
to report cash flows from Cash generated from operations
operating activities using the
rect method. Income taxes paid
Cash flows before extraordinary itemns
Extraordinary items
Net cash flows from operating activities
Cash receipts from customers include receipts from cash sales and receipts
from debtors in respect of eredit sales. To calculate the cash receipts from
debtors, credit sales need to be adjusted for change in the balance of debtors
during the accounting period. If the balance of debtors increases during the
accounting period, the cash receipts from credit sales will be less than the
amount of credit sales. The inverse will be the case when the balance of debt-
ors decreases during the accounting period. The relationship between credit
sales and cash receipts from credit sales is given by
Cash receipts from credit sales = Beginning balance of debtors +Credit sales
-Ending balance of debtors
Cash paid to suppliers includes payment for cash purchases and payments
to ereditors in respect of credit purchases. To calculate the cash payment to
suppliers, credit purchases need to be adjusted for change in the balance of
creditors during the accounting period. If the balance of ereditors inereases
during the accounting period, the cash payment for credit purchases will be
less than the amount of eredit purchases. The inverse will be the case when
the balance of creditors decreases during the accounting period. The rela
tionship between credit purchases and cash paid to suppliers is given by
Cash paid to suppliers = Beginning balance of creditors +Credit purchases
-
Ending balance of creditors
When the information relating to credit purchases or credit sales
is not avail-
able, the entire sales or purchases are assumed to be on
credit basis.
Ilustration 8.1
A company registers sales of Rs. 250 million in a year. The debtors at
the beginning and at the end of the year were Rs. 46 million and Rs. 84
million, respectively. Cash receipts from customers can be calculated in
the following manner.

Statement Showing Cash Receipts from Customers


(Rs. in Million)
Sales 250
Add: Debtors at the beginning
46
296
Less: Debtors at the end
84
Cash receipts from customers 212
The company in the above example reports its cost of sales during
the
year at Rs. 160 million. The inventories at the beginning and at the end
of the year were Rs. 14 million and Rs. 18 million, respectively. Creditors
at the beginning and at the end of the year were Rs. 33 million and
Rs. 30 million, respectively. Cash payments to suppliers can be
calcu-
lated in the following manner.

Statement Showing Cash Payments to Suppliers


(Rs. in Million)
Cost of sales 160
Add: Creditors at the beginning 33
Stock at the end 18
51
Less: Creditors at the end 30
Stock at the beginning 14
44
Cash paid to suppliers 167

Sased on the above information the cash generated from operating


activities using the direct method can be calculated as follows:

Cash Flows from Operating Activities


Cash receipts from customers 212
Cash paid to suppliers and employees 167)
Cash generated from operations 45

Some
emes, of purchases is embedded in the amount of cost of
sales.In suchthe amount amount of cost of sales hastto be adjusted for change
in the amno a case, the the accounting riod. The amount of
closing 10unt of inventories during
g
inven inventories is added to the cost of sales and the ount of beginning
Cntories is deducted from the resulting sum. The relationship between
cost of sales
and purchases is given by
Cost of sales = Beginning inventories +Purchases Ending inventories
out
Payments to employees also need to be adjusted for prepayments and
standing amounts. For example, the expense on salaries and
wages during
10,000 has been paid
an accounting period is Rs. 100,000. An amount of Rs.
paid at the end
as advance salary while an amount of Rs. 15,000 has not been 95,000
case is Rs.
of the accounting period. The payment to employees in this
(Rs. 100,000 + Rs. 10,000 Rs. 15,000).
-

8.7.2 INDIRECT METHOD


by the income
Under the indirect method, the net profit or loss disclosed
statement is adjusted for:
1. Non-cash items such as depreciation, provisions and
unrealized foreign
exchange gains or losses.
2. change in current assets and current liabilities.
3. Any deferrals or accruals of past or future
operating cash receipts or
payments.
4. All other items that affect investing or financing cash
flows.
using the indi-
The format for calculating cash flows from operating activities
rect method is given as follows:

TIP Cash Flows from Operating Activities


-QUICK
Net profit before tax and extraordinary items
eciation is added back Adjustment for:
net income to calculate
cash flows from operating Depreciation
activities even though it is an Non-cash items
operating expense. The reason Non-operating items (dividend, interest income)
is that there is no cash outflow
Operating profit before working capital changes
associated with depreciation.
Working capital adjustment
Cash generated from operations
Income taxes paid
Cash flows before extraordinary items
Extraordinary items
Net cash flows from operating activities

Illustration 8.2
Following information is available from the books of a
company.
(Rs.)
2014 2015
Net profit
500,000
Depreciation
25,000
Income received in advance
1,000 1,200
Prepaid expenses 3,200 2,800
Debtors 210,000 230,000
Creditors 116,000 110,000
Outstanding expenses 5,000 4,000
Accrued income 3,000 2,400
Cash flow from operating activities can be calculated using the indirect
method as follows:
Net profit 500,000
Add: Depreciation 25,000
Operating profit before working capital changes 525,000
Increase in income received in advance 200
Decrease in pre-paid expenses 400
Increase in debtors (20,000)
Decrease in creditors (6,000)
Decrease in outstanding expenses (1,000)
Decrease in accrued income 600
Cash generated from operations 499,200

2. A company had a net income of Rs. 165,000 during 2015. It provided SELF-ASSESSMENT
for a depreciation of Rs. 75,000 during the year. During the year, QUESTIONS
accounts receivable increased by Rs. 55,000 and accounts payable
increased by Rs. 25,000. The company's cash flow from operating
activities was
a. Rs. 320,000 b. Rs. 170,000
C. Rs. 210,000 d. Rs. 120,000
3. Decrease in the amount of creditors results in .
a. increase in cash b. decrease in cash
C. decrease in assets d. no change in assets

entities to report cash flows


ro AS-7Statement of Cash Flows"
nd encourages
sVides operating activities using the direct method. The direct method pro-
information that may be useful in estimating future cash flows and is
0 available under the indirect method.

6 pany reports a net income of Rs. 500,000 for the recently ended year ACTIVITY 2
charging depreciation of Rs. 50,000 and loss on sale of equipment
S. 25,000. The inventory atRs. the beginning of the year was Rs. 150,000
160,000. Determine the cash flows from
at the end of the year was
perating activities during the year:
8.8 REPORTING CASH FLOWS FROM
INVESTING ACTIVITIES
CAsh flows from investing activities arise from purchase and sales of fixed
assets and financial assets. These also include receipt of dividends and inter.
est. Cash Ilows from investing activities are calculated from the changes in
the balance of fixed assets and investments. The cash effect of any transac.
tions related to these assets during the accounting period is also considered

SELFASSESSMENT
QUESTION During 2015, a company purchased land for Rs. 3,150,000. The
company also sold a building for Rs. 950,000. The company's cash
flow from investing activity was
a. Rs. 46,50,000 b. Rs. 28,50,000
C. Rs. 28,00,000 d. Rs. 35,50,000

QUICK TIP
8.9 REPORTING CASH FLOWS FROM
When a company issues FINANCING ACTIVITIES
shares for cash, cash flow Cash flows from financing activities arise from issue and redemption of cap-
ital and loans. These also include payment of dividends and interest.
pancing activity is not Cash
balance
value of the shares flows from financing activities are calculated from the changes in the
but the amount actually of shareholders' funds and borrowings. The cash effect of any transactions
ollected by the company. related to these items during the accounting period is also considered.

SELF-ASSESSMENT 5. Dividend paid is always classified as a/an_


QUESTIONS b. investing activity
a. operating activity
c. financing activity d. none of the above

6. Which of the following is not a financing activity in the cash flow


statement of a non-tinance company?
a. Issue of shares b. Payment of dividends
C. Receipt of dividends d. Borrowing money from a bank

Illustration 8.3

Following information is available from the books of a company:

As on 31.3.2014 As on 31.3.2015
Particular (Rs.) (Rs.)
Equity share capital 9,330,000 15,300,000
Preference share 2,530,000 2,930,000
capital
Loans 116,500,000 115,200,000
Dividend paid
4,660,000
Cash flow from investing activities
can now be worked out as follows:
Cash Flows from Financing Activities
Issue of share capital 5,970,000
Issue of preference capital 400,000
Repayment of loans (1,300,000)
Dividend paid
4.660.000
Net cash inflow from financing activities 410,000

8.10 TREATMENT OF SPECIAL ITEMS


8.10.1 FOREIGN CURRENCY CASH FLOWSs
An entity should record cash flows arising from transactions in a foreign cur-
rency in
the entity's functional currency by applying to the foreign currency
amount the exchange rate between the functional currency and the foreign
currency on the date of cash flow. Functional currency is the currency of the
primary economic environment in which the entity generates and expends
cash. The cash flows of a foreign subsidiary should be translated at the
exchange rate between the functional currency and the foreign currency
on
the dates of cash flows.

8.10.2 INTEREST AND DIVIDEND


Cash flows from interest and dividends received and paid
should be dis-
paid, and interest and
closed separately. Cash flows arising from the interest
dividends received in the case of a financial enterprise
should be classified as
cash flows arising from operating activities. In the
case of other enterprises,
be classified as cash flows from
cash flows arising from interest paid should received should be classi-
hnancing activities, while interest and dividends
ned as cash flows from investing activities.
Dividends paid should be classi-
lied as cash flows from financing activities.

8.10.3 TAXES ON INCOME


are to be separately disclosed, and
Lash flows arising from taxes on income
operating activities unless they can
eed to be classified as cash flows from
E Specifically identified with financing
and investing activities.

o.l0.4NON-CASH INVESTING AND FINANCING


TRANSACTIONSS
that do not require the use of cash or
sing and financing transactions from the cash flow statement. Such
equivalents should be excluded the financial statements in
dnsactions should be disclosed elsewhere in
ay that provides all the relevant information about the investing and
linancing activities.
Examples of non-cash transactions are:
or by-
The acquisition of assets by assuming directly related liabilities
means of a finance lease.
shares.
he acquisition of an enterprise by means of issue of
3. The conversion of debt to equity.
EQUIVALENTS
8.10.5 COMPONENTS OF CASH AND CASH
equivalents
An entity is required to disclose the
components of cash and cash cash flows
amounts in its statement of
and to present a reconciliation of the balance sheet. An entity also
has=
reported in the
wtn the equivalent items determining the composition of cash
to disclose the policy which it adopts in policy for determin-
of any change in the
and cash equivalents and the effectequivalents. a change in the
For example,
components of cash and cash
ing
of financial instruments previously considered to be part of an
classification
entity's investment portfolio.

8.10.6 OTHER DISCLOSURES


by manage-
An entity is required to disclose, together with a commentary
equivalent balances held by-
ment, the amount of significant cash and cash
group. There are various cir-
the entity that are not available for use by the
Cumstances in which cash and cash equivalent
balances held by an entity are
cash and cash equiva-
not available for use by the group. Examples include
in a country where exchange
lent balances held by a subsidiary that operates are not available
controls or other legal restrictions apply when the balances
for general use by the parent or other subsidiaries.

SELFASSESSMENT 7. Which of the following cash flow activities represents a non-cash


QUESTIONS financing transaction?
a. Purchase of goods for cash
b. Issue of shares for cash
c. Sale of equipment for cash
d. Purchase of plant by issuing shares
8. Which of the following cash flow activities represents a non-cash
investing transaction?
a. Purchase of goods for cash
b. Issue of shares for cash
C. Sale of equipment for cash
d. Exchange of plant assets

ACTIVITY 3 Identify the cash flow activity associated with


the following cash flows:
1. Dividend received by a financial
enterprise
2. Payment of income tax
3. Payment of dividend by a
non-financial enterprise
4. Interest paid by a financial
enterprise
5. Interest received by a non-financial
enterprise
11 FORMAT OF CASH FLOW STATEMENT
(DIRECT METHOD)
A. Cash flows from operating activities
Cash receipts from customers
Cash paid to suppliers and employees
Cash generated from operations
Income taxes paid
Cash flows before extraordinary items
Extraordinary items
Net cash flows from operating activities
B. Cash flows from investing activities
Purchase of fixed assets
Purchase of investments (-)
Sale of fixed assets
Sale of investments
Interest received
Dividend received
Net cash flows from investing activities
C. Cash flows from financing activities
Proceeds from issue of share capital
Proceeds from long-term borrowings
Repayment of long-term borrowings
Dividend paid
Net cash flows from financing activities
Net increase (or decrease) in cash and
cash equivalents (A+B +C)
Cash and cash equivalents as at-
(opening)
Cash and cash equivalents as at ---
(closing)

8.12 FORMAT OF CASH FLOW STATEMENT


(INDIRECT METHOD)
A. Cash flows from operating activities
Net profit before tax and extraordinary items
Adjustmentfor
Depreciation
Non-cash items
Non-operating items (dividend, interest income)
(Continued)

NMIMS Global Access- School for Continuing Education


KEY WORDS
disposal of long
hvesting activities include the acquisition andin cash equivalents.
em assets and other investments not included
8. peraing activities are the principal revenue producing activities of
not investing or financing
ne enterprise and other activities that are
activities.

8.14 DESCRIPTIVE QUESTIONS


1. What is a cash flow statement?
cash flow statement?
What are the main purposes of preparing a
. reporting cash flows in.
Describe the three kinds of activities usedofforcash flows from different
examples
bhe cash flow statement, giving
activities.
also involve certain non-cash
4F'inancing and investing activitiesof such transactions and state how
transactions. Give some examples
these transactions are disclosed.
. Describe the direct method of determining
cash flows from operating
activities.
arriving at the
6. State the adjustments made to
the income figure for method.
under the indirect
cash flows from operating activities
examples.
7. Define cash equivalents. Give three
expense
treatment of interest and dividend income and
8. Describe the
by financial enterprises and
other
in the cash flow statement prepared
enterprises.

8.15 ANSWER KEY


SELFASSESSMENT QUESTIONS
Q. No. Answers
Topics 1. a. Cash provided by or used in
Purposes of Cash Flow financing activities
Statement
2. c. Rs. 210,000
Reporting Cash Flows from
Operating Activities
3. b. decrease in cash
4. c. Rs. 28,00,000
Reporting Cash Flows from
Investing Activities
5. c. financing activity
6. C. Receipt of dividends
Treatment of Special Items 7. d. Purchase of plant by issuing shares
8. d. Exchange of plant assets

rTMS Clobal Aceess- School for Continuing Education


ISUGGESTED
16 SUGGES1 BOOKS AND E-REFERENCES
SUGGESTED BOOKS

Anthony R.N., D.E. Hawkins and K.A. Merchant (2015).


Accounting Text
and Cases, Tata MeGraw Hill,
Horngren C.T
T., Sundem G.L. & Elliot J.A. (2013).
o Introduction to Financial
Accounting, Pearson Education.

E-REFERENCES

a http:/mca.gov.in/Ministry/pdf/INDAS7,pdf; accessed on 15/10/2019.


n https://www.charteredclub.com/cash-flow-statement-direct-indirect-
method; accessed on 15/10/2019.

NMIMS Global Access- Snhonl fam


HA
9 PT ER
ANALYSIS OF FINANCIAL STATEMENTSI

CONTENTS

9.1 Introduction
9.1.1 Additional Information
9.2 Profitability Measures
9.2.1 Profit Margin
9.2.2 Earnings per Share
9.2.3 Return on Capital Employed
9.2.4 Decomposing Return on Capital Employed
9.2.5 Return on Equity
9.3 Tests of Efficiency in Investment Utilization (Efficiency Ratios)
9.3.1 Inventory Turnover Ratio
9.3.2 Debtors' Turnover Ratio
Self-Assesment Question
9.3.3 Creditors' Turnover Ratio
9.3.4 Cash-to-Cash Operating Cycle
Activity
9.4 Tests of Financial Position
9.4.1 Current Ratio
9.4.2 Quick Ratio
9.4.3 Debt-Equity Ratio
9.4.4 Interest Coverage Ratio
Self-Assessment Question
9.5 Ratios Involving Share Information
9.5.1 Dividend Payout Ratio
9.5.2 Dividend Yield
9.5.3 Price/Earnings Ratio (P/E Ratio)
Self-Assessment Question
9.6 Limitations of Ratio Analysis
Self-Assessment Questions
ETTER INVESTMENT OPTION

An investor
is considering investment in one of the two companies
A and B. He collects the following financial information relating to the
A and
two companies for the most recent accounting year:

Company A Company B
3,050
Total Revenue (Rs. Million) 1,470
Million) 367 671
Gross Profit (Rs.
305
Operating expenses (Rs. Million) 220
122
Financial expenses (Rs. Million) 37
110 183
Million)
Net Profit (Rs.
400 1000
Equity share capital 220
90
Reserves and surplus 815
245
Debt
a better
is of the opinion that Company B should be that of
The investor revenue and profit than
investment as Company B has higher
Company A.

QUBSTrON
in Company A
or
investor to invest
1. Would you advise the profitability and financial
position
Company B? (Hint: Analyze
decision.)
ratios to arrive at the
LEARNING OBJECTIVES

After reading this chapter, you will be able to: profitability,


compute ratios used ín analyzing
Understand and position and market stand.
elliciency in asset utilization, financial
ing of a company.
Understand limitations of ratio analysis.

9.1 INTRODUCTION
of relationships between the
elements
inancial statement analysis is the study the trend of these
statement different financial statements and mean-
o the same or
statement analysis is to determine the
elements. The purpose of financial statements so that a forecast
significance of the data contained in the and the
hg and
the prospects for future earnings, expected divdends useful
may be nmade of provides
the business to pay interest and debt as it matures. It staterments.
aDilty of contained in financial
information that supplements the information
Analysis -
one of the components of Financial Statement
Let's understand
"Ratio Analysis".
tool of financial analysis that is used by inves-
Ratio Analysis is an important
financial decisions.
tors and lenders to make important
QUICKQUICKTIP relationships between
Analysis is a technique of establishing meaningful relationships
Ratio
ratio may be expressed signiticant variables of financial
statements, and interpreting the
A analysis is
as a number, a fraction, a to form judgment regarding the
financial affairs of the unit. Ratio
condition of
percentage or a proportion. employed to assess the profitability, efficiency and financial
usually classified as:
an enterprise. Depending on the
purpose they serve, ratios may be
1.Measures of Profitability
Utilization (efficiency ratios)
2. Tests of Efficiency in Investment
3. Tests of Financial Position
4. Ratios involving Share Information.

TABLE 9.1 STATEMENT OF PROFIT AND LOSS OF ASIAN PAINTS


LTD. FOR THE YEAR ENDED MARCH 31, 2014
Rs. (in Million) Rs. (in Million)
104,187
Net sales and other operating revenue
Other income 1,737
105,924
Expenses
Cost of materials consumed 57,587
Purchase of stock-in-trade 2,566
Changes in inventories of FG. (753)
WIP and stock-in-trade
Employee benefit expenses 4,824
Otherexpenses 22,191 86,415
(Continued)

NMIMS Global Access School for Continuing


Education
ARLE 9.1 STATEMENT OF PROFIT AND
LOSS OF ASIAN PAINTS
LTD. FOR THE Y EAR ENDED MARCH 31,
2014-CONTINUED
Rs. (in Million) Rs. (in Million)
EBITDA
19,509
Depreciation and amortization 2,123
Finance cost 261
Profit before exceptional items
17,125
Exceptional items
100
Profit before tax 17,025
Tax expense 5,335
Profit after tax 11,690

Ratios are illustrated using the information contained in the following finan-
cial statements of Asian Paints Ltd. given in Tables 9.1 and 9.2 and additional
information.

Raw material purchased by the company during the year ended March 31,
2014 amounted to Rs. 58,531 million.

TABLE 9.2 BALANCE SHEET OF ASIAN PAINTS LTD.


AS ON MARCH 31, 2014
(Rupees in Million)
31.03.2014 31.03.2013
Equities and Liabilities
Shareholders' funds
Share capital 959 959
Reserves and surplus 35,050 29,263
36,009 30,222
Non-current liabilities
Long-term borrowings 395 463
Deferred tax liability net) 1,771 1,433
Long-term provisions 803 771
2,969 2,667
Current liabilities
Trade payables 14,988 12,141
7,475 7,194
Other current liabilities
Short-term provisions 5,375 4,237
27,838 23,572
Total 66,816 56,461

Assets
Non-current assets
Fixed assets
19,732 20,749
Tangible assets
(Continued)
TABLE 9.2 BALANCE SHEET OF ASIAN PAINTS LTD.
AS ON MARCH 31, 2014 CONTINUBD
(Rupees in Million)
31.03.2014 31.03.2013
Intangible assets 390 270
Capital work-in-progress 379 525
20,501 21,544
Non-current investments 5,482 3,597
Long-term loans and advances 946 911

Other non-current assets 63

Current Assets
Current investments 4,820 1,050

Inventories 16,650 14,808

Trade receivables 7,124 6,339

Cash and bank 7,454 5,515


2,015 1,627
Short-term loans and advances
1,761 1,070
Other current assets
39,824 30,409
66,816 56,461
Total

9.1.1 ADDITIONAL INFORMATION


each.
Share capital of the company consists of 959 million shares of Rs.
1

The company paid a dividend of Rs. 5.30 per share for the year ended
March
31, 2014. The total amount paid as dividend was Rs. 5,084 million, and the
from
company paid Rs. 820 million as dividend distribution tax. Cash flows
million.
operating activities for the year ended March 31, 2014 was Rs. 13,688
The closing price of the share on the Bombay Stock Exchange as on March
31, 2014 was Rs. 546.50.

9.2 PROFITABILITY MEASURES


The profitability ratios are used to check if the company is generating an
acceptable return for its owners. Both ereditors and investors are inter
ested in the profit-making ability of a company. Lack of adequate profitabil-
ity adversely affects the liquidity of the company, its ability to raise external
financing and its growth prospects. Widely used measures of profitability
include profit margins, earnings per share (EPS), return on capital employed
(ROCE), return on assets (ROA) and return on equity (ROE).
9,2.1 PROFIT
MARGIN
Profit margins are used to analyze the profit made per
unit of sales. Three
kinds of profit margins are generally used: gross profit margin, operating
profit margin and net profit margin.
Gross prof margin is caleulated as follows:

Gross profit ratio= Grossprofit 100


Sales

Censs profit is the difference between sales value and cost of goods sold. The
t goods sold is not directly provided in the Statement of profit and loss
cost of IMPORTANT CONCEPT
and needs separate computation.
Profitability of operations and efficiency of the management have a bearing Gross profit is the difference
between sales value and cost
on gross profit. Companies enjoying a monopoly in the market have a high
of goods sold.
gross profit ratio.
Operating profit is the profit before interest and tax and does not include
other income. Net profit is the profit after tax.

Profit before int erest and tax


Operating profit ratio = 100
Sales

For Asian Paints, the operating profit margin for the year ended March 31,
2014 is IMPORTANT CONCEPT

19,509 2,123x 100 =16.69%% Operating profit is the profit


Operating profit ratio before interest and tax and
104,187
does not include other
The operating profit is calculated as EBITDA minus depreciation and income.
amortization expense.
And the net profit margin for the year ended March 31, 2014 is
Net profit100
Net profitratio =
Sales
11,690x
100 11.22%
104,187

9.2.2EARNINGS PER SHARE


equity share. It is calcu-
arnings per share are the net income available per
lated as:

Net profit Preference dividend


-

Earnings per share= Number of equity shares


11,690 IMPORTANT CONCEPT
Rs. 12.19
959
Earnings per share are the net
of EPS: Basic EPS and income available per equity
Li companies are required to report two versions
d
Diluted EPS.
share.

NMIMS Global Access- School for Continuing Education


O nelps in evaluating the prevailing market price of the
share. Higher
because it indicates
ngs per share translate into a higher market price
Detter performance and prospects of the company

9.2.3 RETURN ON CAPITAL EMPLOYED


veturn on Capital Emploved measures the returns generated by the busi.
employed refers to total
ess on the amount invested in the business. Capital hence represents funds
Owhers funds and non-current liabilities, and time. It can also be caleu-
eurusted to a concern for relatively long periods ofThis ratio is calculated as:
1ated by adding net working capital to fixed assets.

Profit before interest and tax 100


Return on capital employed Average(Long-term liabilitie +
s

Owner's equity)

! IMPORTANTCONCEPT
For Asian Paints, ROCE for the year ended March
31, 2014 is
Capital employed refers to
total of owners' funds and 17,386
ROCE
= x100
non-current liabilities. (36,404+30,685)/2
17,386
100
33,545
= 51.8%

profits
ROCE becomes difficult to interpret when the total capital is low; the
the year and only used for
are volatile; new capital has been raised during
part of the year; and the assets are at historie values and are out of date.

A variant of this ratio is ROA. This ratio relates profit to investment in


the
enterprise and shows how much the firm has earned on the investment of al
the financial resources, that is, owners' equity, long-term liabilities and cur
rent liabilities. It is also expressed as a percentage. ROA is often used by the
top management to evaluate the performance of divisional managers in the
use of assets. The divisional manager has a significant influence over the use
of assets, but little control on how these assets are financed. ROCE is a better
measure for those division managers who have a significant influence on
asset acquisition, purchasing and production schedules, credit policy, cash
management and the level of current liabilities.

Profit before interest and tax


Return on assets =
100
Average total assets

For Asian Paints, ROA for the year ended March 31, 2014 is

ROA=- 17,386
x100
56461+66,816)/2
=28.2%

NMIMS Global Access- School for Continuing


Education
9,2.4
DECOM MPOSING RETURN ON CAPITAL
EMPLOYED
TheROCE can be decomposed into two ratios:
eet Utilization
Asset Util (Turnover) Ratio. It reflects the efficiency with which
ssets are utilized, and is calculated as:

Asset utilzation ratio = Sales revenue


Average (Shareholder's funds + Long-term debt)

ar Asian Paints, the asset utilization ratio for the year ended March 31, 2014
is
104,187/33,545 = 3.1.

Profit Margin (or Return on Sales) Ratio. It reflects the profits made per
unit of sales, and is calculated as:

Profit margin =
Profit before interest and tax
Sales revenue

For Asian Paints, the profit margin for the year ended March 31, 2014 is
17,386/104,187 = 0.167 or 16.7%.

ROCE is the product of the above two ratios:


IMPORTANT CONCEPT
ROCE = 3.1x 16.7% = 51.8%
Return on capital employed
depends on efficiency in
Companies selling undifferentiated products (e.g. fast moving consumer utilization of assets (asset
goods) generally work on low margins and high turnover. On the contrary,
turnover) and return on sales
companies selling differentiated products (e.g. customized furniture) work (profit margin).
on high margins and low turnover. The decomposition of ROCE into Profit
Margin and Asset Utilization Ratio, also called the "Du-Pont Analysis", helps
analysts to understand the reason underlying the change in ROCE among
different companies during a time period and for the same company over a
period of time. Comparison of profit margin and turnover usually is mean-
ngful only in evaluating firms in the same industry. Cross-industry compari-
Son of these two ratios is often meaningless and can even be misleading.

9.2.5 RETURN ON EQUITY


Ihe return on equity relates profits to owners equity, and
is expressed as a
percentage. Equity stands for owners' funds and includes equity share cap-
lal, general reserves, capital reserves, balance in share
premium account
Eind other reserves available to shareholders. However, accumulated losses
and fictitious assets to be written off should be deducted from equity. The
crm"profit" here means the profits to which the shareholders are entitled to
auLer meeting all expenses, including interest on loans and income tax. Also

0n-operating incomes such as interest income on investment are included


profit. The ratio is calculated as under:

Profit after interest and tax Preference dividend


-
Returnon equity =F
Average equity
1s
March 31, 2014
51an Paints, ROE for the vear ended
= 35.3%
11,690 x 100
ROE (36,009-+30,222) /2
up of reserves
declaration of dividend and buildingin the business
S ratio may be used for
It also indicates the efficiency with which funas 10% and the
nterest available
inte
on equity is are not profitably
For example, if the rate of return indicate that the funds
!IMPORTANT CONCEPT on Dank deposits is 12%, it would in unat enterprise.
not be willing to invest
deployed and the investors would ularly its trend over
Return on equity depends on
firm's ROE, particula "decompose
three factors, namely, profit a
To understand the factors affect to competitors, analysts olten
margin, asset turnover and
ne and its performance relativeof ratios.
leverage. ROE into the product of a series
EFFICIENCY IN INVESTMENT
9.3 TESTS OF RATIOS)
UTILIZATION (EFFICIENCY which a concern use
Ises
the effectiveness with usually calcu lculated on
measure are
he efticiency ratios
the resources or Ssets at its
disposal. These
ratios
expressed
per of times rather
in number
are
the basis of sales or cost of sales, and
type
should be calculated separately for eachusage
ratios of asset
than as a percentage. Suchratio, the more will be the efficiency to the concem
the
oI asset. The greater underutilization of resources available
concem:
A lower ratio will show efficiency ratios usually calculated by a
The following are the important
IMPORTANT CONCEPT RATIO
9.3.1 INVENTORY TURNOVER sold to the average
Efficiency ratios measure the turnover ratio relates the cost of goods is sold during
effectiveness with which a
The inventory
how many times the average inventory
inventory. It measures
of the opening inventory
and closing
concern uses the resources or year. Average inventory is the mean
the
assets at its disposal inventory. The ratio is calculated as:
Cost of goods sold or Sales
Inventory turnover ratio =
Average inventory

ratio for the year ended March 31,


For Asian Paints, the inventory turnover
2014, is
104,187
(14,808+16,650) /2
=
104,187
15,729
= 6.62

The cost of goods sold can either be calculated as (Sales - Gross profit) or as
(Opening stock + Purchases + Manufacturing expenses Closing stoek). na
the amount of cost of goods sold is not directly available from the finanelau
statements of the company, we have used the sales
of the ratio.
amount in the numet or
.
te watio reflects the efficiency of inventory
his management. A higher ratio 1s a
gn of higher efficiency and vice versa.
ueen a very high or a very low However, there is a need to balance
mely low level of inventory may ratio. A very high ratio resulting from
he inability to deliver goods result in loss of sales in future due to
promptly.
ficiency in inventory management can also
other ratio, the number of days' inventory be determined by calculating
dicates how long the inventory is held. Number of days' inventory
held by the company on an average.
his ratio is calculated as follows:

Number of days inventoryCostAverage inventory x 365


Cost of
of goods sold orSale
Sales

r as 365/1nventory turnover ratio.


or Asian Paints:

Number of days' inventory 15,729


= x 365
104,187
55 days
lternatively,
365
Number of days inventory = 55 days
6.62
his figure can be used to compare the efficiency in inventory management
vith other units in the same industry.

3.2 DEBTORS' TURNOVER RATIO


The debtors' turnover ratio shows the relation between sales and outstand-
ng amount due from the debtors to whom goods were sold on credit. The
atio is calculated as follows:

Net credit sales


Debtors turnoverratio =.
Average debtors

For Asian Paints,

104,187
Debtors turnover ratio =
(6,339+7,124)/2
104,187 = 15.48
6,732

For the
caleulation of this ratio, debtors include sundry debtors and trade
bills
receivables.It is preferable to take the average of the value of the debtors
in the beginning
and the end. If the company sells goods both for cash and on
Credit, only credit
sales figure should be used to calculate debtors' turnover
raio. Since the information on credit sales is not available in the financial
slatements of the company, the ratio may be calculated with reference to the
distorted, it still may be useful to
Sales figure. Though the ratio becomes cash
lonpare the ratio of an entity over time if the proportion ot eredit and
Sales remains constant from year to year:
prompt collection ot bills, and a lots
ow
gh debtors' turnover ratio shows difficulty in collection of dues from
io shows that the enterprise is having average col.
used to caleulate the
aebtors. Debtors' turnover ratio can be shows the accounts receivables in
ection period. Average collection period during a particular period. This
erms of the number of days of credit sales
taken for debtors to settle their
measure of the average length of timecalculated as follows:
Delance. Average collection period can be
365
Average collection periodDebtors' turnover ratio

For Asian Paints


365
Averagecollection period =
15.48
24 days

It can also be calculated as:

Average debtors x Number of days in a period


Averagecollection period
Net credit sales
6,732x 365
104,187
= 24 days

The average collection period shows how the credit policy of the concern is
enforced. For example, if a company allows 30 days' credit to its customers and
the collection period is 45 days, it means collection from debtors is not efficient.

SELF-ASSESSMENT
QUESTION 1. The debt collection period may increase (decrease)
period and another for a number of between one
reasons except for any one of
those mentioned below:
a. If credit is given to unsatisfactory
customers.
b. Earlier the business had a
zero debt collection period.
C. Credit terms to an
existing customer changes.
d. If there is no consistent
follow-up of overdue
debts.
9.3.3 CREDITORS'
TURNOVER RATIO
The creditors' turnover ratio
standing amount due to shows the relation
between
the ereditors
credit. The ratio is calculated from whom goods purchases and out-
as follows: were purchased on

Creditors' turnover
ratio = Credit purchases
Average creditors
NMIMS Global
Access- School fon Con
For
Asian Paints,

Creditors' turnoverratio= 58,531+2,566


(12,141+14,988)/2
61,097
13,565
=4.5
man
fhe calculation of this ratio, creditors include sundry creditors and trade
alls payables. It is preterable to take the average of the value of the
creditors
dnthe beginning and at the end. If the company
purchases goods both for cash
and on credit, only credit purchases figure should be used to caleulate credi-
tors'turnover ratio. Since the information on credit purchases is not available
in the financial statements of the company, the ratio may be caleulated with
reference to the total purchases figure. Though the ratio becomes distorted, it
still may be useful to compare the ratio of an entity over time if the proportion
oferedit and cash purchases remains constant from year to year.
Creditors' turnover ratio can be better interpreted by converting it into
Average Payment Period.
The average payment period shows the average number of days of credit that
the company has from its suppliers. It can be caleulated as follows:

365
Averagepayment period
Creditors turnover ratio
365
4.5
81days

It can also be calculated as:

Average creditors Number of daysin a period


Average payment period =
Credit purchases
13,56x 365 81 days
61,097

a long time to
a nighereditors' turnover ratio means that the company takes
pay tor credit purchases. This may be due to the company's ability to obtain
period is always good for
a 1ong
credit period from its suppliers. A long credit
a company's
cash flow.
the figures of purchases of
redit purchases are calculated by adding of raw materi
materials and purchases of stock in trade. Purchase
s 1s calculated by adjusting the figure of raw materials consumed with
nge in inventory of raw materials, raw materials transit, and
ang
in pack-
materials.

NMIMS Global Access- School for Continuing Education


annual report of the
11OWing is the schedule of inventories given in the
company:

31.03.2013 31.03.2014
4,788 5,091
Raw materials
123 700
Raw materials in transit 361
297
Packing material
5,208 6,152
8,446 8,923
F'inished goods 15
10
F'inished goods in transit 774
601
Work-in-progress 408
338
Stores, spares, and consumablesS 375
205
Stock-in-trade 3
Stock-in-trade in transit 16,650
14,808
Total

Credit purchases are calculated as follows:


57,587
Cost of materials consumed
(from statement of profit and
loss on page 162)
6,152
Add: Ending inventory of raw
materials, raw materials in
transit and packing materials
63,739
5,208
Less: Beginning inventory of
raw materials, raw materials in
transit and packing materials
58,531
Credit purchases of raw
materials
Purchases of stock-in-trade 2,566
Total credit purchases 61,097

9.3.4 CASHTO-CASH OPERATING CYCLE

QuUICK TIP Cash-to-cash operating cycle measure the length of time between purchase
of inventory and collection of cash from sales. It is the sum of number of days
Cash-to-cash operating cycle inventory and the average collection period.
is the sum of number of days'
inventory and the average Cash-to-cash operating cycle =
collection period.
Number of days' inventory+
Average collection period

NMIMS Global Aceess- School for Continuing


Education
Paints
ForAsian
Cash-to-cash operatingcycle = 55 days
+24 days= 79 days
ended March 31, 2014, it took Asian
r end
year
In the purchase Paints 79 days on an average
nvert inventories into cash.
to c

Match each of the following ratios with the associated formula ACTIVITY 1

Ratio Formula
Number of days' inventory A Number of days' inventory
Averagecollection period
Averagecreditors
2 Average collection period B 365
Credit purchases

Average inventory 365


3 Average payment period C Cost of goods sold or Sales
k

Average debtors
Cash-to-cash operating cyele D 365
Net credit sales

9.4 TESTS OF FINANCIAL POSITION


Tests of financial position include tests of both short-term and
long-term
solvency of the business. Tests of short-term solvency focus on the
liquidity
position of the company. While a business should be profitable,
profit, by
itself, is not sufficient to ensure survival of the business. A
company must
A company
have sufficient liquid assets to meet its short-term obligations.
could be forced into liquidation in the absence of sufficient
liquid funds. Twvo
Ratio and
mportant ratios used to measure short-term liquidity are Current
called "Liquidity Ratios".
Quick Ratio. These two ratios are commonly
Tests of long-term solvency focus on the ability of the
company to pay inter-
est and repay principal of its long-term borrowings.
The main ratios in this
coverage ratio. These ratios are
calegory are debt-equity ratio and interest
commonly called "Solvency Ratios".

9.4.1
CURRENT RATIO
assets to its current liabil-
Current ratio is the relation of a company's current short-term
ues. This ratio establishes the ability of the business to meet its
significance to short-term creditors.
gations and is therefore of particular that are expected to be converted
urrent assets are "cash and other assets goods or rendering of services in
LO cash or consumed in the productioninclude cash in hand, cash at bank,
of
These
uEnormal course of business." loans and advances, inventory, prepaid
undry debtors, bills receivables,
investments in the form of mar-
ses, accrued income and short-term
ex
ketable securities.
and bank over-
including loans, deposits
Current liability is a "liability relatively short period, normally not-
uratt which fall due for pavment in a short-term
liabilities include bank Overdratt,
12 months." Current proposed divi-
Te than creditors, provision for taxation,
s, bills payable, sundry
dend and outstanding expenses.
The current ratio is calculated as follows:
Currrent assets
Current ratio Trent liabilities
For Asian Paints,
39,8241.43
Current ratio27,838

for 2013-14; and


30,401.29
Currentratio23,572

for 2012-13.
A low current ratio (less than 1) mayindicate that a company would have
difficulty in paying bills as they become due without selling some long-term
assets. A high current ratio may not always be good as it may indicate too
much money being tied up in inventory, receivables and unproductive cash
balances. It is difficult to specify a normal level of current ratio as this level
differs from one industry to another It is advisable to compare a compa-
ny's current ratio with the industry average and to observe its trend over a
number of years.
SELFASSESSMENT
QUESTION 2. The current ratio of a company depends on a number of factors
listed below except one of the following options:
a. Volatility of the working capital requirement.
b. Nature of company's business.
C. Imminence of current liabilities.
d. Long-term investments of the company.

9.4.2 QUICK RATIO


Quick ratio is a more precise measure
This ratio is also known as "Acid Test of liquidity than the current ratio.
relates quick current assets to Ratio" or "Liquid Ratio".
current Quick ratio
those current assets, which are liabilities. Quick current
convertible into cash assets are
cash, marketable securities, rather early such as
treated as a quick current assetdebtors and bills receivables.
as it is not likely Inventory is not
ratio is calculated as follows: to be realized early. Quick

Quick ratio =urrent


assets-Inventories
Current liabilities

NMIMS Global Access


School for Continuing
Education
Asian Paints,
For
Quick ratio =39,824 -16,650 = 0.83
27,838
2013-14; and
for
Quick ratio = 30,40914,808 0.66
23,572
2012-13.
for
shows the ability of the firm to pay its obligations without relying
This ratio
he sale and collection of inventories. In a business, a 1:1 ratio of quick
urrent assetst
ts to current liabilities is treated as a satisfactory relation.
SELF-ASSESSMENT
TF X = Current assets Stocks)/(Current liabilities), X is known as
- QUESTION
3.

a. quick ratio b. acid test ratio


C. current ratio d. both options (a) and (b)

9.4.3 DEBT-EQUITY RATIo


The debt-equity ratio relates debt to equity or owners'
funds. Debt here
means long-term liabilities that mature after year and include
1 long-term
loans from financial institutions and banks, public deposits and debentures.
Equity means owners' funds and includes equity share capital, preference
share capital, general reserves, capital reserves, share premium
and other
reserves available to equity shareholders. Accumulated losses and
fictitious
assets such as preliminary expenses, discount on issue of shares or
deben-
tures, which are yet to be written off, should be deducted from the equity.
The debt-equity ratio is calculated as follows:

Debt
Debt-equity ratio =- Equity

For Asian Paints,

395 0.01
Debt-equity ratio = 36,009 =

for 2013-14; and


463 = 0.01
Debt-equityratio= 30,222

for 2012-13.

of debt in its capital structure. This ratio


San Paints has a very small amount
enjoyed by long-term lenders. The lower the
dicates the degree of protection to the lenders. A debt-equity
atlo, the higher will be the degree of protection as
auo of 2:1 is considered satisfactory. For capital-intensive industries such
ratio is allowed.
up-building, power units, cement units etc. a higher

NMIMS Global Access School for Continuing Education


RATIO inter
9.4.4 1NTEREST CoVERAGE interest obligations to protits before 1s cOvered
relates obligation more than
he interest coverage ratiothe number of times interestto have protits
tax and indicates
est andprofits for the period. It is always desirable lenders is unsafe. This
y the the position of the
payable; otherwise
he nterest
ratio is calculated as follows: and tax
Profit before interest
Interest coverage ratio
=
Int erest

For Asian Paints,


17,386-66.7 times
Interestcoverageratio= 261

for 2013-14; and


15,464 50.5times
Interestcoverageratio 306

for 2012-13. that has improved in


healthy interest
coverage ratio
Asian Paints has a very
2013-14 over 2012-13. on page 162. These
2012-13 are not reported
The profit figures for the year
figures are given below:
and tax 15,158B
items
Profit before exceptional 306
Add: Finance cost
and tax 15,464
Profit before interest
for tax pur-
on profit
is a charge and is allowed as deduction This ratio is
Since interest before interest and tax.
poses, profit in the numerator is profit
expressed as a number and not as a percemtage.
financial position ratios for
a comparison of Asian Paint's
Table 9.3 provides and Table 9.4 presents the com-
its peers
the year ended March 31, 2014 with period 2010-2014.
pany's position ratios over the 5-year
financial
POSITION OF ASIAN PAINTS AND
TABLE 9.3 TESTS OF FINANCIAL
PEERS
ITS AS ON MARCH 31, 2014
Measure of Financial Asian Akzo Berger Kansai
Position Paints Nobe1 Paints Nerolac
Current ratio (times) 1.43 1.23 1.36 1.79
Quick ratio (times) 0.83 0.94 0.73 0.84
Debt-equity ratio (times) 0.01 0.10 0.01 0.06
Interestcoverage ratio (times) 66.7 136.6 10.6 681.7
Short-term liquidity position of Asian Paints is comparable with its peers. All
the major companies in the paints industry employ a very small amount of
debt in their capital structure. Interest coverage ratio of Asian Paints is quite
high though Akzo Nobel and Kansai Nerolac have a much higher ratio.

NMIMS Global Access School for Continuing Education


E94
TABL 9.4 TESTS OF FINANCIAL POSITION OF ASIAN
PAINTS FOR
THE PERIOD 31.3.2010-31.3.2014
Measure of Financial
Position 31.3.2010 31.3.2011 31.3.2012 31.3.2013 31.3.2014
Current ratio (times) 0.92 0.99 1.29 1.29 1.43
Quick ratio (times) 0.40 0.38 0.69 0.66 0.83
Debt-equity ratio 0.04 0.03 0.02 0.01 0.01
(times)
Interest coverage 79.4 74.2 45.2 50.6 66.7
ratio (time)

mao trend of the current ratio and the quick ratio shows an improved liquid
ity position of the company. The solvency position of the company is strong
a high
as it has a very low amount of debt. This is also reflected in interest
coverage ratio.

4. Whichof the following would normally be included in the calculation


SELF ASSESSMENT
QUESTION
of debt-equity ratio?
a. Debentures b. Preference shares
c. Bank overdraft d. All of the above

TIP
9.5 RATIOS INVOLVING SHARE INFORMATION QUICK
Investors in equity shares are more interested in the return from their invest- Current ratio and Quick ratio
ment in the form of dividend and price appreciation. Ratios such as Dividend are used to measure short-
Payout, Dividend Yield and Price Earnings ratio that capture the relation- term liquidity. Debt-equity
ship among dividend, earnings and market price of share are of particular ratio and Interest coverage
interest to existing and potential investors in a company's shares. ratio are used to measure long
term solvency.
9.5.1 DIVIDEND PAYoUT RATIO
Ihe dividend payout (D/P) ratio measures the relationship between the earn-
ings belonging to equity shareholders and the dividend paid to them. It can
becalculated in one of the following two ways:

Total dividend paid


to equity shareholders -x100
Dividend payout ratio =
Total net profit belonging
to equity shareholders

For Asian
Paints,

Dividend payoutratio=,064100
11,690
= 43.5%
share (DPS)
Dividend per equity K 100
i) Dividend payout ratio - (EPS)
Earnings per share
5.30 100
12.19
=43.5%
policy followed by the company, and the
s ratio reflects the dividend
profits are retained in the business
(which can be determined
to be low for a
which
Dy deducting the D/P ratio from 100). The
D/P ratio is likely
large amount of funds for
company as such a company would require many profitable investment
OWh
Temvestment. A mature company that has not
opportunities is likely to have a higher D/P ratio.

9.5.2 DIVIDEND YIELD


provided by the div.
dividend yield ratio indicates the percentage return follows:
he
Tdend on the market price of the share and is
calculated as
Dividend per share x 100
DividendyieldMarket price per share

purchases the share


For Asian Paints, the dividend yield for an investor who
on March 31, 2014 for Rs. 473.05 is

Dividend yield =
Rs.5.30x100
Rs.546.50
0.97%

Alow dividend yield may mean that either the investors expect the dividends
to grow rapidly or the share is overpriced. A high dividend yield may indi-
cate that investors consider investment in the company's share to be a risky
investment or the share is underpriced. Dividend yield should not be inter-
preted as expected return from the share. There is another component of
returns from investment in a share: the change in price over the holding
period.

SELFASSESSMENT 5. Given that earnings per share = Rs. 50, dividend payout ratio = 40%,
QUESTION dividend yield = 3.2%. The price of ordinary shares implied by the
above data is
a. Rs. 78 b. Rs. 625
C. Rs. 1,563 d. Rs. 3,906

9.5.3 PRICE/EARNINGS RATIO (P/E RATIO)


Market price of the share incorporates everything
the company. Earnings are the net profit the market knows about
Relating the market price to the earnings available to equity shareholders.
gives an insight into how the inves
tors judge the performance of the concern.

NMIMS Global Access School for Continuing


Education
calculated as follows:
u is

Price/Earnings ratio = arket price per share


Earnings per share

rAsian Paints,
ForA P/E ratio for 2013-14 is Rs. 546.50/Rs. 12.19 = 44.83

The arnings
per shard
are used in the denominator can be last year's figure or
forecasted figure for the next year.
the

A high
P/E ratio suggests that the share is an attractive investment in the
eyes finvestors. The
e attractiveness may arise out of the belief that the share
orries a low risk or that the earnings are expected to grow quickly in the
tre. An unduly high P/E ratio relative to companies with similar risk-
future.
return profile may mean that the share is overpriced.
This measure is not under the direct control of the company. But if there is a
Hecline the P/E ratio of a company without a general decline in the stock
in
market prices, it becomes a cause of concern for the management.

6. Which of the following ratio is considered to assess the likely growth


SELF ASSESSMENT
QUESTION
prospects of the company and whether the company is a low risk
investment?
a. Earnings per share b. Diluted earnings per share
C. Price-earnings ratio d. Dividend yield

9.6 LIMITATIONS OF RATIO ANALYSIS


Ratio analysis fails to take into account many qualitative factors that affect
acompany's performance and future prospects. For example, ratios do not
capture the size effect. Large companies have better bargaining power and
enjoy economies of scale. Notes to the accounts contain important informa-
tion which is not reflected in ratios. For example, contingent liabilities and
commitments faced by the company.
Different accounting policies followed by companies in respect of deprecia-
inventory valuation and other matters can distort comparison among
tion,
companies. Ratio analysis also ignores the effect of industry characteristics
on profitability.

Some companies deliberately manipulate financial statements by creative


aCcounting and window-dressing. Ratio analysis becomes useless in such cases.

SELFASSESSMENT
7. Financial analysis of a business may not be able to achieve any one QUESTIONS
of the following issues:
a. Improve the profitability of the project.
b. Delineate the risks involved in the project.
C. Highlight the salient factors that lead to the greatest uncertainty.
d. Possibly suggest methods by which the risks might be reduced.
180 FINANCIAL ACCOUNTING AND ANALYSIS

by an error in
8. Which of the following financial ratios will be affected
recording the value of inventory in the financial statement?
() Inventory turnover ratio (ii) Current ratio
(iii) Earnings per share (iv) Interest coverage ratio
a. Option (i) only b. Options (i) and (ii)
C. Options (i), (ii) and (iii) d. All of the above

9.7 SUMMARY
profitability, eff
Understand and compute ratios used in analyzing standing of a
ciency in asset utilization, financial position and market
to check if the company is gen-
company. The profitability ratios are used
Widely used measures of prof
erating an acceptable return for its owners. return on capital
per share (EPS),
itability include profit margins, earnings on equity ROE).
employed (ROCE), return on assets (ROA) and return
measure the effec-
Ratios used to measure efficiency in asset utilization
or assets at its disposal.
tiveness with which a concern uses the resources ratio, inventory
Main ratios in this category include debtors'
turnover
and cash-to-cash operating
turnover ratio, creditors turnover ratio,
cycle.
and long-
Tests of financial position include tests of both short-term on the
term solvency of the business. Tests of short-term solvency focusmeasure
important ratios used to
liquidity position of the company. Two
long-term
short-term liquidity are: current ratio and quick ratio. Tests of repay
solvency focus on the ability of the company to pay interest and
principal of its long-term borrowings. The main ratios in this category
are: debt-equity ratio and interest coverage ratio.
of
OThe market standing of the company is reflected in the market price
the share. Ratios such as dividend payout, dividend yield and price earn-
ings ratio capture the relationship among dividend, earnings and market
price of share.
O Understand limitations of ratio analysis. Ratio analysis fails to
take
into account the size and contingent liabilities of the company. Ditferent
accounting policies followed by companies in respect of depreciation,
inventory valuation and other matters can distort comparison among
companies.

KEYKEY WORDS 1. collection period shows the accounts receivables in terms


Average
of number of days of credit sales during a particular period. It is
calculated dividing 365 days by debtors' turnover ratio.
2. Current ratio is the relation of a company's current assets to its
current liabilities. This ratio establishes the ability of the business to
meet its short-term obligations.
2 Debt-equity ratio relates debts to equity
measures the ability of the business to meet or owners' funds, and
its long-term obligations.
4. External analysis is the analysis carried out by parties external to
organization such as investors, credit rating the
agencies, government
agencies etc.
5. Interest coverage ratio relates interest obligations to the profits
before interest and tax and indicates the number of times interest
obligation is covered by the profits for the period.
G. Inventory turnover ratio relates the cost of goods sold to the average
stock. It measures how many times the average stock is sold during
the year.
7. Price-earnings (P/E) ratio compares the market price per share to
the earnings per share. It is calculated as the market price per share
divided by earnings per share.
8. Quick ratio is a more precise measure of liquidity than the current
ratio. Quick ratio relates quick current assets to current liabilities.
Quick current, assets are current assets minus inventories. Quick
ratio is also known as "Acid Test Ratio" or "Liquid Ratio".
9. Return on assets relates profit to investment in all the financial
resources, that is, owners' equity, long-term liabilities and current
liabilities.
10. Return on capital employed measures the returns generated by the
business on total of owners' funds and non-current liabilities.

9.8 DESCRIPTIVE QUESTIONS


1. List the possible reasons for high P/E ratio of a share.
2. Which of the profitability ratios is the most reliable for analysis and why?

3. Explain the limitations of financial ratio analysis in the


interpretation
of the financial statements of a company.
. Explain how P/E ratio and dividend yield ratio
can be used in
recommendations
tormulating appropriate equity investment
Briefly explain the reasons for
Define current ratio and quick ratio.
caleulating these ratios.
. Out ofa mature and a growth company,
which company is likely to have
a higher dividend payout ratio and why?
inventory turnover ratio and explain
Define debtors' turnover ratio and
their use.

School for Continuing Edueation


NMIMS Global Access
9.9 ANSWER KEY
SELF-ASSESSMENT QUESTIONS
Topics QNo. Answers
Tests of Efficiency in Investment 1. b. Earlier the business
had a
Utilization (Efficiency Ratios) zero debt collection period
Tests of Financial Position 2. d. Long-term investments of
the company
3. d. both options (a) and (b)
4. d. All of the above
Ratios Involving Share 5. b.Rs.625
Information
6. c. Price-earnings ratio
Limitations of Ratio Analysis 7. a. Improve the profitability of
the project
8. d. All of the above

9.10 SUGGESTED BOOKS AND E-REFERENCES


SUGGESTED BOOKS
aDatt G. and A. Mahajan (2012): Datt and Sudharam Indian Economy
(New Delhi: S. Chand & Company Ltd.).
aDepartment ofIndustrial Policy & Promotion.
(2010-11). Annual Report
2010-11. Ministry of Commerce and Industry, Government of India.

E-REFERENCES
Food and Agriculture Organisation, Statistical Database, Various years,
http://faostat.fao.org accessed on 30 April, 2011.
Indiastat, Statistical database, various years, www.indiastat.com accessed
on 2 August 2011.

NMIMS Global Access- School for Continuing Education


10
CHA PT ER
ANALYSIS OF FINANCIAL STATEMENTS II

CONTENTS

10.1 Introduction
10.2 Techniques of Financial Analysis
10.3 Common-Size Analysis
Self-Assessment Questions
Activity
10.4 Trend Analysis
Activity
10.5 Percentage Change Analysis (Comparative Financial Statements)
Self-Assessment Questions
Activity
10.6 Management Discussion and Analysis
10.7 Thinking Beyond Numbers
10.8 Quality of Earnings
10.9 Sustainable Income
Solved Problems
Self-Assessment Question
10.10 Summary
Key Words
10.11 Descriptive Questions
10.12 Answer Key
Self-Assessment Questions
10.13 Suggested Books and E-References
INTRODUCTORY CASELET

ALPHA LIMITED

The CFO of Alpha Limited is looking at the summary income statement


of the company for the year ended on March 31, 2019.

Summary Income Statement of Alpha Limited for the year ended on


March 31, 2019:

March 31, 2019 March 31, 2018


(Rs. Million) (Rs. Million)
Sales revenue 2,079 1,890
Operating expenses 1,351 1,134
Interest expense 313 284
Profit before tax 415 472

The CFO is trying to analyze why, despite a 10% increase in sales rev-
enue over the last year's figure, the profit before tax has declined by
nearly 12% compared to the previous year.

QUESTION
1. Analyze the reason for the decline in profit before tax in the
current year from that of the previous year. (Hint: Prepare
common-size income statement for the 2 years.)
LEARNING OBJECTIVES

After reading chapter,


this you will be able to:
Understand the objectives of financial analysis.
Describe and perform horizontal, vertical and trend analysis.
Understand the concept of quality of earning.
Understand the eoncept of sustainable income.

10.1 INTRODUCTIONN
Financial statement analysis involves rearrangement of financial informa-
tion, comparison, analysis and interpretation of that information. It can be
external or internal; horizontal or vertical; and intra-firm or inter-firm.
Analysis done by the management to assess the financial health of the organi-
efficiency is called internal analysis. Analysis car
zation and its operational such as investors, credit rating
ried out by parties external to the organization
agencies, government agencies etc. is called external analysis. Horizontal
analysis compares financial data overa number of years to analyze the trend.
data of a particular year. Inter-firm
Vertical analysis is based on the financial get an idea of
variables of two or more firms to
analysis compares financial compares the perfor-
position. Intra-firm analysis
their relative competitive
mance of different units of the same firm.

10.2 TECHNIQUES OF FINANCIALtheANALYSIs


financial statements:
analyzing
The following techniques can be used for
1. Vertical analysis (common-size analysis)
2. Horizontal analysis
(a) Trend analysis
(b) Percentage change analysis
3. Ratio analysis
4. Quality of earnings
. Sustainable income
9 of the book. The other tech-
o analysis has been covered in Chaptercovered in this chapter.
ues of financial statement analysis
are

10.3 COMMON-SIZE ANALYSIS


analysis, can be used to com-
Onnon-size analysis, also known as vertical to identity variations which
pare statements of two periods
he financial further analysis. In common-size analysis, all
figures
the basis for as a percentage of a common base,
of
whnancial
statement are expressed base is the sales figure in the case of
Ch is taken as 100. This common
Slatement of Profit and Loss and the total of assets or of liabilities in the case
10.1 and
O Salance Sheet. Common-size balance sheets are shown in Table
common-size income statements in Table 10.2.

SELF ASSESSMENT
QUESTIONS
By what other name common-size analysis known as?
is
a. Vertical analysis b. Directional analysis
C. Horizontal analysis d. Trend analysis
2. Expressing each item of a balance sheet as a percent is an example
of
a. Trend analysis b. Common-size analysis
C. Horizontal analysis d. Comparative analysis
3. In vertical analysis, each item in a financial statement is expressed
as
a. An amount in Rupees
b. A percent of some base figure
C. A percent of the amount
of the same item in the preceding year
d. An amount in Euro
4. The relationship of components of a financial
statement is best
expressed by preparing:
a. A common size statement b.
A trend analysis report
C. An analysis of ratios
d. A profit-loss analysis

TABLE 10.1 COMMON-SIZE


BALANCE SHEETS
2018 (Rs.)
Equity share capital 2019 (Rs.)
10,000,000
52.8 12,500,000 52.7
Reserves and surplus
4,825,000 25.5
Long-term loans 6,250,000 26.3
2,000,000 10.6
Accounts payable 2,500,000 10.5
1,250,000
Taxes payable 6.6 1,500,000 6.3
850,000
Total current liabilities 4.5 1,000,000
2,100,000 4.2
Total liabilities and stockholders' 11.1 2,500,000
18,925,000 10.5
equity 100.0 23,750,000 100.0

Land
=====
=:
4,375,000
Property, plant and equipment 23.1 5,000,000 21.1
2,375,000
12.6
5,375,000 22.6

NMIMS Globul AccessSe


School for Continuing
Education
TABLE 10.1 COMMOON-SIZE BALANCE SHEETS-CONTINUED
2018 (Rs.) % 2019 (Rs.) o
Accumulated depreciation (2,000,000) (10.6) (2,625,000) (11.1)
Long-term investments 1,875,000 9.9 2,750,000 11.6
Cash 2,750,000 14.5 3,250,000 13.7
Accounts receivable 3,000,000 15.9 3,250,000 13.7

Inventory 5,375,000 28.4 5,625,000 23.7


Prepaid expenses 1,175,000 6.2 1,125,000 4.7

Total current assets 12,300,000 65.0 13,250,000 55.8


Total assets 18,925,000 100.0 23,750,000 100.0

The common-size balance sheets indicate the growing size of the business
ofthe company. The assets have increased from Rs. 18.93 million in 2013 to
Rs.23.75 million. New investment has mainly been made in property, plant
and equipment, their percentage of total assets has increased from 12.6%to
22.6%. Other assets have also increased in absolute terms but not in percent
terms because of a higher increase in the overall size of the balance sheet.
A major part of the funds for this investment have come from retained
profits which are reflected in increase in reserves and surplus to 26.3% of
equity and liabilities from 25.5% in the previous year. Equity share capital
and long-term loans have also increased in absolute amount but not in
percent terms.
Composition of assets and liabilities has changed. The share of current QUICK TIP
assets in the total assets has declined from 65% to 55.8% with a corre-
liabilities In acommon-size balance
ponding increase in the share of non-current assets. Current in sheet, all items of the balance
nave declined 0.6%o from 11.1% to 10.5% with a corresponding increase
to release sheet are expressed as
1On-current liabilities. This implies that the company is able percentage oftotal assets.
efficient management
Ore funds for long-term investment with a more
of its working capital.

SELFASSESSMENT
100% figure?
common size balance sheet, which item represents the
naa. Total assets b. Total current assets
QUESTIONS

C. Total non-current assets d. None of the above


property, plant and
The base figure in terms of which value of
statement is:
equipment is expressed in a common size
a. Total non-current assets b. Total assets
C. Net sales d. Total revenue
RACTIvITY 1 summary financial information
for the
ABC Ltd. provides the following
year 2018 and year 2019.

2018 2019

Rs. Million Rs. Million


4,800 5,280
Share capital
3,801
Non-current liabilities 3,360
720 633
Current liabilities
720 846
Investments
3200 3,300
Current assets
4960 5,568
Non-current assets
for the two years.
Prepare the common-size balance sheet of the company

TABLE 10.2 COMMON-SIZE INCOME STATEMENTS


2018 (Rs.) 2019 (Rs.)
16,250,000 100.0 17,500,000 100.0
Sales revenue
Cost of goods sold 11,375,000 70.0 12,500,000 71.4

Gross profit 4,875,000 30.0 5,000,000 28.6


Employee benefit expenses 1,056,250 6.5 1,250,000 7.1

Other administrative 1,175,000 7.2 1,200,000 6.9


expenses
Depreciation 375,000 2.3 875,000 5.0
Total expenses 2,606,250 16.0 3,325,000 19.0
Operating income 2,268,750 14.0 1,675,000 9.6
Interest expense (125,000) (0.8) (175,000) (1.0)
Other income 1,250,000 7.7 10.7
1,875,000
Net income before tax 3,393,750 20.9 3,375,000 19.3
Tax (1,006,250) (6.2) (1,000,000) (5.7)
Net income 2,387,500 14.7 2,375,000 13.6

Common-size income statements of the company


QUICK TIP cnst of goods sold and an show an 1.4% in
ome has declined by 4.40. increase ot 8.0% in operating increase of
Interest expense has expenses. Operating
In acommon-size income also increased by 0.20
statement, all items of However due to inerease ln otner lncome by 3%
exDense, the decrease in net income is and decrease of 0.5o in tax
the income statement are only 1.1%.
expressed as percentage of
total revenue.

lobal Aneess- Sehool fo Cr


SELF-ASSESSMENT
mhebase that is used to express the percent material consumption in
7 common-Size analysis is: QUESTIONS
assets
a. Fixed b. Net sales
c. Total assets d. Fixed assets
oTn
a common size income statement, which item represents the 100%
figure?
a. Cost of goods sold b. Net sales
c. Net income d. Total expenses
9. The base figure in terms of which cost of goods sold is expressed in
a common size statement is
a. Total selling expenses b. Total revenue
c. Total expenses d. Net sales
10. Which of the following is used to represent sales revenue in a
common size statement?
a. Net income b. Cost of goods sold
C. Net sales d. Total revenue

10.4 TREND ANALYSIS


These are useful for making a comparative study of the financial statements QUICKTIP
over a number of years. The earliest year used for comparison is treated as
the base year. The base year figure for each item of the financial statements In trend analysis, the base year
is taken as 100. The figures for the subsequent years are expressed as per figure for each item of the
centages of the base year figure. An illustration of trend percentages is given financial statements is taken
in Table 10.3. as 100.

TABLE 10.3 TREND ANALYSIS


Amount (Rs. Millions) Trend Percentage
2018 2015 2016 2017 2018
2015 2016 2017
Income Statement
Net Sales 118.0 115.5 121.7
94,300 111,300 108,890 114,810 100.0
Operating 84.8 78.4
EBITDA 19,210 21,960 16,290 15,070 100.0 114.3
Profit 94.2 79.7 73.7
before Tax 15,400 14,510 12,270 11,350 100.0
Profit 80.0 82.7 88.2
after Tax 13,250 10,610 10,960 11,680 100.0
Balance
Sheet
Shareholders
S Funds 71,920 73,830 78,250 82,360 100.0 102.7 108.8 114.5
Borrowings 6.8
0 100.0 31.9
5,110 1,630 350
NetFixed 93.9 96.2 114.3
Assets 65,730 61,750 63,240 75,130 100.0
Cash
16,860 100.0 107.0 89.4 57.5
29,320 31,370 26,210
Current 94.4
Assets 34,850 100.0 83.9 94.2
36,910 30,980 34,760
Current
liabilities 37,260 39,000 100.0 102.5 98.9 103.5
37,680 38,630
Capital
Employed 82,210 80,630 83,670 87,710 100.0 98.1 101.8 106.7

NMIMS Global Access School for Continuing Education


190 FINANCIAL
ACCOUNTING
AND ANALYSIS

ACTTvIrY
2 Alpha Limited
rupees) for the reports the following
last five years: amounts ofsales
revenue (in million
Year
2015
Sales revenue 2016 2017
50 2018 2019
55 60
Present the above 63
58
the year 2015 as information in the form of a
the base. trend analysis report
using

10.5 PERCENTAGE
CHANGE ANALYSIS
(COMPARATIVE
Figures
FINANCIAL STATEMENTS)
of two or
more periods are
ute as well as percentage placed side by
to derive change in the figures side. Comparison of abso-
meaningful over the periods is
horizontal analysis. conclusions. Percentage change analysis made
Table 10.4 and of An illustration of comparative is a type of
comparative income statements balance sheets is given in
in Table 10.5.

TABLE 10.4 COMPARATIVE


BALANCE SHEETIS
Rupee
2018
Change T% Change
2019 Increase Increase
Rs. Rs. (Decrease) (Decrease)
Equity share capital 10,000,000 12,500,000 2,500,000
Reserves and Surplus 25.0
4,825,000 6,250,000 1,425,000 29.5
Long-term Loans 2,000,000 2,500,000 500,000 25.0
Accounts Payable 1,250,000 1,500,000 250,000 20.0
Taxes Payable 850,000 1,000,000 150,000 17.6
Total Liabilities and
Stockholders' Equity 18,925,000 23,750,000 48,25,000 25.5
Land 4,375,000 5,000,000 625,000 14.3
Property, Plant & 2,375,000 5,375,000
Equipment 3,000,000 126.3

Accumulated Depreciation (2,000,000)) (2,625,000)


(625,000) (31.3)
Long-term investments 1,875,000 2,750,000 875,000 46.7
Cash 2,750,000 3,250,000 500,000 18.2
Accounts receivable 3,000,000 3,250,000 250,000 8.3
Inventory 5,375,000 5,625,000 250,000 4.7
Prepaid expenses 1,175,000 1,125,000 (50,000) (4.3)
Total current assets 12,300,000 13,250,000 950,000 7.7
Total assets 18,925,000 23,750,000 4,825,000 25.5
TABLE 10.5 COMPARATIVE INCOME
STATEMENTS
Rupee Change % Change
2018
Increase Increasee
2019 (Decrease) (Decrease)
Sales revenue 16,250,0000 17,500,000 1,250,000 7.7
Cost of goods sold 11,375,000 12,500,000 1,125,000 9.9
Gross Profit 4,875,000 5,000,000 125,000 2.6
Employee benefit 1,056,250 1,250,000 193,750 18.3
expenses
Other administrative 1,175,000 1,200,000 25,000 2.1
expensesS
Depreciation 375,000 875,000 500,000 133.3
Total expenses 2,606,250 3,325,000 718,750 27.6
Operating income 2,268,750 1,675,000 593,750 (26.2)
Interest expense (125,000) (175,000) (50,000) 40.0
Other income 1,250,000 1,875,000 625,000 50.0
Net income before tax 3,393,750 3,375,000 (18,750) (0.06)
Tax (1,006,250) (1,000,000) 6,250 (0.06)
Net income 2,387,500 2,375,000 (12,500) (0.05)

SELF-ASSESSMENT
11. Horizontal analysis is based on data pertaining to C QUESTIONS
a. One period of time b. Anumber of periods
C. A particular date d. None of the above
12. Comparative financial statements:
a. Are prepared for one year
b. Are prepared for at least 2 years
C. Show only the rupee amount
d. Present comparison of only balance sheet items

ACTIvITY 3
following data taken
Pr
pare a comparative income statement from the
rom the financial statements of Kohinoor Foods Limited.
Rs. (million)
2019 2018
700 600
Net Sales
602 510
Cost of Goods Sold
98 90
Gross
profit

NMIMS Global Access- School for Continuing Edueation


QUICK TIP
Management Discussion
10.6 MANAGEMENT DISCUSSION AND ANALYSIS
Analysis (MD&A) and agement Discussion and Analysis (MD&A) Report is a requirement of
Report is the listing agreement of the company with the stock exchange. This require-
not a requirement
Companies Act, under the ment, therefore, applies to a listed company only.
2013, but
of the listing agreement Clause 49 of the listing agreement provides that as part of the directors
the company with of
the stock eport or as an addition thereto, a MD&A report should form part of the
exchange. Annual Report to the shareholders. This MD&A should include discussion
on the following matters within the limits set by the company's competitive
position:
1. Industry structure and developments
2. Opportunities and
Threats
3. Segment-wise or product-wise performance
4. Outlook
5. Risks and concerns
6. Internal control systems and their adequacy
7. Discussion on financial
performance with respect to operational
performance
8. Material developments
in Human Resources/Industrial Relations
including the number of people employed. front,

MD&A report provides useful


insights about the industry in
pany operates, its competitive
position in the industry,
which a com-
the company and the risks facing
the company.
future prospects of

10.7 THINKING BEYOND NUMBERS


Ratio Analysis and other tools of
many qualitative factors that financial analysis fail
affect a company's
to take into account
prospects. For example performance
ratios
nies have better bargaining do not capture the size effect. and future
accounts contain important
power and enjoy Large compa-
information
economies of scale. Notes to
example contingent liabilities which is the
and commitments not reflected in ratios. For
Different accounting policies faced by the company.
followed by
tion, inventory valuation companies in respect
companies. Ratio and other matters can of deprecia-
analysis also ignores distort comparison among
on profitability. the effect of
industry characteristies
Came eompanies deliberately
accounting and manipulate financial
window-dressing.
cases. A proper analysis Ratio analysis statements by creative
company ot 1inancial
performancebecomes useless in sueh
se requires looking
are measurement beyond numbers.
ot quality
of earnings
and financial
Two important position of
tools for
and sustainable income.th
10.8 QUALITY
OF EARNINGS
The term quality of arnings
is used
choices. It is the degree to which in the
managers,context of making accounting
when faced
with a choice of
NMIMS lobal Access School for Continuing
Edueation
high impact on earnings, choose items that result in income
items that have a
.

gnition that is more likely to lead to recurring patterns of income.


rnore likely an item of income is to recur, the higher its quality will be.
Tb
generat confidence
ce of investors and lenders in financial reports and to
apital markets efficient, it is necessary that the earnings reported by
make
companies are of high quality.

Asolity of earnings suffers when


Qua earnings are managed. Earnings manage-
nent is planned
ment timing of revenues, expenses, gains and losses to smooth net
income. The following ways are generally used to manage earnings:

1. Use of one-time items such as sale of investments or fixed assets.


2. Inflate revenues in the short-run by providing sales incentives to achieve IMPORTANT CONCEPT
higher sales at the end of the year. These goods might be returned by
the customers in the next year. Earnings management is
planned timing of revenues,
3. Use of improper adjusting entries, for example, recognizing revenue expenses, gains and losses to
on multi-year contracts too quickly and treating operating expenses as smooth net income.
capital expenditure.

10.9 SUSTAINABLE INCOME


For forecasting future cash flows, analysts need to ensure that current earn- !IMPORTANTCONCEPT
ings do not include irregular items. Net income adjusted for irregular items is
called Sustainable Income. It is the most likely level of income to be achieved Sustainable income is the net
in the future. It differs from the actual net income by the amount of irregular income adjusted for irregular
revenues, expenses, gains and losses. items.
Two major irregular items are as follows:
!IMPORTANT CONCEPT
L.
Discontinued Operations
2.
Extraordinary items Discontinued operations refer
to the disposal of a significant
DISCOntinued Operations refer to the disposal ofa significant part of the busi part of the business or an
uSS or an activity. As the results of discontinued operations will
not be a part
activity.
entity's results in the future, the entity is required to report
the profit
an
and loss from
discontinued operations net of tax. IMPORTANT CONCEPT
Eaxtr are both unusual in
Ordanary items are events and transactions that by
occur infrequently, for example, loss caused a natural calamity Extraordinary items are events
and crop due to weather
not usual in a particular area. However, loss to examples of
and transactions that are both
not
an extraordinary item as it is not infrequent. Other government, unusual in nature and occur
foreign
rdinary items are expropriation of property by a infrequently.
and effect
of a newly elected law.
SOLVED PROBLEMS
following information for the
Amar Raja Limited provides the
immediately preceding two years:

2019 2018

Rs. Million Rs. Million

Sales 5,000 3,750

Cost of goods sold 3,000 2,450

Operating expenses 750 490

Financial expenses 500 340

Income tax 150 95

Net profit 600 375

Prepare Comparative Income statement of the company for the two-


year period.
Solution
Comparative Income Statements of Amar Raja Limited for the
Years 2018 and 2019
2018 2019 Rupee To Change
(Rs. Million) (Rs. Million) Change Increase
Increase (Decrease)
(Decrease)
Sales revenue 3,750 5,000 1,250 33.33
Cost of goods sold 2,450 3,000 550 22.45
Gross Profit 1,300 2,000 700 53.85
Operating (490) (750) (260) 53.06
expenses
Operating income 810 1,250 440 54.32
Financial (340) (500) 160 47.06
expenses
Net income 470 750 280 59.57
before tax
Income tax (95) (150) 55 57.89
Net profit 375 600 225 60.00
The following items appear in the financial statement of Steelco Limited.

Rs. Million
2018 2017 2016
Net Sales 169 162 150
Cost of Goods Sold 111 105 94
Gross profit 58 57 156
Prepare a trend analysis statement for three years treating 2016 as the
base and comment on the results.
Solution
Trend Analysis of Gross Profit of Steelco Limited for
the period 2016-2018
Amount (Rs. Million) Trend Percentages
2016 2017 2018 2016 2017 2018
Net sales 150 162 169 100.0 108.0 112.7
Cost of goods sold 94 105 111 100.0 111.7 118.1
Gross profit 56 57 58 100.0 101.8 103.5
Net Sales, Cost of goods sold and gross profit show an increasing trend.
Increase in sales is higher than increase in cost of goods sold due to
which gross profit is increasing.

Prepare common-size income statement from the following data taken


from the financial statements of Xi Limited for the years 2018 and 2019.

Rs. Million
2019 2018
Net Sales 540 475
Cost of Goods Sold 405 378
Gross profit 135 97
Solution

Common-Size Income Statement of Xi Limited for


the Years 2018 and 2019
2018 2019
(Rs. Million) (Rs. Million)
Net sales 475 540
100.0 100.0
Cost of goods sold
Gross profit
378 79.6 405 75.0
97 20.4 135 25.0
SELFASSESSMENT
QUESTION l3. Disposal of a significant part of the business or an activity 1s known
as
a. An exceptional item b. An extraordinary item
C. Discontinued operations d. Cessation of business

10.10 SUMMARY
Understand the objectives of financial analysis. The purpose of finan-
Cial statement analysis is to determine the meaning and significance
of
the data contained in the statements so that a forecast may be made of
future earnings and financial position.
analysis.
Describe and perform horizontal, vertical and trend
years to
Horizontal analysis compares financial data over a number of
analyze the trend. Comparative Balance Sheets and Comparative Profit
analysis. Vertical
and Loss Statements are used to perform horizontal
analysis is based on the financial data of a particular year. It carried
is
out by preparing common-size Balance Sheet and common-size Income
Statement.
Understand the concept of quality of earnings. Quality of earnings indi-
cates the degree to which full and transparent information is provided to
the users of financial statements. Quality of earnings suffers when earn-
ings are managed. Earnings management is the planned timing
of reve-
nues, expenses, gains and losses to smooth the net income.
O Understand the concept of sustainable income. Net income adjusted for
irregular items is called sustainable income. It is the most likely level of
income to be achieved in the future

KEY WORDS 1.Common base In common-size analysis, all figures of a financial


statement are expressed as a percentage of a common base, which
is taken as 100. This common base is the sales figure in the case of
Statement of Profit and Loss and the total of assets or of liabilities in
the case of Balance Sheet.
2. Common-size analysis, also known as vertical analysis, can be
used to compare the financial statements of two periods to identify
variations which form the basis for further analysis.
3. Discontinued operations refer to the disposal of a significant part of
the business or an activity.
4. Extraordinary items are events and transactions that are both
unusual in nature and occur infrequently.
5. Financial statement analysis is the study of relationships between
the elements of the same statement or different financial statements
and the trend of these elements.
6. Horizontal analysis compares financial data over a
to analyze the
number of years
trend.
Ouality
7.
of earnings indicates the degree to which full and
transparent
information is provided to the users of financial
statements.
Sustainable income is the most likely level of income to be achieved
in the future. It differs from the actual net income by the amount of
irregular revenues, expenses, gains and losses.
Vertical analysis is based on the financial data of a particular year
and is carried out by expressing each item in the financial statement
as a percentage of a base amount.

10.11 DESCRIPTIVE QUESTIONS


Explain what kind of information can be obtained from common-size
analysis?
2 How is vertical analysis used to make competitive analysis?
3. Explain the difference between horizontal analysis and vertical analysis.
4. Provide three examples of the ways that companies use to manage
their
earnings.
5. Explain the term 'Sustainable Income'. How is sustainable incomne
related to the treatment of irregular items in the statement of profit
and loss?
6. Explain the limitations of financial ratio analysis in the interpretation
of the financial statements of a company.
7. When are the earnings of an entity considered to be of good quality?

10.12 ANSWER KEY

SELF-ASSESSMENT QUESTIONS
Topics Q. No. Answers
Common-Size Analysis 1. a. Vertical analysis
2. b. Common-size analysis
3. b. A percent of some base
figure
4. a. A common-size statement
5. a. Total assets
6. b. Total assets
7. b. Net sales
8. d. Total expenses
9. d. Net sales
10 c. Net sales
Percentage Change Analysis 11. b. A number of periods
Comparative Financial Statements)

NMIMS Global Access- School for Continuing Education


Topics Q. No. Answers
12. b. Are prepared for at least
2 years
Sustainable Income 13. c. Discontinued operations

10.13 SUGGESTED BOOKS AND E-REFERENCES


SUGGESTED BOOKS
Narasimhan M.S. (2016). Financial Statement and Analysis. Cengage
Learning India Private Limited; First edition.
Grewal T. S., Grewal H. S., Grewal G. S., Khosla R. K. (2019). T.S. Grewal's
Analysis of F'inancial Statements. Sultan Chand & Sons Private Limited.

E-REFERENCESS
Ahsan M. L(2019). CFA 2019 Level 1: Financial Reporting and Analysis:
Complete FRA in one week Kindle Edition.
Goel S. 2019). Financefor Non-Finance People. 2nd Edition, Kindle Edition.

NMIMSGlobal Access-School
for Continuing
Education
11
CH A
PT E R

CASE STUDIES

CONTENTS

Case Study 1: Modern Coffee House


Case Study 2: Aarti Enterprises
Case Study 3: Asian Chemicals Limited
Case Study 4: Remco Limited
Case Study 5: Rama Sales Corporation
Case Study 6: Fresca Limited
11
CHA PT ER
CASE STUDIES

CONTENTS
Case Study 1: Modern Coffee House
Case Study 2: Aarti Enterprises
Case Study 3: Asian Chemicals Limited
Case Study 4: Remco Limited
Case Study 5: Rama Sales Corporation
Case Study 6: Fresca Limited
ING AND ANALYSISs

CASE STUDY 1

MODERN COFFEE HOUSE

Modern Coffee House is a partnership firm formed on January 1, 2017, by


two persons Mr. Ram Ashish and Mr. Ram Naresh. Mr. Ram Ashish was
Working with a manufacturing company on the shop floor and had about
with a
10 years' experience. Mr. Ram Naresh was working as an ad hoc
victim of
multinational company for quite some time. He became the
sought retirement
downsizing and was finally retrenched. Ram Ashishcompany. The two,
under Early Separation Scheme (ESS) from the
a partnership in
enthusiastic and energetic, in their early 30s, formed is well connected
a well-developed township near Delhi. The township
number of multinational
with different parts of the country and a large
corporate offices there. Ram
and Indian companies have set up their
working and determined young
Ashish and Ram Naresh were very hard to them and they did not
men. However, running a coffee house was new
possess any commercial experience.
hired a small premises on rent,
They contributed Rs. 2,000,000 each,
utensils for Rs. 920,000, equip-
purchased furniture for Rs. 1,050,000, deposit of Rs. 1,050,000 with a
ment for Rs. 630,000 and made a securityproper accounting records but
keep
soft drinks company. They did not daybook. At the end of July 2017,
a
just maintained a cash register and other items as follows:
and
they found their assets, liabilities
Rs. 250,000 Bank Balance Rs. 1,000,000
Cash 630,000
920,000 Equipment
Utensils
2,600,000 Rent paid 150,000
Sale proceeds 1,050,000
Furniture
Total expense on
1,550,000
Food & beverages
to 31, 2018, activities of the I increased consid-
From August 1, 2017 July helpers and took a loan of Rs. 2,500,000 from
erably. They employed four
their takings and expenses were as follows:
a bank. During this period,

Materials Rs. 1,500,000 Repayment of Rs. 250,000


purchased bank loan
6,000,000 Traveling 160,000
Sale proceeds &
collections expenses
Expenses on 2,280,000 Interest on 250,000
eatables bank loan
Gas & fuel 360,000 Miscellaneous 36,000
expenses
Wages 60,000 Loan (balance) 2,250,000
Soft drink expenses 600,000 Withdrawals 400,000
Rent 300,000
CASE STUDIES 201

CASE STUDY 1

During this period, they maintained a simple cashbook and a daybook.


They also provided depreciation on fixed assets: utensils 20%, furniture
10%, and equipment 15%. The closing stock was Rs. 1,050,000.
The activities of Modern Coffee House expanded further. They started
Dacked lunch delivery to customers. At the end of July 2019, they found
7,700,000.
their revenues total to Rs.
Expense and other items were as follows:

Materials Rs. 40,00,000 Repayment of bank Rs. 2,50,000


purchased loan
4,80,000 Travelling expenses 2,32,000
Wages
Interest on bank loan 225,000
Gas & fuel 5,00,000
Miscellaneous 280,000
Soft drink 8,48,000
expenses expenses
8,00,000
Fixed deposits 3,00,000 Operating expenses
2,000,000
Rent paid 300,000 Loan balance
Drawings 5,00,000
employed an accoun-
As the business had expanded considerably, they they could prepare
books of accounts, so that
tant who could keep propermeasure business
statements and the performance of their
inancial accrual basis of accounting.
more accurately. The accountant adoptedJuly 31, 2019 were Rs. 360,000.
Wages and other outstanding
expenses on
furniture @10%, utensils @20T%, and
depreciation on
They provided of materials was Rs.
1,850,000.
otherequipment @ 15%. Stock

QUESTIONS 31,
of Modern Coffee House as on July
Draw up a balance sheet place up to that
2017, taking into
account the events that took
Rs., 4,900,000.)
date. (Hint: Total of
Balance Sheet
accounting8
Mr. Ram Ashish did not possess accountant
ZMr. Ram Naresh and
beginning thought that hiring an their small
the
skills and in
records would involve costs that view? Give
accounting their
Do you agree with a professional
LO keep
Dusiness could not afford. benefit of hiring
your comments. (Hint: The offset the cost by facilitating better
accountant will more than
decisions.) Modern
statement and a balance sheet of 2018.
raw up an income
Coffee House for the Balance Sheet total
on July 31,
accounting year ended Rs. 7,870,500.)
Hint: Profit Rs. 1,120,500; measurement
the two bases of income Comment.
Do you think that basis) are the same?
0.e., Cash basis and
acerual
Concept.)
(1Hint: Refer Chapter 3: Acerual
204 FINANCIAL
ACCOUNTING
AND ANALYSIS

CASE STUDY 2

QUEsTIONS
repare balance sheet of the company as on
the corrected
March 31, 2019. (a) Replace inventories of Rs.350
(Hint:
million under assets with inventories of Rs. 250 million. (b).
Adjust retained earnings for decrease in inventories and tor
outstanding expenses. (c). Show outstanding expenses as a new
item of liabilities.)
2.Based on the corrected balance sheet, state whether the
company met the two requirements of eligibility for the bank
0.94.)
loan. (Hint: The new current ratio 1.31; quick ratio
CASE STUDIES
205

CASE STUDY 3

ASIAN CHEMICALS LIMITED

At the tenth annual general meeting of


Asian Chemicals Limited, the
Chief Financial Officer (CFO) was presenting
the financial statements of
he company for the year ended March 31, 2019, before the
shareholders.
When the CFO was presenting the schedule of the balance
sheet relating
to'Other Equity, some shareholders requested the CFO to explain the
various components of Other Equity'.
The balance sheet schedule covering 'Other Equity' is presented in
the next page.

QUESTION
1. As CFO of Asian Chemicals Limited, explain each element of
Other Equity' to the shareholders. (Hint: Refer Chapter 7 for
components of 'Other Equity')
Notes to the Financial
Statements
Other Equity (Rs. Million)

Capital Debt Equity


Capital Redemption General Retained Instruments Instruments
Reserve Reserve Reserve Earnings through OCI hrough CI Total
Balance as at 31.03.2018 450 5 41,500 33,900 12 1,020 76,887
Additions during the year
21,350 21,350
Profit for the year
Items of OCI, net of tax
Re-measurement of defined (265) (265)
benefit plans
Net fair value gain on investment
95 95
in equity instruments through OCI
Net fair value (loss) on invest- (12) (12)
ment in debt instruments
through OCI
Total Comprehensive Income for 21,085 (12) 95 21,168
the year 2018-19
Reductions during the year
Dividends (8,540) (8,540)
Income tax on dividends (1,735) (1,735)
Total (10,275) (10,275)
Balance as at 31.03.2019 450 5 41,500 44,710 1,115 87,780
REMCO LIMITED

The Accountant of Remco


Limited (RL) has prepared the following
year ended March 31, 2019.
statement of sources and uses of cash for the
31,
Statement of sources and uses of cash for the year ended March
2019

Sources of Cash Rs. Million


Sale of goods and services 2,000
Issue of shares 2,310

Sale of investments 440


Borrowings from bank 110

Dividend received 303

Interest received 33

Total Sources of Cash (A) 5,196

Uses of Cash
1,419
Purchases of materials
1,815
Purchase of equipment
1,100
Operating expenses
Repayment of bonds 358
23
Interest paid
Total uses of Cash (B) 4,715
B) 481
Increase in cash balance (A -

performed very well as


The Accountant claims that the company has
reflected in the increase in the cash balance. The
accounts manager, how-
ever, infers that the operations of the company have
resulted in a loss. He
flow statement in proper form also
asks the accountant to prepare a cash
taking in account the beginning cash balance of Rs. 200 million.

QUESTIONS
year ended March
1, Prepare the cash flow statement of RL for the activities (Rs. 519
31, 2019. (Hint: Cash flows from: operating financing
million);
million); investing activities (Rs. 1,039
activities Rs. 2039 million.)
2, Comment on the performance of the
company during the year
in the cash balance.
and on the factors contributing to increasemillion; Ending cash
(Hint: Beginning cash balance Rs. 200
losses in
balance Rs. 681 million. 'The company has incurred
is due to raising of
operations. The increase in cash balance
additional financial resources from issue of shares.)
208 FINANCIAL
ACCOUNTING
AND ANALYSIs

CASE STUDY 5

RAMA SALES CORPORATION

Rama Sales Corporation


approached its bank on September 20, 2019,
for a term loan of Rs. 5 million.
its inconme statement for The bank asked the company to
the accounting year ended on March 31,submit
and a balance sheet on that 2019,
date by October 30, 2019. On October 3,
certain important records of the 2019,
an office. These records company were lost due to flooding of
included financial statements for the year
2019.
Management of Rama Sales started looking
them in constructing the financial for records that could help
statements required by the bank.
Fortunately, they were able to get
(Table 1), abridged balance the abridged income statement
year and some other information
sheet (Table 2) of the previous accounting
and (Table 3). The ratios were
calculated on the basis of the year-endratios
financial
ing to the current accounting information pertan-
year.
The management also remembered that the sales revenue
25% over that of 2018 and both gross in 2019 grew
margin and net profit margin per-
centage in 2019 was the same as in 2018.

TABLE 1 INCOME STATEMENT OF RAMA


SALES CORPORATION FOR THE YEAR
ENDED ON MARCH 31, 2018
Rs.
Sales 57,616,000
Less: Cost pf sales 43,212,000
Gross profit 14,404,00o
Less: Other expenses 8,642,400
Net profit 5,761,600
TABLE 2 BALANCE SHEET OF RAMA
CORPORATION AS ON MARCH 31, SALES
2018
Rs.
Cash and cash equivalents 2,560,000
Accounts receivable 3,640,000
Inventories
5,930,000
Non-current assets 19,785,000

Total assets
31,915,000

Current liabilities
11,150,000
Long-term loan
6,005,000
Equity
14,760,000

Total capital and Liabilities


31,915,000
CASE STUDIES 209

CASE STUDY 5

TABLE 3 SELECT FINANCIAL DATA AND RATIOS OF


RAMA SALES CORPORATION FOR THE YEAR ENDED
ON MARCH 31, 2019
turnover (times) 2
Total assets
Debt: equity ratio 0.60:1
Long-ternm liabilities/total assets 0.68:1
1.2:1
Current ratio
Quick ratio 0.67:1
Debtors' turnover (times) 13

QUESTION
1. Prepare the income statement of Rama Sales Corporation for
the year ended on March 31, 2019, and a balance sheet on that
date using the given information. (Hint: Solve following these
steps. 1. Calculate Sales for the year 2019; 2. Calculate total
assets at the end of year 2019; 3. Calculate long-term liabilities;
4. Calculate debt, equity and current liabilities; 5. Calculate
current assets; 6. Calculate debtors; 7. Calculate inventories;
8. Calculate cash balance; 9. Caleulate non-current assets; 10.
Calculate gross profit and cost of sales; 11. Calculate net profit
and other expenses. Gross Profit Rs. 18,005,000, Net profit
Rs. 7,202,000, Total assets Rs. 36,010,000, Current liabilities
Rs. 11,523,200, Current assets Rs. 13,827,840.)
CASE STUDY 6

FRESCA LIMITED

sheet of the company


Fres tor the
Llnmited has provided the balance
years 2018 and 2019 as follows:
2019 2018
Rs. Million Rs. Million
Assets
Non-current assets 2,046 1,997
Cash and cash equivalents 2,543 4,642
Short-term investments 8,171 9,768
Accounts receivable 3,235 3,085
Inventories 6,138 5,033
Total assets 22,133 24,525
Capital and Liabilities
Equity share capital 9,643 9,544
Retained earnings 9,251 12,833
Long-term loan 112 378
Accounts payable 2,354 913
Outstanding expenses 773 857
Total capital and liabilities 22,133 24,525

QUESTIONS
1. Prepare common-size balance sheet
comment on the results. (Hint: Express for the 2 years and
every
ofRs. 24,525 million in 2018 and of Rs. 22,133 item as a percent
million in 2019.)
2, Analyze the short-term liquidity
position
company facing any short-termliquidity of the company. Is the
Current ratio and uick ratio for problem? (Hint:
both the years.) Calculate
2 Identify the possible reasons
for the change
equity share capital and in the amount
the 2-year period. (Hint: retained earnings of the company of
from Changes in equity over
oceurissue or buyback of shares. Changes share capital result
due to retained profits, that in
paid.) is, profits retained earnings
earned less dividend

You might also like